You are on page 1of 273

2016 OITE

STUDY GUIDE
FOR RESIDENTS

Orthopaedic In-Training Examination®


2016 Orthopaedic In-Training Examination
Copyright  2016
by the American Academy of Orthopaedic Surgeons

All Rights Reserved. No part of the electronic media program may be reproduced, stored in a
retrieval system, or transmitted in any form or by any means, electronic, mechanical, photocopying,
recording, or otherwise without the prior written permission from the American Academy of
Orthopaedic Surgeons. Made in the USA.

Produced 2016 by the


American Academy of Orthopaedic Surgeons
9400 West Higgins Road
Rosemont, IL 60018-4976

Requests for permission to reproduce any part of the work should be mailed to:
Attention Examinations Department
American Academy of Orthopaedic Surgeons
9400 West Higgins Road
Rosemont, IL 60018-4976

 2016 American Academy of Orthopaedic Surgeons 2016 Orthopaedic In-Training Examination

© 2016 American Academy of Orthopaedic Surgeons 2016 Orthopaedic In-Training Examination


November 12, 2016

Dear Residents and Colleagues,

On behalf of the Central Evaluation Committee, I would like to thank you for participating in this
year’s examination.

Being the first such examination among all the medical specialties, this year marks the 53rd
administration of the Orthopaedic In-training Examination™.

Every year, the OITE™ is created by a group of orthopaedic surgeons who are deeply committed to
education through a rigorous peer-review process. While many topics in orthopaedic surgery lack
conclusive evidence, many of our practices are based on sound medical principles and a consensus of
experts. Sometimes, there may appear several correct answers to a test item, but each question has
been vetted through this peer-review process to select the one best preferred response.

With this and each subsequent administration, we strive to make the OITE™ better. Thank you for
being part of this year’s examination, and I hope that you find this to be an enriching educational
experience.

© 2016 American Academy of Orthopaedic Surgeons 2016 Orthopaedic In-Training Examination


AMA PRA CREDIT
U. S. Physicians: The American Academy of Orthopaedic Surgeons designates this enduring
material for a maximum of 20 AMA PRA Category 1 Credits™. Physicians should claim only
the credit commensurate with the extent of their participation in the activity.

International Physicians: AMA PRA credit may only be claimed by, and awarded to, physicians,
defined by the AMA as individuals who have completed an allopathic (MD), osteopathic (DO),
or an equivalent medical degree from another country.

Allied Health Professionals: The American Academy of Orthopaedic Surgeons is not accredited
to offer credit for nurses and other allied health professionals. To determine if activities offering
AMA PRA Category 1 Credits™ are acceptable for your licensing or certification needs, please
contact the relevant organization(s) directly.

EDUCATIONAL OBJECTIVES
As a result of completing the 2016 Orthopaedic In-Training Examination, I am able to

• interpret patient data for making clinical decision.


• determine the appropriate patient management procedures in orthopaedic surgery.
• integrate relevant evidence-based information regarding treatment procedures into my
patient practice.
• apply basic science principles in diagnosing and developing patient management plans for
musculoskeletal diseases.

INQUIRIES
Questions regarding completing this CME activity or other comments may be sent to
exams@aaos.org or write to: Attention: Examinations Department, American Academy of
Orthopaedic Surgeons, 9400 West Higgins Road, Rosemont, IL 60018.

Release Date: November 2016


Expiration Date: June 1, 2017
No CME credit will be awarded for this activity after June 1, 2017.

 2016 American Academy of Orthopaedic Surgeons 2016 Orthopaedic In-Training Examination

© 2016 American Academy of Orthopaedic Surgeons 2016 Orthopaedic In-Training Examination


Produced by the American Academy of Orthopaedic Surgeons
Central Evaluation Committee

M. Daniel Wongworawat, MD, Editor in Chief


Professor, Program Director, Orthopaedic Surgery Residency, Loma Linda University Medical Center,
Department of Orthopaedic Surgery, Loma Linda, California

Contributors
Joseph Benevenia, MD
Professor and Chair, Department of Orthopaedics, Rutgers-New Jersey Medical School, Newark, New
Jersey
Brandon D. Bushnell, MD, MBA
Chairman, Department of Orthopaedics and Sports Medicine, Harbin Clinic, LLC, Rome, Georgia;
Assistant Clinical Professor, Department of Orthopaedic Surgery, Medical College of Georgia, Augusta
University, Augusta, Georgia
Robert Hyun Cho, MD
University of California Los Angeles Clinical Assistant Professor of Orthopaedic Surgery, Chief of Staff,
Shriners Hospital for Children, Los Angeles, California
Michael P. Clare, MD
Director of Foot and Ankle Fellowship, Florida Orthopaedic Institute; Affiliate Associate Professor,
Department of Orthopaedic Surgery, University of South Florida, Tampa, Florida
David G. Dennison, MD
Assistant Professor of Orthopaedics, Mayo Clinical College of Medicine, Department of Orthopedic
Surgery, Rochester, Minnesota
Thomas C. Dowd, MD
Associate Residency Director, Orthopaedic Surgery Residency Program, San Antonio Military Medical
Center, Fort Sam Houston, Texas
Stephen T. Duncan, MD
Assistant Professor, Assistant Program Director, Department of Orthopaedic Surgery, University of
Kentucky School of Medicine, Lexington, Kentucky
Patrick B. Ebeling, MD
Assistant Professor, Department of Orthopedic Surgery, University of Minnesota Medical School,
Minneapolis, Minnesota; Orthopaedic Foot and Ankle Surgeon, Twin Cities Orthopedics, Burnsville,
Minnesota
Michael J. Elliott, MD
Assistant Clinical Professor, University of California, San Francisco School of Medicine, Department of
Orthopedics, Valley Children’s Hospital, Madera, California
Robert J. Esther, MD
Program Director, Department of Orthopaedics, University of North Carolina, Chapel Hill, North
Carolina
Albert O. Gee, MD
Assistant Professor, University of Washington, Department of Orthopaedics & Sports Medicine; Chief,
Sports Medicine, Seattle, Washington
Konrad I. Gruson, MD
Associate Professor, Department of Orthopaedic Surgery, Albert Einstein College of Medicine, Bronx,
New York
Steven J. Hospodar, MD
Orthopaedic Surgeon, Private Practice, Gloucester, Virginia
Kyle T. Judd, MD
Assistant Professor and Director of Orthopaedic Trauma Fellowship Program, University of Rochester,
Department of Orthopaedic Surgery, Rochester, New York
James P. Lawrence, MD
Associate Professor, Albany Medical College, Department of Surgery, Division of Orthopaedics, Albany,
New York

 2016 American Academy of Orthopaedic Surgeons 2016 Orthopaedic In-Training Examination

© 2016 American Academy of Orthopaedic Surgeons 2016 Orthopaedic In-Training Examination


Timothy A. Moore, MD
Associate Professor, Department of Orthopaedic Surgery, Case Western Reserve University School of
Medicine, MetroHealth Medical Center, Cleveland, Ohio
Varatharaj Mounasamy, MD
Associate Professor, Director of Orthopedic Trauma, Department of Orthopedic Surgery, Virginia
Commonwealth University Health System, Richmond, Virginia
Kevin M. Neal, MD
Pediatric Orthopedic Fellowship Director, Nemours Children’s Specialty Care; Chief of Pediatric
Orthopedic Surgery, Wolfson Children’s Hospital; Instructor in Orthopedics, Mayo Clinic School of
Medicine; Courtesy Assistant Professor in Pediatrics, University of Florida College of Medicine,
Jacksonville, Florida
Fred R. T. Nelson, MD
Nonoperative Orthopaedic Surgeon, Henry Ford Hospital; Clinical Associate Professor, Wayne State
Medical School, Detroit, Michigan
Matthew Oetgen, MD
Assistant Professor, Department of Orthopaedic Surgery and Sports Medicine, Children’s National
Medical Center, Washington, DC
Shervin V. Oskouei, MD
Assistant Professor, Department of Orthopaedic Surgery, Emory University School of Medicine, Atlanta,
Georgia
Norman Y. Otsuka, MD
Chief, Pediatric Orthopaedic Surgery, The Children's Hospital at Montefiore; Professor of Clinical
Orthopaedic Surgery and Clinical Pediatrics, Albert Einstein College of Medicine, Bronx, New York
Rajeev Pandarinath, MD
Assistant Professor, Orthopaedic Surgery, The George Washington University School of Medicine,
Washington, DC
Christopher W. Peer, MD, MS
Clinical Assistant Professor, Department of Orthopaedic Surgery, University of Missouri-Kansas City
School of Medicine, Kansas City, Kansas
Richard W. Pope, MD
Orthopaedic Surgeon, Private Practice, Augusta, Georgia
Sheeraz Qureshi, MD
Associate Professor, Orthopaedic Surgery, Minimally Invasive Spinal Surgery, Co-Director, Spinal
Surgery Fellowship, Mount Sinai Hospital Icahn School of Medicine, New York, New York
Timothy Rapp, MD
Associate Professor of Orthopaedic Surgery, Chief, Division of Orthopaedic Oncology New York
University Langone Medical Center, Department of Orthopaedic Surgery, New York, New York
Tom E. Reinsel, MD
Orthopaedic Spine Surgeon, Northland Bone and Joint, North Kansas City, Missouri
Christopher J. Roach, MD
LTC, MC, Chief, Ambulatory Surgery, Department of Orthopaedics, San Antonio Military Medical
Center; Asst. Professor, Orthopaedic Surgery, Uniformed Services University of the Health Sciences,
San Antonio, Texas
J. Milo Sewards, MD
Program Director, Orthopaedic Surgery Residency, Associate Professor of Orthopaedic Surgery and
Sports Medicine, Team Physician, Temple University School of Medicine, Philadelphia, Pennsylvania
Krishna R. Tripuraneni, MD
Attending Physician, New Mexico Orthopaedic Associates, Albuquerque, New Mexico
Rahul Vaidya, MD, CM, FRCSc
Professor of Orthopaedic Surgery Wayne State University; Interim Chair Detroit Medical Center/Wayne
State University Residency Training Program, Detroit, Michigan
Christina M. Ward, MD
Assistant Professor, Department of Orthopaedic Surgery, Fellowship Director, Healthpartners Hand
Fellowship, University of Minnesota, St Paul, Minnesota

 2016 American Academy of Orthopaedic Surgeons 2016 Orthopaedic In-Training Examination

© 2016 American Academy of Orthopaedic Surgeons 2016 Orthopaedic In-Training Examination


John S. Xenos, MD
Orthopaedic Surgeon specializing in Adult Reconstruction, President and Founder, Colorado
Orthopedics, Lone Tree, Colorado
Madhusudhan R. Yakkanti, MD
Assistant Clinical Professor of Orthopaedic Surgery, University of Louisville, Louisville, Kentucky
Lee M. Zuckerman, MD
Assistant Professor, Director of Adult and Pediatric Orthopaedic Oncology, Department of Orthopaedic
Surgery, Loma Linda University Medical Center, Loma Linda, California

ACKNOWLEDGMENTS

Medical Editing
Brenda Moss Feinberg, ELS, AAOS Examinations Consultant

Graphic Design and Digital Media Specialist


John Cisco, AAOS Examinations Consultant

American Academy of Orthopaedic Surgeons


Ellen Moore, Chief Education Officer

Department of Education, Examination Programs


Jonathan Sprague, MS, Manager, Examinations
Marcie L. Lampert, Senior Coordinator, Examinations
Christina Crumlish, Coordinator, Examinations
David Marshall, Coordinator and Technology Specialist, Examinations
Irene Bogdal, Administrative Assistant, Examinations

 2016 American Academy of Orthopaedic Surgeons 2016 Orthopaedic In-Training Examination

© 2016 American Academy of Orthopaedic Surgeons 2016 Orthopaedic In-Training Examination


Disclosure Information for the
2016 Orthopaedic In-Training Examination

Montri D Wongworawat, MD (Chair): Submitted on: 04/07/2016


AAOS: Board or committee member
Alumni Association School of Medicine of Loma Linda University: Board or committee member
Association of Bone and Joint Surgeons: Board or committee member
Clinical Orthopaedics and Related Research: Editorial or governing board
Health Services International: Board or committee member
Neufeld Society (Alumni Association): Board or committee member
San Bernardino County Medical Society: Board or committee member

Joseph Benevenia, MD (Member): Submitted on: 04/08/2016


AAOS, Musculoskeletal Transplant Foundation, Musculoskeletal Tumor Society: Board or committee member
AAOS/OKU Tumors 2, 3, Musculoskeletal Tumor Society: Editorial or governing board
Creosso LLC: Stock or stock Options
Implant Cast: Unpaid consultant
MereteNJOS: Unpaid consultant
Musculoskeletal Transplant Foundation: Paid presenter or speaker; Research support
Rutgers University/Creosso LLC: IP royalties

Brandon Dubose Bushnell, MBA, MD (Member): Submitted on: 05/02/2016


AAOS: Board or committee member
Arthroscopy Association of North America: Board or committee member
Georgia Orthopedic Society: Board or committee member

Robert Hyun Cho, MD (Member): Submitted on: 04/19/2016


DePuy Spine: Paid consultant
Medtronic Sofamor Danek: Paid consultant
Nuvasive: Paid consultant
OrthoPediatrics: Paid consultant
Orthopedics: Editorial or governing board

Michael Patrick Clare, MD (Member): Submitted on: 04/08/2016


AAOS: Board or committee member
BESPA: Paid consultant
LifeNet: Paid consultant
MergeNet: Stock or stock Options
Orthopaedic Trauma Association: Board or committee member
Pacira: Paid consultant
Synthes: Paid consultant; Paid presenter or speaker; Research support
The Orthopaedic Implant Company: Stock or stock Options

David G Dennison, MD: Submitted on: 06/01/2016


AAOS: Board or committee member
Mid America Ortho Assoc Finance Committee: Board or committee member

Thomas Charles Dowd, MD (Member): Submitted on: 05/11/2016


American Orthopaedic Association: Board or committee member
American Orthopaedic Foot and Ankle Society: Board or committee member
Zimmer: Research support

Stephen Thomas Duncan, MD (Member): Submitted on: 04/04/2016


American Association of Hip and Knee Surgeons: Board or committee member
Journal of Arthroplasty: Editorial or governing board
Kentucky Orthopaedic Society: Board or committee member
Mitek: Paid consultant
Morph: Unpaid consultant
Smith & Nephew: Paid consultant

 2016 American Academy of Orthopaedic Surgeons 2016 Orthopaedic In-Training Examination

© 2016 American Academy of Orthopaedic Surgeons 2016 Orthopaedic In-Training Examination


Patrick Brian Ebeling, MD (Member): Submitted on: 04/04/2016
AAOS: Board or committee member
American Orthopaedic Foot and Ankle Society: Board or committee member
Foot and Ankle International: Editorial or governing board
Journal of Bone and Joint Surgery - American: Editorial or governing board

Michael Johnathon Elliott, MD (Member): (This individual reported nothing to disclose); Submitted on:
08/25/2016

Robert J Esther, MD: Submitted on: 05/19/2016


Musculoskeletal Transplant Foundation: Other financial or material support

Albert Ooguen Gee, MD (Member): Submitted on: 05/31/2016


Medbridge Inc.: Publishing royalties, financial or material support

Konrad Izumi Gruson, MD (Member): Submitted on: 06/11/2016


AAOS: Board or committee member
Amgen Co: Stock or stock Options
Bristol-Myers Squibb: Stock or stock Options
DePuy, A Johnson & Johnson Company: Paid presenter or speaker
Eli Lilly: Stock or stock Options
GlaxoSmithKline: Stock or stock Options
Johnson & Johnson: Stock or stock Options
Medtronic: Stock or stock Options
Merck: Stock or stock Options
Pfizer: Stock or stock Options
Procter & Gamble: Stock or stock Options
Stryker: Stock or stock Options
Zimmer: Stock or stock Options

Steven John Hospodar, MD (Member): Submitted on: 12/01/2015


Pfizer: Stock or stock Options

Kyle T Judd, MD: (This individual reported nothing to disclose); Submitted on: 04/05/2016

James Patrick Lawrence, MBA, MD (Member): (This individual reported nothing to disclose); Submitted on:
05/21/2016

Timothy A Moore, MD: (This individual reported nothing to disclose); Submitted on: 06/21/2016

Varatharaj Mounasamy, MD (Member): Submitted on: 05/14/2016


AAOS: Board or committee member
European journal of orthopedic surgery and traumatology: Editorial or governing board

Kevin M Neal, MD: Submitted on: 04/05/2016


AAOS: Board or committee member
Medtronic: Unpaid consultant
Orthopediatrics: IP royalties
Pediatric Orthopaedic Society of North America: Board or committee member
Scoliosis Research Society: Board or committee member

Fred R T Nelson, MD: Submitted on: 03/16/2016


Elsevier for book royalty: Publishing royalties, financial or material support

Matthew Oetgen, MD (Member): Submitted on: 04/05/2016


AAOS: Board or committee member
Pediatric Orthopaedic Society of North America: Board or committee member
Scoliosis Research Society: Board or committee member

 2016 American Academy of Orthopaedic Surgeons 2016 Orthopaedic In-Training Examination

© 2016 American Academy of Orthopaedic Surgeons 2016 Orthopaedic In-Training Examination


Shervin V Oskouei, MD (Member): Submitted on: 08/24/2016
Georgia Orthopaedic Society: Board or committee member

Norman Yoshinobu Otsuka, MD (Member): Submitted on: 06/10/2016


AAOS: Board or committee member
American Academy of Pediatrics: Board or committee member
American College of Surgeons: Board or committee member
American Journal of Orthopedics: Editorial or governing board
Journal of Children's Orthopaedics: Editorial or governing board
Journal of Orthopaedic Surgical Advances: Editorial or governing board
Journal of Pediatric Orthopedics, Part B: Editorial or governing board
Pediatric Orthopaedic Society of North America: Board or committee member

Rajeev Pandarinath, MD (Member): Submitted on: 06/09/2016


AAOS: Board or committee member
Procter & Gamble, AstraZeneca: Stock or stock Options

Christopher William Peer, MD, MS: Submitted on: 09/06/2016


AAOS: Board or committee member
Arthrex, Inc: Other financial or material support
Breg: Other financial or material support
DePuy, A Johnson & Johnson Company: Other financial or material support
Smith & Nephew: Other financial or material support
XR Medical: Other financial or material support

Richard W Pope, MD (Member): Submitted on: 03/04/2016


AAOS Evaluations Committee (Hand/Wrist) starting in MAR11: Board or committee member

Sheeraz Qureshi, MD (Member): Submitted on: 05/02/2016


AAOS: Board or committee member
Cervical Spine Research Society: Board or committee member
Clinical Orthopaedics and Related Research: Editorial or governing board
Contemporary Spine Surgery: Editorial or governing board
Global Spine Journal: Editorial or governing board
Globus Medical: Paid presenter or speaker
Medtronic: Paid consultant
Medtronic Sofamor Danek: Paid presenter or speaker
Musculoskeletal Transplant Foundation: Board or committee member
NASS: Board or committee member
Orthofix, Inc.: Paid consultant
Spine (reviewer): Editorial or governing board
Spine Journal (reviewer): Editorial or governing board
Stryker: Paid consultant; Paid presenter or speaker
Zimmer: IP royalties; Paid consultant

Timothy Rapp, MD (Member): Submitted on: 06/17/2016


AAOS: Board or committee member
American Board of Orthopaedic Surgery, Inc.: Board or committee member
Clinical Orthopaedics and Related Research Bulletin of the NYU Hospital for Joint Diseases American Journal of
Orthopaedics: Editorial or governing board
Research or Institutional Support-- Department of Orthopaedic Surgery Hospital for Joint Diseases at NYU
Langone Medical Center: AO Spine Arthrex Arthritis Foundation– NY Chapter Arthritis National Research
Foundation Asterland Biomet Depuy Encore Exactech/ DJO Ferring Pharmaceuticals Geisinger Integra Johnson &
Johnson KCI Medtronic NIHOMEGAOREF Orthopaedic Trauma Association Osteosynthesis and Trauma Care
Foundation Paradigm Spine Progenics Sbi Smith and Nephew Stryker Surgix Synthes: Research support

 2016 American Academy of Orthopaedic Surgeons 2016 Orthopaedic In-Training Examination

© 2016 American Academy of Orthopaedic Surgeons 2016 Orthopaedic In-Training Examination


Tom E Reinsel, MD (Member): Submitted on: 04/05/2016
AAOS (Central Evaluation Committee, Spine): Board or committee member
Missouri State Orthopedic Ass'n (secretary): Board or committee member
Spine: Editorial or governing board
The Spine Journal (reviewer): Editorial or governing board

Christopher James Roach, MD (Member): Submitted on: 04/04/2016


AAOS: Board or committee member

Joseph Milo Sewards, MD (Member): Submitted on: 04/17/2016


Aesculap/B.Braun: Research support

Krishna Raj Tripuraneni, MD (Member): Submitted on: 05/14/2016


Arthroplasty Today: Editorial or governing board
DJO Surgical: Research support
Journal of Arthroplasty: Editorial or governing board
Orthopaedic Implant Company: Stock or stock Options

Rahul Vaidya, MD: Submitted on: 06/01/2016


European Spine Journal: Editorial or governing board
Smith & Nephew: IP royalties
Stryker: Paid consultant; Paid presenter or speaker; Unpaid consultant
Synthes: IP royalties; Other financial or material support; Paid presenter or speaker; Research support

Christina M Ward, MD (LFP Member): (This individual reported nothing to disclose); Submitted on: 06/05/2016

John S Xenos, MD (Member): Submitted on: 08/31/2016


Regenesis Biomedical: Stock or stock Options
Smith & Nephew: Paid consultant

Madhusudhan Reddy Yakkanti, MD (Member): Submitted on: 05/31/2016


Synthes- Received honorarium for participation as a table instructor in a shoulder course sponsored by Synthes:
Other financial or material support

Lee Michael Zuckerman, MD (Member): Submitted on: 04/27/2016


AAOS: Board or committee member

Jon Sprague (Staff Liaison): (This individual reported nothing to disclose); Submitted on: 04/04/2016

 2016 American Academy of Orthopaedic Surgeons 2016 Orthopaedic In-Training Examination

© 2016 American Academy of Orthopaedic Surgeons 2016 Orthopaedic In-Training Examination


Disclaimer
The goals of the in-training examination are: 1) to provide a way for exam participants to assess personal
knowledge and compare scores with peers; and 2) to permit faculty to evaluate the strengths and weaknesses of
training programs as indicated by exam results. Residency training programs should not use examination results as a
basis for dismissal of a resident from the program.

The material presented in this in-training examination has been made available by the American Academy of
Orthopaedic Surgeons for educational purposes only. This material is not intended to present the only, or
necessarily the best, methods or procedures for the medical situations discussed, but rather is intended to represent
an approach, view, statement, or opinion of the author(s) or producer(s), which may be helpful to others who face
similar situations.

Some drugs or medical devices demonstrated in Academy courses or described in Academy print or electronic
publications have not been cleared by the Food and Drug Administration (FDA) or have been cleared for specific
uses only. The FDA has stated that it is the responsibility of the physician to determine the FDA clearance status of
each drug or device he or she wishes to use in clinical practice.

Furthermore, any statements about commercial products are solely the opinion(s) of the author(s) and do not
represent an Academy endorsement or evaluation of these products. These statements may not be used in advertising
or for any commercial purpose.

The Orthopaedic In-Training Examination® (OITE) is the intellectual property of the American Academy of
Orthopaedic Surgeons. A non-exclusive license to use OITE and its content does not confer on the user any
ownership rights, including but not limited to copyright.

 2016 American Academy of Orthopaedic Surgeons 2016 Orthopaedic In-Training Examination

© 2016 American Academy of Orthopaedic Surgeons 2016 Orthopaedic In-Training Examination


2016 Orthopaedic In-Training Examination

TABLE OF CONTENTS

EXAMINATION QUESTIONS Page


SECTION 1: Basic Science and Orthopaedic Diseases ..................................................... 17
SECTION 2: Foot and Ankle ............................................................................................. 39
SECTION 3: Hand.............................................................................................................. 64
SECTION 4: Hip and Knee Reconstruction....................................................................... 83
SECTION 5: Oncology....................................................................................................... 106
SECTION 6: Pediatrics....................................................................................................... 133
SECTION 7: Shoulder and Elbow...................................................................................... 163
SECTION 8: Spine ............................................................................................................. 186
SECTION 9: Sports Medicine ............................................................................................ 206
SECTION 10: Trauma.......................................................................................................... 225

APPENDIX
Content Domain Item Numbers............................................................................................. 269

 2016 American Academy of Orthopaedic Surgeons 2016 Orthopaedic In-Training Examination


© 2016 American Academy of Orthopaedic Surgeons 2016 Orthopaedic In-Training Examination
SECTION 1: Basic Science and Orthopaedic Disease • 17

SECTION 1: Basic Science and Orthopaedic Disease

Question 9
When skeletal muscle myofibrils are activated through neural pathways, which muscle fiber type is
recruited first?

1. Type IIA
2. Type IIB
3. Fast glycolytic
4. Oxidative glycolytic
5. Slow oxidative

PREFERRED RESPONSE: 5

RECOMMENDED READINGS
Wright A, Gharaibeh B, Huard J. Form and function of skeletal muscle. In: O’Keefe RJ, Jacobs JJ, Chu
CR, Einhorn TA. Orthopaedic Basic Science: Foundations of Clinical Practice. 4th ed. Rosemont, IL:
American Academy of Orthopaedic Surgeons; 2013:229-238.

Staron RS. Human skeletal muscle fiber types: delineation, development, and distribution. Can J Appl
Physiol. 1997 Aug;22(4):307-27. Review. PubMed PMID: 9263616.

Question 22
Which biochemical that is associated with maintenance of the articular cartilage phenotype is most
important during growth and development?

1. Decorin
2. Matrillin
3. Parathyroid-related protein (PTHrP)
4. 1,24-dihydroxychlolecalciferol
5. Cartilage oliogomeric protein (COMP)

PREFERRED RESPONSE: 3

© 2016 American Academy of Orthopaedic Surgeons 2016 Orthopaedic In-Training Examination


18 • American Academy of Orthopaedic Surgeons

RECOMMENDED READINGS
Pacifici M. The development and growth of the skeleton. In: O’Keefe RJ, Jacobs JJ, Chu CR, Einhorn
TA, eds. Orthopaedic Basic Science Foundations of Clinical Practice. 4th ed. Rosemont, IL: American
Academy of Orthopaedic Surgeons; 2013:135-148.

Chubinskaya S, Malfait A, Wimmer MA. Form and function of articular cartilage. In: O’Keefe RJ,
Jacobs JJ, Chu CR, Einhorn TA, eds. Orthopaedic Basic Science Foundations of Clinical Practice. 4th ed.
Rosemont, IL: American Academy of Orthopaedic Surgeons; 2013:183-197.

Zhang W, Chen J, Zhang S, Ouyang HW. Inhibitory function of parathyroid hormone-related protein
on chondrocyte hypertrophy: the implication for articular cartilage repair. Arthritis Res Ther. 2012 Aug
31;14(4):221. doi: 10.1186/ar4025. Review. PMID: 22971952.

Question 24
Which anterior cruciate ligament (ACL) graft type has the most tensile stiffness?

1. Bone-patellar tendon-bone
2. Quadriceps tendon
3. Quadrupled hamstring
4. Tibialis anterior
5. Achilles tendon

PREFERRED RESPONSE: 1

RECOMMENDED READINGS
Sherman SL, Chalmers PN, Yanke AB, Bush-Joseph CA, Verma NN, Cole BJ, Bach BR Jr. Graft
tensioning during knee ligament reconstruction: principles and practice. J Am Acad Orthop Surg. 2012
Oct;20(10):633-45. doi: 10.5435/JAAOS-20-10-633. Review. PubMed PMID: 23027693.

Noyes FR, Butler DL, Grood ES, Zernicke RF, Hefzy MS. Biomechanical analysis of human ligament
grafts used in knee-ligament repairs and reconstructions. J Bone Joint Surg Am. 1984 Mar;66(3):344-52.
PubMed PMID: 6699049.

© 2016 American Academy of Orthopaedic Surgeons 2016 Orthopaedic In-Training Examination


SECTION 1: Basic Science and Orthopaedic Disease • 19

Question 36
During the course of intervertebral disk degeneration and enzymatic degradation, the molecular release
that plays a predominant role in disk degeneration is

1. biglycan.
2. collagen II propeptide.
3. fibronectin fragmentation.
4. the G3 terminal domain of the core protein of aggrecan.
5. the thrombospondin portion of cartilage oligomeric protein.

PREFERRED RESPONSE: 3

RECOMMENDED READINGS
Lotz JC, Haughton V, Boden SD, An HS, Kang JD, Masuda K, Freemont A, Berven S, Sengupta DK,
Tanenbaum L, Maurer P, Ranganathan A, Alavi A, Marinelli NL. New treatments and imaging strategies
in degenerative disease of the intervertebral disks. Radiology. 2012 Jul;264(1):6-19. doi: 10.1148/
radiol.12110339. Review. PMID: 22723559.

Feng H, Danfelter M, Strömqvist B, Heinegård D. Extracellular matrix in disc degeneration. J Bone Joint
Surg Am. 2006 Apr;88 Suppl 2:25-9. Review. PubMed PMID: 16595439.

Question 52
Which immune cell type is involved in innate immunity?

1. Killer T cells
2. Helper T cells
3. Natural killer (NK) cells
4. Plasma cells
5. Memory B cells

PREFERRED RESPONSE: 3

RECOMMENDED READINGS
Campbell KS, Hasegawa J. Natural killer cell biology: an update and future directions. J Allergy Clin
Immunol. 2013 Sep;132(3):536-44. doi: 10.1016/j.jaci.2013.07.006. Epub 2013 Jul 30. Review. PubMed
PMID: 23906377.

Rosier RN, Reynolds PR, O’Keefe RJ. Molecular and cell biology in orthopaedics. In: Buckwalter
JA, Einhorn TA, Simon SR, eds. Orthopaedic Basic Science: Biology and Biomechanics of the
Musculoskeletal System, 2nd ed. Rosemont, IL: American Academy of Orthopaedic Surgeons; 2000:19-76.

© 2016 American Academy of Orthopaedic Surgeons 2016 Orthopaedic In-Training Examination


20 • American Academy of Orthopaedic Surgeons

Question 61
What is the maximum length that can be bridged by a nerve conduit when repairing a nerve gap?

1. 1 cm
2. 2 cm
3. 3 cm
4. 4 cm
5. 5 cm

PREFERRED RESPONSE: 3

RECOMMENDED READINGS
Deal DN, Griffin JW, Hogan MV. Nerve conduits for nerve repair or reconstruction. J Am Acad Orthop
Surg. 2012 Feb;20(2):63-8. doi: 10.5435/JAAOS-20-02-063. Review. PubMed PMID: 22302443.

Griffin JW, Hogan MV, Chhabra AB, Deal DN. Peripheral nerve repair and reconstruction. J Bone Joint
Surg Am. 2013 Dec 4;95(23):2144-51. doi: 10.2106/JBJS.L.00704. Review. PubMed PMID: 24306702.

Question 67
A 58-year-old woman has had gradually worsening weakness in her dominant hand for several weeks and
now has a wrist drop. What is the site of nerve entrapment?

1. Arcade of Frohse
2. Ligament of Struthers
3. Cubital tunnel
4. Guyon’s canal
5. Edge of the lacertus fibrosus

PREFERRED RESPONSE: 1

RECOMMENDED READINGS
Urch EY, Model Z, Wolfe SW, Lee SK. Anatomical Study of the Surgical Approaches to the Radial
Tunnel. J Hand Surg Am. 2015 Jul;40(7):1416-20. doi: 10.1016/j.jhsa.2015.03.009. Epub 2015 Apr 18.
PubMed PMID: 25899182.

Clavert P, Lutz JC, Adam P, Wolfram-Gabel R, Liverneaux P, Kahn JL. Frohse's arcade is not the exclusive
compression site of the radial nerve in its tunnel. Orthop Traumatol Surg Res. 2009 Apr;95(2):114-8. doi:
10.1016/j.otsr.2008.11.001. Epub 2009 Mar 17. PubMed PMID: 19297265.

© 2016 American Academy of Orthopaedic Surgeons 2016 Orthopaedic In-Training Examination


SECTION 1: Basic Science and Orthopaedic Disease • 21

Figure 72

Question 72
Figure 72 is the MR image of a torn structure consisting predominantly of which collagen type?

1. I
2. II
3. IV
4. VI
5. X

PREFERRED RESPONSE: 1

RECOMMENDED READINGS:
Fox AJ, Bedi A, Rodeo SA. The basic science of human knee menisci: structure, composition, and
function. Sports Health. 2012 Jul;4(4):340-51. PubMed PMID: 23016106.

Rodeo SA, Kawamura S. Form and function of the meniscus. In: Einhorn TA, O’Keefe RJ, Buchwalter
JA, eds. Orthopaedic Basic Science: Foundations of Clinical Practice. 3rd ed. Rosemont, IL: American
Academy of Orthopaedic Surgeons; 2007:175-191.

© 2016 American Academy of Orthopaedic Surgeons 2016 Orthopaedic In-Training Examination


22 • American Academy of Orthopaedic Surgeons

Question 84
Three months after surgery, an orthopaedic surgeon is studying the postsurgical range of motion of 2
patient cohorts that underwent total knee surgery. This is a single-surgeon study, and the patients are
prospectively randomized to a cruciate-retaining knee implant and a posterior-stabilized total knee
implant. The null hypothesis is that patients who received cruciate-retaining total knee implants would not
have better range of motion at 3 months. At the end of the study, it is discovered that the patients in the
posterior-stabilized cohort received outpatient physical therapy for a longer period of time. This variable is
an example of

1. selection bias.
2. confounding.
3. follow-up bias.
4. conflict of interest.
5. determination bias.

PREFERRED RESPONSE: 2

RECOMMENDED READINGS
Schmidt AH, Leopold SS, Stovitz, SD. Detection of bias in clinical research. In: O’Keefe RJ, Jacobs
JJ, Chu CR, Einhorn TA, eds. Orthopaedic Basic Science: Foundations of Clinical Practice. 4th ed.
Rosemont, IL: American Academy of Orthopaedic Surgeons; 2013:483-494.

Hayashi AC. A case for evidence-based orthopaedic clinical research. J Orthop Res. 2011 Jun;29(6):I-III.
doi: 10.1002/jor.21412. PubMed PMID: 21520252.

Question 87
A 12-year-old boy who is relatively short in stature has sustained several low-energy fractures despite
having dense-appearing bones. Radiographs reveal that his distal phalanges are short. The front and
back of his head are prominent. The boy’s baby teeth arrived late, and his permanent teeth were slow to
develop. There are no cervical cords or other neurologic defects. Although his bones are denser, there is
now widening of the metaphysis. The likely cause of this disorder is a genetic defect of

1. activin A receptor, type 1 (ACVR-1).


2. integrins.
3. cathepsin K (CatK).
4. carbonic anhydrase.
5. chloride channel gene (CICN7).

PREFERRED RESPONSE: 3

© 2016 American Academy of Orthopaedic Surgeons 2016 Orthopaedic In-Training Examination


SECTION 1: Basic Science and Orthopaedic Disease • 23

RECOMMENDED READINGS
Kennedy OD, Majeska RJ, Schaffler MB. Form and function of bone. In: O’Keefe RJ, Jacobs JJ, Chu
CR, Einhorn TA, eds. Orthopaedic Basic Science Foundations of Clinical Practice. 4th ed. Rosemont, IL:
American Academy of Orthopaedic Surgeons; 2013:149-181.

Yates CJ, Bartlett MJ, Ebeling PR. An atypical subtrochanteric femoral fracture from pycnodysostosis: a
lesson from nature. J Bone Miner Res. 2011 Jun;26(6):1377-9. doi: 10.1002/jbmr.308. PubMed PMID:
21611976.

Question 100
Following peripheral nerve injury, distal recovery typically occurs first in

1. motor function.
2. proprioception.
3. sympathetic activity.
4. temperature.
5. pain perception.

PREFERRED RESPONSE: 3

RECOMMENDED READINGS
Jackson WM, Diao E. Peripheral nerves: form and function. In: O’Keefe RJ, Jacobs JJ, Chu CR, Einhorn
TA, eds. Orthopaedic Basic Science: Foundations of Clinical Practice. 4th ed. Rosemont, IL: American
Academy of Orthopaedic Surgeons; 2013: 239-252.

Dodds SD. Peripheral nervous system. In: Boyer, MI. ed. AAOS Comprehensive Orthopaedic Review 2.
Rosemont, IL: American Academy of Orthopaedic Surgeons; 2014: 113-126.

© 2016 American Academy of Orthopaedic Surgeons 2016 Orthopaedic In-Training Examination


24 • American Academy of Orthopaedic Surgeons

Question 106
A 56-year-old woman sustained a low-energy fracture and has a bone mineral density T-score of -2.7.
Other than mild hypertension, she has no systemic disorder. She is taking amlodipine for hypertension and
clonazepam as an antiepileptic. She should be advised that she

1. does not need testing for 25 hydroxycholecalciferol levels and that clonazepam use could be
related to her osteoporosis.
2. should be tested for 25 hydroxycholecalciferol; the test finding may be normal, but
clonazepam use may be related to her osteoporosis.
3. should be tested for 25 hydroxycholecalciferol and that her medications are unrelated.
4. should be tested for 1,25 hydroxycholecalciferol and that her medications are unrelated.
5. should be tested for 1,25 dihydroxycholecalciferol and that both medications are known to
interfere with vitamin D metabolism.

PREFERRED RESPONSE: 2

RECOMMENDED READINGS
Hassler N, Roschger A, Gamsjaeger S, Kramer I, Lueger S, van Lierop A, Roschger P, Klaushofer K,
Paschalis EP, Kneissel M, Papapoulos S. Sclerostin deficiency is linked to altered bone composition. J
Bone Miner Res. 2014 Oct;29(10):2144-51. doi: 10.1002/jbmr.2259. PubMed PMID: 24753092.

El-Hajj Fuleihan G, Dib L, Yamout B, Sawaya R, Mikati MA. Predictors of bone density in ambulatory
patients on antiepileptic drugs. Bone. 2008 Jul;43(1):149-55. doi: 10.1016/j.bone.2008.03.002. Epub 2008
Mar 15. PubMed PMID: 18467202.

Question 115
A 6-year-old child was born with a fixed flexion deformity of both little finger proximal interphalangeal
joints, and several other fingers have similar flexion deformities. The child has a slight waddle, so gait
radiographs of the hip were obtained and coxa vara was noted. Laboratory study findings including
complete blood count, erythrocyte sedimentation rate, and rheumatoid factor are negative. A referral to a
pediatrician reveals that the child has a cardiac rub. What is the most likely diagnosis?

1. A defect in the EXT1 gene


2. Juvenile rheumatoid arthritis (JRA)
3. Juvenile-onset lupus erythematosus
4. Camptodactyly
5. Camptodactyly-arthropathy-coxa vara-pericarditis syndrome (CACP)

PREFERRED RESPONSE: 5

© 2016 American Academy of Orthopaedic Surgeons 2016 Orthopaedic In-Training Examination


SECTION 1: Basic Science and Orthopaedic Disease • 25

RECOMMENDED READINGS
Chubinskaya S, Malfait A, Wimmer MA. Form and function of articular cartilage. In: O’Keefe RJ,
Jacobs JJ, Chu CR, Einhorn TA, eds. Orthopaedic Basic Science Foundations of Clinical Practice. 4th ed.
Rosemont, IL: American Academy of Orthopaedic Surgeons; 2013:183-197.

Bulutlar G, Yazici H, Ozdoğan H, Schreuder I. A familial syndrome of pericarditis, arthritis,


camptodactyly, and coxa vara. Arthritis Rheum. 1986 Mar;29(3):436-8. PubMed PMID: 3964320.

Offiah AC, Woo P, Prieur AM, Hasson N, Hall CM. Camptodactyly-arthropathy-coxa vara-pericarditis
syndrome versus juvenile idiopathic arthropathy. AJR Am J Roentgenol. 2005 Aug;185(2):522-9. PubMed
PMID: 16037531.

Question 118
Both achondroplasia and Jansen-type metaphyseal dysplasia are examples of inherited disorders
characterized by a signaling disorder of

1. ligand mutation inherited as autosomal recessive.


2. ligand mutation inherited as autosomal dominant.
3. activated receptor mutation inherited as autosomal recessive.
4. activated receptor mutation inherited as autosomal dominant.
5. activated receptor mutation inherited as sex-linked recessive.

PREFERRED RESPONSE: 4

RECOMMENDED READINGS
Pacifici M. The development and growth of the skeleton. In: O’Keefe RJ, Jacobs JJ, Chu CR, Einhorn
TA, eds. Orthopaedic Basic Science Foundations of Clinical Practice. 4th ed. Rosemont, IL: American
Academy of Orthopaedic Surgeons; 2013:135-148.

Martínez-Frías ML, de Frutos CA, Bermejo E, Nieto MA; ECEMC Working Group. Review of
the recently defined molecular mechanisms underlying thanatophoric dysplasia and their potential
therapeutic implications for achondroplasia. Am J Med Genet A. 2010 Jan;152A(1):245-55. doi: 10.1002/
ajmg.a.33188. Review. PubMed PMID: 20034074.

© 2016 American Academy of Orthopaedic Surgeons 2016 Orthopaedic In-Training Examination


26 • American Academy of Orthopaedic Surgeons

Question 126
A 76-year-old man with long-standing degenerative joint disease of his right hip has developed a
contracture of that hip. He walks with a characteristic crouched posture. To maintain his center of mass
while walking in a straight line, he compensates by

1. keeping his contralateral hip extended.


2. swinging his arms in a widened stance.
3. dorsiflexing his ipsilateral foot.
4. flexing his ipsilateral knee.
5. increasing lumbar kyphosis.

PREFERRED RESPONSE: 4

RECOMMENDED READINGS
Lim MR, Huang RC, Wu A, Girardi FP, Cammisa FP Jr. Evaluation of the elderly patient with an
abnormal gait. J Am Acad Orthop Surg. 2007 Feb;15(2):107-17. Review. PubMed PMID: 17277257.

Baldwin K, Keena MA. Normal and pathologic gait. In: Boyer MI. ed. AAOS Comprehensive Orthopaedic
Review 2. Rosemont, IL: American Academy of Orthopaedic Surgeons; 2014:179-188.

Question 134
A 22-year-old woman has a relatively low-energy distal radius fracture. She takes no medications and is in
good health, but her vitamin D levels are very low. Vitamin D metabolite and sclerostin levels are assessed
before and after 3 months of vitamin D replacement therapy to determine if therapy will improve her long-
term bone health. There is expected elevation in

1. 25-hydroxycholecalciferol and sclerostin levels but not 1,25-dihydroxycholecalciferol level.


2. 1,25-dihydroxycholecalciferol level with a lower level of sclerostin but no effect on
25-hydroxycholecalciferol level.
3. 1,25-dihydroxycholecalciferol and 25-hydroxycholecalciferol levels with a lower level
of sclerostin.
4. 1,25-dihydroxycholecalciferol and 25-hydroxycholecalciferol levels with a higher level
of sclerostin.
5. 1,25-dihydroxycholecalciferol and 25-hydroxycholecalciferol levels but no effect on
sclerostin level.

PREFERRED RESPONSE: 3

© 2016 American Academy of Orthopaedic Surgeons 2016 Orthopaedic In-Training Examination


SECTION 1: Basic Science and Orthopaedic Disease • 27

RECOMMENDED READINGS
Virdi AS, Irish J, Sena K, Liu M, Ke HZ, McNulty MA, Sumner DR. Sclerostin antibody treatment
improves implant fixation in a model of severe osteoporosis. J Bone Joint Surg Am. 2015 Jan
21;97(2):133-40. doi: 10.2106/JBJS.N.00654. PubMed PMID: 25609440.

Cidem M, Karacan I, Arat NB, Zengi O, Ozkaya M, Guzel SP, Ozkan C, Beytemur O. Serum sclerostin is
decreased following vitamin D treatment in young vitamin D-deficient female adults. Rheumatol Int. 2015
Oct;35(10):1739-42. doi: 10.1007/s00296-015-3294-1. Epub 2015 May 26. PubMed PMID: 26007153.

Question 142
What is the predominant function of aggrecan in articular cartilage?

1. Controls collagen fibril size


2. Controls collagen spatial orientation
3. Maintains osmotic water balance
4. Transmits mechanical forces to chondrocytes
5. Exerts swelling pressure against the restraint of collagen

PREFERRED RESPONSE: 5

RECOMMENDED READINGS
Chubinskaya S, Malfait A, Wimmer MA. Form and function of articular cartilage. In: O’Keefe RJ,
Jacobs JJ, Chu CR, Einhorn TA, eds. Orthopaedic Basic Science Foundations of Clinical Practice. 4th ed.
Rosemont, IL: American Academy of Orthopaedic Surgeons; 2013:183-197.

Yanagishita M. Function of proteoglycans in the extracellular matrix. Acta Pathol Jpn. 1993
Jun;43(6):283-93. Review. PubMed PMID: 8346704.

© 2016 American Academy of Orthopaedic Surgeons 2016 Orthopaedic In-Training Examination


28 • American Academy of Orthopaedic Surgeons

Figure 148

Question 148
The characteristics of the region labeled A on the stress-strain curve of a ligament as shown in Figure 148
predominantly are attributed to which molecule?

1. Aggrecan
2. Fibronectin
3. Decorin
4. Elastin
5. Lubricin

PREFERRED RESPONSE: 4

RECOMMENDED READINGS:
Bhandari M. Evidence-based orthopaedics. In: Einhorn TA, O’Keefe RJ, Buchwalter JA, eds. Orthopaedic
Basic Science: Foundations of Clinical Practice. 3rd ed. Rosemont, IL: American Academy of Orthopaedic
Surgeons; 2007:87-105.

Yanagishita M. Function of proteoglycans in the extracellular matrix. Acta Pathol Jpn. 1993
Jun;43(6):283-93. Review. PubMed PMID: 8346704.

© 2016 American Academy of Orthopaedic Surgeons 2016 Orthopaedic In-Training Examination


SECTION 1: Basic Science and Orthopaedic Disease • 29

Figure 153

Question 153
Which structure is identified by the arrow in Figure 153?

1. Semimembranosus
2. Semitendinosis
3. Gracillis
4. Popliteus
5. Sartorius

PREFERRED RESPONSE: 1

RECOMMENDED READINGS:
Stoller DW. Magnetic Resonance Imaging in Orthopaedics and Sports Medicine. 3rd Edition. Lippincott
2006.

LaPrade RF, Engebretsen AH, Ly TV, Johansen S, Wentorf FA, Engebretsen L. The anatomy of the medial
part of the knee. J Bone Joint Surg Am. 2007 Sep;89(9):2000-10. PubMed PMID: 17768198.

© 2016 American Academy of Orthopaedic Surgeons 2016 Orthopaedic In-Training Examination


30 • American Academy of Orthopaedic Surgeons

Question 155
To reconstruct a segmental tibia defect for a patient who has sustained a gunshot wound, an orthopaedic
surgeon chooses to use a freeze-dried allograft. Which potential disadvantage is associated with this type
of graft?

1. Reduced torsional and bending strength compared to frozen allograft


2. Reduced compressive and tensile strength compared to frozen allograft
3. Lower shelf life compared to fresh and frozen allograft
4. Increased immunogenicity compared to fresh allograft
5. Loss of osteoconductive properties compared to frozen allograft

PREFERRED RESPONSE: 1

RECOMMENDED READINGS
Bauer TW, Muschler GF. Bone graft materials. An overview of the basic science. Clin Orthop Relat Res.
2000 Feb;(371):10-27. Review. PubMed PMID: 10693546.

Bae HW, Bhamb N, Kanim LEA, Field JS. Bone grafts, bone morphogenetic proteins, and bone
substitutes. In: Boyer, MI. ed. AAOS Comprehensive Orthopaedic Review 2. Rosemont, IL: American
Academy of Orthopaedic Surgeons; 2014:73-80.

Question 166
Ceramics and inorganic compounds often are used in orthopaedic clinical practice to fill bone defects.
Their biomaterial properties are best described as

1. very rapid resorption and remodeling of synthetic hydroxyapatite.


2. very slow resorption of calcium sulfate.
3. providing excellent osteoinductive properties.
4. having bioceramics that typically exhibit weak compressive strength.
5. having bioceramics with larger pore sizes that have a better osteoconductive effect.

PREFERRED RESPONSE: 5

RECOMMENDED READINGS
Bauer TW, Muschler GF. Bone graft materials. An overview of the basic science. Clin Orthop Relat Res.
2000 Feb;(371):10-27. Review. PubMed PMID: 10693546.

Bae HW, Bhamb N, Kanim LEA, Field JS. Bone grafts, bone morphogenetic proteins, and bone
substitutes. In: Boyer, MI. ed. AAOS Comprehensive Orthopaedic Review 2. Rosemont, IL: American
Academy of Orthopaedic Surgeons; 2014:73-80.

Dinopoulos H, Dimitriou R, Giannoudis PV. Bone graft substitutes: What are the options? Surgeon. 2012
Aug;10(4):230-9. doi: 10.1016/j.surge.2012.04.001. Epub 2012 Jun 6. Review. Retraction in: Surgeon.
2013 Apr;11(2):115. PubMed PMID: 22682580.

© 2016 American Academy of Orthopaedic Surgeons 2016 Orthopaedic In-Training Examination
SECTION 1: Basic Science and Orthopaedic Disease • 31

Question 170
Aspirin affects bleeding time by which mechanism?

1. Activates antithrombin III


2. Blocks adenosine diphosphate (ADP) receptor on platelet membranes
3. Blocks thromboxane A2
4. Inhibits factor Xa
5. Inhibits vitamin K epoxide reductase

PREFERRED RESPONSE: 3

RECOMMENDED READINGS
Triplett DA. Coagulation and bleeding disorders: review and update. Clin Chem. 2000 Aug;46(8 Pt
2):1260-9. Review. PubMed PMID: 10926920.

Sahebally SM, Healy D, Walsh SR. Aspirin in the primary prophylaxis of venous thromboembolism in
surgical patients. Surgeon. 2015 Dec;13(6):348-58. doi: 10.1016/j.surge.2015.05.001. Epub 2015 Jun 10.
Review. PubMed PMID: 26071929.

Question 176
Which abnormality causes excessive knee extension during the swing phase of gait?

1. Hip flexion weakness


2. Ankle dorsiflexion weakness
3. Hamstring contracture
4. Quadriceps spasticity
5. Quadriceps weakness

PREFERRED RESPONSE: 5

RECOMMENDED READINGS
Perry J. Gait Analysis: Normal and Pathological Function. Thorofare, NJ. SLACK Inc. 1992.

Simon RS, Alaranta H, et al. Kinesiology. In: Buckwalter JA, Einhorn TA, Simon SR, eds. Orthopaedic
Basic Science: Biology and Biomechanics of the Musculoskeletal System, 2nd ed. Rosemont, IL: American
Academy of Orthopaedic Surgeons; 2000:731-827.

© 2016 American Academy of Orthopaedic Surgeons 2016 Orthopaedic In-Training Examination


32 • American Academy of Orthopaedic Surgeons

Question 186
What is the dominant contributing factor in intervertebral disk degeneration?

1. Genetics
2. Nutrition
3. Inflammation
4. Vascular supply
5. Trabecular remodeling

PREFERRED RESPONSE: 1

RECOMMENDED READINGS
Battié MC, Videman T, Kaprio J, Gibbons LE, Gill K, Manninen H, Saarela J, Peltonen L. The Twin Spine
Study: contributions to a changing view of disc degeneration. Spine J. 2009 Jan-Feb;9(1):47-59. doi:
10.1016/j.spinee.2008.11.011. PubMed PMID:19111259.

Kalb S, Martirosyan NL, Kalani MY, Broc GG, Theodore N. Genetics of the degenerated intervertebral
disc. World Neurosurg. 2012 Mar-Apr;77(3-4):491-501. doi: 10.1016/j.wneu.2011.07.014. Epub 2011 Nov
7. Review. PubMed PMID: 22120330.

© 2016 American Academy of Orthopaedic Surgeons 2016 Orthopaedic In-Training Examination


SECTION 1: Basic Science and Orthopaedic Disease • 33

Question 192
The mechanism of action for fondaparinux, a synthetic pentasaccharide, is via

1. powerful and irreversible thrombin binding.


2. indirect inhibition of the clotting cascade at the level of factor X.
3. antagonism of platelet aggregation.
4. orally administered direct inhibition of activated factor X.
5. direct inhibition of activated factor II (thrombin).

PREFERRED RESPONSE: 2

RECOMMENDED READINGS
Shoda N, Yasunaga H, Horiguchi H, Fushimi K, Matsuda S, Kadono Y, Tanaka S. Prophylactic effect of
fondaparinux and enoxaparin for preventing pulmonary embolism after total hip or knee arthroplasty: A
retrospective observational study using the Japanese Diagnosis Procedure Combination database. Mod
Rheumatol. 2015 Jul;25(4):625-9. doi: 10.3109/14397595.2014.997424. Epub 2015 Feb 11. PubMed
PMID: 25529031.

Drescher FS, Sirovich BE, Lee A, Morrison DH, Chiang WH, Larson RJ. Aspirin versus anticoagulation
for prevention of venous thromboembolism major lower extremity orthopedic surgery: a systematic
review and meta-analysis. J Hosp Med. 2014 Sep;9(9):579-85. doi: 10.1002/jhm.2224. Epub 2014 Jul 17.
Review. PubMed PMID: 25045166.

© 2016 American Academy of Orthopaedic Surgeons 2016 Orthopaedic In-Training Examination


34 • American Academy of Orthopaedic Surgeons

Question 198
The Small, Integrin-Binding LIgand, N-linked Glycoprotein (SIBLING) family of noncollagenous
proteins includes osteopontin, bone sialoprotein, dentin matrix protein-1, and matrix extracellular
phosphoglycoprotein. Which biochemical characteristic is responsible for much of the function?

1. Glycosylation of collagen
2. An arg-gly-asp (RGD) sequence
3. Control of procollagen fibril winding
4. A vitamin K–dependent gamma-carboxylation site
5. A glycogen side chain responsible for control of fibril size

PREFERRED RESPONSE: 2

RECOMMENDED READINGS
Kennedy OD, Majeska RJ, Schaffler MB. Form and function of bone. In: O’Keefe RJ, Jacobs JJ, Chu
CR, Einhorn TA, eds. Orthopaedic Basic Science Foundations of Clinical Practice. 4th ed. Rosemont, IL:
American Academy of Orthopaedic Surgeons; 2013:149-181.

Huang B, Sun Y, Maciejewska I, Qin D, Peng T, McIntyre B, Wygant J, Butler WT, Qin C. Distribution
of SIBLING proteins in the organic and inorganic phases of rat dentin and bone. Eur J Oral Sci. 2008
Apr;116(2):104-12. doi: 10.1111/j.1600-0722.2008.00522.x. PubMed PMID: 18353003.

© 2016 American Academy of Orthopaedic Surgeons 2016 Orthopaedic In-Training Examination


SECTION 1: Basic Science and Orthopaedic Disease • 35

Figure 203

Question 203
Which molecular loss is most associated with Figure 203?

1. Versican
2. Aggrecan
3. Hyaluronic acid
4. Type I collagen
5. Type II collagen

PREFERRED RESPONSE: 2

RECOMMENDED READINGS
Moss IL, An HS. Form and function of the intervertebral disk. In: O’Keefe RJ, Jacobs JJ, Chu CR,
Einhorn TA, eds. Orthopaedic Basic Science Foundations of Clinical Practice. 4th ed. Rosemont, IL:
American Academy of Orthopaedic Surgeons; 2013:253-260.

Lotz JC, Haughton V, Boden SD, An HS, Kang JD, Masuda K, Freemont A, Berven S, Sengupta DK,
Tanenbaum L, Maurer P, Ranganathan A, Alavi A, Marinelli NL. New treatments and imaging strategies
in degenerative disease of the intervertebral disks. Radiology. 2012 Jul;264(1):6-19. doi: 10.1148/
radiol.12110339. Review. PubMed PMID: 22723559.

© 2016 American Academy of Orthopaedic Surgeons 2016 Orthopaedic In-Training Examination


36 • American Academy of Orthopaedic Surgeons

Question 207
Which structure(s) is/are found at the bone-tendon junction and is/are responsible for nociception?

1. Free nerve endings


2. Golgi organs
3. Golgi apparatus
4. Pacini corpuscles
5. Ruffini endings

PREFERRED RESPONSE: 1

RECOMMENDED READINGS
Jackson WM, Diao E. Peripheral nerves: form and function. In: O’Keefe RJ, Jacobs JJ, Chu CR, Einhorn
TA. Orthopaedic Basic Science: Foundations of Clinical Practice. 4th ed. Rosemont, IL: American
Academy of Orthopaedic Surgeons; 2013:239-252.

Slobodin G, Rozenbaum M, Boulman N, Rosner I. Varied presentations of enthesopathy. Semin Arthritis


Rheum. 2007 Oct;37(2):119-26. Epub 2007 Mar 13. Review. PubMed PMID: 17350676.

Question 227
One of the nonmechanical causes of osteoarthritis is the DNA methylation of the metalloprotease promoter
sequence. This is an example of

1. anticipation.
2. epigenetic change.
3. posttranslational change.
4. spontaneous mutation.
5. single-nucleotide polymorphism (SNP).

PREFERRED RESPONSE: 2

RECOMMENDED READINGS
Lee FY, Drissi MH, Zuscik MJ, Chen D, Nizami S, Goto H. Molecular and cell biology in orthopaedics.
In: O’Keefe RJ, Jacobs JJ, Chu CR, Einhorn TA, eds. Orthopaedic Basic Science Foundations of Clinical
Practice. 4th ed. Rosemont, IL: American Academy of Orthopaedic Surgeons; 2013:3-42.

Kang D, Cho Y, Kim JH. Epigenetic Regulation of Chondrocyte Catabolism and Anabolism in
Osteoarthritis. Kim H, Mol Cells. 2015 Aug 31;38(8):677-84. doi: 0.14348/molcells.2015.0200. Epub
2015 Aug 5. Review. PubMed PMID: 26242192.

© 2016 American Academy of Orthopaedic Surgeons 2016 Orthopaedic In-Training Examination


SECTION 1: Basic Science and Orthopaedic Disease • 37

Question 237
The first bone tissue that forms in developing long bones is intramembranous. This is initiated by a lateral-
diffusion Indian hedgehog. At this point, the hypertrophic chondrocytes elaborate

1. Sox-9.
2. osteocalcin.
3. fibroblast growth factor 3 (FGF3).
4. vascular endothelial growth factor (VEGF).
5. peroxisome proliferator-activated receptors gamma 2 (PPARy2).

PREFERRED RESPONSE: 4

RECOMMENDED READINGS
Pacifici M. The development and growth of the skeleton. In: O’Keefe RJ, Jacobs JJ, Chu CR, Einhorn
TA, eds. Orthopaedic Basic Science Foundations of Clinical Practice. 4th ed. Rosemont, IL: American
Academy of Orthopaedic Surgeons; 2013:135-148.

Ferguson C, Alpern E, Miclau T, Helms JA. Does adult fracture repair recapitulate embryonic skeletal
formation? Mech Dev. 1999 Sep;87(1-2):57-66. PubMed PMID: 10495271.

Question 255
Which nuclear transcription factor is important in tendon and ligament formation?

1. Osterix
2. CCAAT/enhancer-binding protein (C/EBP)
3. Twist 1
4. Scleraxis
5. Sclerostin

PREFERRED RESPONSE: 4

RECOMMENDED READINGS
Lee FY, Drissi MH, Zuscik MJ, Chen D, Nizami S, Goto H. Molecular and cell biology in orthopaedics.
In: O’Keefe RJ, Jacobs JJ, Chu CR, Einhorn TA, eds. Orthopaedic Basic Science Foundations of Clinical
Practice. 4th ed. Rosemont, IL: American Academy of Orthopaedic Surgeons; 2013:3-42.

Liu H, Zhu S, Zhang C, Lu P, Hu J, Yin Z, Ma Y, Chen X, OuYang H. Crucial transcription factors


in tendon development and differentiation: their potential for tendon regeneration. Cell Tissue Res.
2014 May;356(2):287-98. doi: 10.1007/s00441-014-1834-8. Epub 2014 Apr 8. Review. PubMed
PMID:24705622.

© 2016 American Academy of Orthopaedic Surgeons 2016 Orthopaedic In-Training Examination


38 • American Academy of Orthopaedic Surgeons

Question 274
Which biomaterial has the highest rate of bacterial adherence?

1. Pure titanium
2. Stainless steel
3. Tantalum
4. Titanium alloy
5. Silver

PREFERRED RESPONSE: 4

RECOMMENDED READINGS
Schildhauer TA, Robie B, Muhr G, Köller M. Bacterial adherence to tantalum versus commonly used
orthopedic metallic implant materials. J Orthop Trauma. 2006 Jul;20(7):476-84. PubMed PMID:
16891939.

Campoccia D, Montanaro L, Arciola CR. A review of the clinical implications of anti-infective


biomaterials and infection-resistant surfaces. Biomaterials. 2013 Nov;34(33):8018-29. doi: 10.1016/j.
biomaterials.2013.07.048. Epub 2013 Aug 8. Review. PubMed PMID: 23932292.

Brennan SA, Ní Fhoghlú C, Devitt BM, O'Mahony FJ, Brabazon D, Walsh A. Silver
nanoparticles and their orthopaedic applications. Bone Joint J. 2015 May;97-B(5):582-9. doi:
10.1302/0301-620X.97B5.33336. Review. Erratum in: Bone Joint J. 2015 Jul;97-B(7):1012. PubMed
PMID: 25922449.

© 2016 American Academy of Orthopaedic Surgeons 2016 Orthopaedic In-Training Examination


SECTION 2: Foot and Ankle • 39

SECTION 2: Foot and Ankle

Figure 1a Figure 1b

Figure 1c Figure 1d

Question 1
Figures 1a through 1d are the weight-bearing radiographs of a 76-year-old man who has a rigid, painful
foot deformity. Examination reveals he has a severe planovalgus foot position. In addition to addressing
the soft-tissue contracture, which surgical plan offers appropriate correction while minimizing potential
soft-tissue complications?

1. Isolated talonavicular arthrodesis through a medial approach


2. Isolated subtalar arthrodesis through a lateral approach
3. Subtalar and talonavicular arthrodesis through a medial approach
4. Triple arthrodesis through a medial approach
5. Triple arthrodesis through a 2-incision approach

PREFERRED RESPONSE: 3

© 2016 American Academy of Orthopaedic Surgeons 2016 Orthopaedic In-Training Examination


40 • American Academy of Orthopaedic Surgeons

RECOMMENDED READINGS
Saville P, Longman CF, Srinivasan SC, Kothari P. Medial approach for hindfoot arthrodesis with a valgus
deformity. Foot Ankle Int. 2011 Aug;32(8):818-21. PubMed PMID: 22049869.

Anand P, Nunley JA, DeOrio JK. Single-incision medial approach for double arthrodesis of
hindfoot in posterior tibialis tendon dysfunction. Foot Ankle Int. 2013 Mar;34(3):338-44. doi:
10.1177/1071100712468564. PubMed PMID: 23520290.

Sammarco VJ, Magur EG, Sammarco GJ, Bagwe MR. Arthrodesis of the subtalar and talonavicular joints
for correction of symptomatic hindfoot malalignment. Foot Ankle Int. 2006 Sep;27(9):661-6. PubMed
PMID: 17038274.

© 2016 American Academy of Orthopaedic Surgeons 2016 Orthopaedic In-Training Examination


SECTION 2: Foot and Ankle • 41

Figure 16a Figure 16b

Figure 16c Figure 16d

Question 16
Figures 16a through 16c are the sagittal and axial proton density images and Figure 16d is the MRI sagittal
short tau inversion recovery MR image of a 60-year-old man who has had ankle pain for several months.
There is tenderness to palpation approximately 3 to 4 cm above the top of the calcaneus. Which form of
exercise most consistently and effectively addresses this condition?

1. Eccentric
2. Isometric
3. Plyometric
4. Isotonic
5. Concentric

PREFERRED RESPONSE: 1

© 2016 American Academy of Orthopaedic Surgeons 2016 Orthopaedic In-Training Examination


42 • American Academy of Orthopaedic Surgeons

RECOMMENDED READINGS
Rowe V, Hemmings S, Barton C, Malliaras P, Maffulli N, Morrissey D. Conservative management of
midportion Achilles tendinopathy: a mixed methods study, integrating systematic review and clinical
reasoning. Sports Med. 2012 Nov 1;42(11):941-67. doi: 10.2165/11635410-000000000-00000. Review.
PubMed PMID: 23006143.

Mafi N, Lorentzon R, Alfredson H. Superior short-term results with eccentric calf muscle training
compared to concentric training in a randomized prospective multicenter study on patients with chronic
Achilles tendinosis. Knee Surg Sports Traumatol Arthrosc. 2001;9(1):42-7. PubMed PMID: 11269583.

Figure 31a Figure 31b Figure 31c

Question 31
Figures 31a through 31c are the weight-bearing radiographs of an active 32-year-old man who has lateral
foot pain that is most notable with high-impact activity. There is no history of trauma or giving-way
episodes. He has full heel cord flexibility and is able to perform a single-leg heel rise. Coleman block test
findings are positive. His physical therapist told him he has “fallen arches” and recommended custom
inserts. Which orthosis is optimal in this scenario?

1. Semirigid insert with a medial post and medial arch support


2. Soft, full-length closed cell polyethylene foam inlay
3. Lateral heel and sole wedge with a first metatarsal head recess
4. Full-length steel shank with a rocker-bottom sole
5. Carbon shank with a Morton’s extension

PREFERRED RESPONSE: 3

© 2016 American Academy of Orthopaedic Surgeons 2016 Orthopaedic In-Training Examination


SECTION 2: Foot and Ankle • 43

RECOMMENDED READINGS
Chilvers M, Manoli A 2nd. The subtle cavus foot and association with ankle instability and lateral foot
overload. Foot Ankle Clin. 2008 Jun;13(2):315-24, vii. doi: 10.1016/j.fcl.2008.01.003. Review. PubMed
PMID: 18457776.

Manoli A 2nd, Graham B. The subtle cavus foot, "the underpronator". Foot Ankle Int. 2005
Mar;26(3):256-63. Review. PubMed PMID: 15766431.

Figure 46a Figure 46b Figure 46c

Question 46
Figures 46a through 46c are the radiograph and MR images of a 55-year-old woman who had immediate
pain after hearing a pop while twisting her ankle. What is the most appropriate surgical treatment?

1. Lateral ligament reconstruction


2. Lateralizing calcaneus osteotomy
3. Superior peroneal retinaculum (SPR) repair
4. Microfracture of the talus
5. Deltoid repair

PREFERRED RESPONSE: 3

RECOMMENDED READINGS
Philbin TM, Landis GS, Smith B. Peroneal tendon injuries. J Am Acad Orthop Surg. 2009 May;17(5):306-
17. Review. PubMed PMID: 19411642.

Ogawa BK, Thordarson DB. Current concepts review: peroneal tendon subluxation and dislocation. Foot
Ankle Int. 2007 Sep;28(9):1034-40. Review. PubMed PMID: 17880883.

© 2016 American Academy of Orthopaedic Surgeons 2016 Orthopaedic In-Training Examination


44 • American Academy of Orthopaedic Surgeons

Figure 54a Figure 54b Figure 54c

Question 54
Figures 54a through 54c are the current standing radiographs of a 48-year-old woman who sustained an
open talar neck fracture-dislocation 2 years ago. She experiences consistent pain in her ankle with weight-
bearing activities and has failed nonsurgical treatment. What is the most prudent treatment method?

1. Distraction arthroplasty
2. Ankle arthrodesis
3. Osteochondral allograft
4. Arthroscopic debridement and implant removal
5. Total ankle arthroplasty

PREFERRED RESPONSE: 2

RECOMMENDED READINGS
Deorio JK, Easley ME. Total ankle arthroplasty. Instr Course Lect. 2008;57:383-413. Review. PubMed
PMID: 18399599.

Bugbee WD, Khanna G, Cavallo M, McCauley JC, Görtz S, Brage ME. Bipolar fresh osteochondral
allografting of the tibiotalar joint. J Bone Joint Surg Am. 2013 Mar 6;95(5):426-32. doi: 10.2106/
JBJS.L.00165. PubMed PMID: 23467865.

© 2016 American Academy of Orthopaedic Surgeons 2016 Orthopaedic In-Training Examination


SECTION 2: Foot and Ankle • 45

Figure 69a Figure 69b Figure 69c

Question 69
Figure 69a is a lateral weight-bearing radiograph of a 55-year-old woman with diabetes who has an
18-month history of unilateral plantar heel pain. She has received 5 injections during the last year to
address plantar fasciitis. She has deep, central, nonradiating plantar heel pain that is worse when barefoot
and resolves when she walks on her toes. She has tenderness to palpation at the central aspect of her
heel. Figures 69b and 69c are proton density (PD) and short tau inversion recovery (STIR) sagittal MRI
findings. What is the most appropriate next step?

1. Nonweight-bearing activity for 3 to 4 weeks followed by a gradual return to activity


2. Partial plantar fasciectomy
3. Decompression of the first branch of the lateral plantar nerve
4. Physical therapy for Achilles tendon stretching and modalities
5. External heel padding

PREFERRED RESPONSE: 5

RECOMMENDED READINGS
Lareau CR, Sawyer GA, Wang JH, DiGiovanni CW. Plantar and medial heel pain: diagnosis and
management. J Am Acad Orthop Surg. 2014 Jun;22(6):372-80. doi: 10.5435/JAAOS-22-06-372. Review.
PubMed PMID: 24860133.

Johnson JE, Klein SE, Putnam RM. Corticosteroid injections in the treatment of foot & ankle disorders:
an AOFAS survey. Foot Ankle Int. 2011 Apr;32(4):394-9. doi: 10.3113/FAI.2011.0394. PubMed PMID:
21733442.

© 2016 American Academy of Orthopaedic Surgeons 2016 Orthopaedic In-Training Examination


46 • American Academy of Orthopaedic Surgeons

Question 80
Which structure is most at risk when the anterolateral ankle arthroscopy portal is established?

1. Deep peroneal nerve


2. Dorsalis pedis artery
3. Extensor digitorum longus tendon
4. Distal tibial articular surface
5. Superficial peroneal nerve

PREFERRED RESPONSE: 5

RECOMMENDED READINGS
Young BH, Flanigan RM, DiGiovanni BF. Complications of ankle arthroscopy utilizing a contemporary
noninvasive distraction technique. J Bone Joint Surg Am. 2011 May 18;93(10):963-8. doi: 10.2106/
JBJS.I.00977. PubMed PMID: 21593373.

Barber FA, Click J, Britt BT. Complications of ankle arthroscopy. Foot Ankle. 1990 Apr;10(5):263-6.
Review. PubMed PMID: 2187776.

Kim DH. Arthroscopy of the Foot and Ankle. In: Chou LB, ed. Orthopaedic Knowledge Update: Foot and
Ankle 5. Rosemont, IL: American Academy of Orthopaedic Surgeons; 2014:401-409.

© 2016 American Academy of Orthopaedic Surgeons 2016 Orthopaedic In-Training Examination


SECTION 2: Foot and Ankle • 47

Figure 93a Figure 93b Figure 93c

Question 93
Figures 93a through 93c are the weight-bearing radiographs of a 58-year-old man who has hindfoot pain.
He has a remote history of a calcaneal fracture treated with closed reduction and casting. He exhibits
minimal hindfoot motion and lacks 10 degrees of ankle dorsiflexion compared to the contralateral side.
What is the most appropriate treatment method?

1. In situ subtalar arthrodesis


2. Lateral wall exostectomy
3. Hindfoot arthrodesis
4. Romash osteotomy with subtalar joint salvage
5. Subtalar bone block arthrodesis

PREFERRED RESPONSE: 5

RECOMMENDED READINGS
Carr JB, Hansen ST, Benirschke SK. Subtalar distraction bone block fusion for late complications of os
calcis fractures. Foot Ankle. 1988 Oct;9(2):81-6. PubMed PMID: 3066724.

Clare MP, Lee WE 3rd, Sanders RW. Intermediate to long-term results of a treatment protocol for
calcaneal fracture malunions. J Bone Joint Surg Am. 2005 May;87(5):963-73. PubMed PMID: 15866957.

© 2016 American Academy of Orthopaedic Surgeons 2016 Orthopaedic In-Training Examination


48 • American Academy of Orthopaedic Surgeons

Figure 105a Figure 105b Figure 105c

Question 105
After satisfactorily obtaining and confirming reduction of this fracture (Figures 105a through 105c), what
is the next step in the evaluation of this fracture pattern?

1. Stress radiographs to assess ligaments


2. Gravity external rotation stress view
3. Knee radiographs
4. CT scan through the ankle
5. Foot radiographs

PREFERRED RESPONSE: 4

RECOMMENDED READINGS
Hinds RM, Garner MR, Lazaro LE, Warner SJ, Loftus ML, Birnbaum JF, Burket JC, Lorich DG. Ankle
fracture spur sign is pathognomonic for a variant ankle fracture. Foot Ankle Int. 2015 Feb;36(2):159-64.
doi: 10.1177/1071100714553470. Epub 2014 Oct 2. PubMed PMID: 25278340.

Haraguchi N, Haruyama H, Toga H, Kato F. Pathoanatomy of posterior malleolar fractures of the ankle. J
Bone Joint Surg Am. 2006 May;88(5):1085-92. Erratum in: J Bone Joint Surg Am. 2006 Aug;88(8):1835.
PubMed PMID: 16651584.

© 2016 American Academy of Orthopaedic Surgeons 2016 Orthopaedic In-Training Examination


SECTION 2: Foot and Ankle • 49

Figure 117a Figure 117b Figure 117c

Video117d (scan to view) Video 117e (scan to view)

Question 117
Figures 117a through 117c are the radiographs of a 32-year-old recreational soccer player who has
chronic posterolateral ankle pain. She has stiffness during activity and increased pain when pushing off,
but has not had episodes of giving way. An examination reveals her ankle and hindfoot range of motion
are normal, but she has pain with maximum ankle plantar flexion. There is no tenderness at the distal
lateral calf. Neurovascular examination findings are normal. Anti-inflammatory drugs, physical therapy,
and activity modification have been unsuccessful. An injection into her subtalar joint provided minimal
improvement. MRI sequence scans are shown in Videos 117d and 117e. What is the most appropriate next
step?

1. Peroneal debridement and repair


2. Arthroscopy of the subtalar joint
3. Excision of an anomalous soleus muscle
4. Excision of a symptomatic os trigonum
5. Decompression of the sural nerve at the lateral ankle

PREFERRED RESPONSE: 4

RECOMMENDED READINGS
Nault ML, Kocher MS, Micheli LJ. Os Trigonum Syndrome. J Am Acad Orthop Surg. 2014
Sep;22(9):545-553. Review. PubMed PMID: 25157036.

Schon LC, Baxter DE. Neuropathies of the foot and ankle in athletes. Clin Sports Med. 1990
Apr;9(2):489-509. Review. PubMed PMID: 2183956.

© 2016 American Academy of Orthopaedic Surgeons 2016 Orthopaedic In-Training Examination
50 • American Academy of Orthopaedic Surgeons

Question 130
A 22-year-old woman has an equinus contracture. She was involved in a motor vehicle collision 1 year
ago in which she sustained a humeral shaft fracture, iliac wing fracture, and closed-head injury, which
necessitated prolonged rehabilitation. Ankle dorsiflexion is to 10 degrees shy of neutral with her knee
in extension and in flexion, without full tension on the Achilles, and despite extensive physical therapy.
Treatment should consist of

1. fascial turndown/flexor hallucis longus (FHL) transfer.


2. the Strayer procedure.
3. the Baumann procedure.
4. Hoke triple-cut Achilles lengthening.
5. posterior capsular release/open Achilles lengthening.

PREFERRED RESPONSE: 5

RECOMMENDED READINGS
Chen L, Greisberg J. Achilles lengthening procedures. Foot Ankle Clin. 2009 Dec;14(4):627-37. doi:
10.1016/j.fcl.2009.08.002. Review. PubMed PMID: 19857837.

Barske HL, DiGiovanni BF, Douglass M, Nawoczenski DA. Current concepts review: isolated
gastrocnemius contracture and gastrocnemius recession. Foot Ankle Int. 2012 Oct;33(10):915-21. doi:
DOI: 10.3113/FAI.2012.0915. Review. PubMed PMID: 23050719.

© 2016 American Academy of Orthopaedic Surgeons 2016 Orthopaedic In-Training Examination


SECTION 2: Foot and Ankle • 51

Figure 147a Figure 147b

Question 147
Figures 147a and 147b are the weight-bearing radiographs of a 45-year-old laborer with a history of hallux
rigidus who has persistent foot pain despite undergoing surgery 1 year ago. He has pain with weight
bearing that is refractory to shoe modifications. He has stiffness through the first metatarsophalangeal
(MTP) joint, a positive axial grind test result, and tenderness to palpation of the dorsal midfoot. In addition
to stabilization of the second metatarsal, treatment should include

1. dorsal cheilectomy.
2. distraction bone block arthrodesis.
3. Moberg-type phalanx osteotomy.
4. capsular interposition arthroplasty.
5. first MTP hemiarthroplasty.

PREFERRED RESPONSE: 2

RECOMMENDED READINGS
Malhotra K, Nunn T, Qamar F, Rao V, Shanker J. Interposition bone block arthrodesis for revision
hallux metatarsophalangeal joint surgery: a case series. Foot Ankle Int. 2015 May;36(5):556-64. doi:
10.1177/1071100714563536. Epub 2014 Dec 10. PubMed PMID: 25504225.

Myerson MS, Schon LC, McGuigan FX, Oznur A. Result of arthrodesis of the hallux metatarsophalangeal
joint using bone graft for restoration of length. Foot Ankle Int. 2000 Apr;21(4):297-306. PubMed PMID:
10808969.

© 2016 American Academy of Orthopaedic Surgeons 2016 Orthopaedic In-Training Examination


52 • American Academy of Orthopaedic Surgeons

Figure 158a* Figure 158b*

Figure 158c*
*Used with permission from Anderson RB, Hunt KJ, McCormick II. Management of common sports-related injuries about the foot and ankle.
J Am Acad Orthop Surg. 2010 Sep;18(9):546-56. Review.

Question 158
A collegiate basketball player sustained the hyperextension injury shown in Figures 158a through 158c.
What is the preferred treatment?

1. First metatarsophalangeal (MTP) joint arthrodesis


2. Direct plantar plate repair
3. Taping of the first MTP joint and physical therapy
4. Sesamoidectomy
5. Plantar flexion casting of the first ray

PREFERRED RESPONSE: 2

RECOMMENDED READINGS
Anderson R: Turf toe injuries of the hallux metatarsophalangeal joint. Techniques in Foot & Ankle
Surgery 2002;1:102-111.

Anderson RB, Hunt KJ, McCormick JJ. Management of common sports-related injuries about the foot and
ankle. J Am Acad Orthop Surg. 2010 Sep;18(9):546-56. Review. PubMed PMID: 20810936.

© 2016 American Academy of Orthopaedic Surgeons 2016 Orthopaedic In-Training Examination


SECTION 2: Foot and Ankle • 53

Figure 179a Figure 179b

Question 179
Figures 179a and 179b are the anteroposterior and lateral radiographs of a 43-year-old man who is seen 1
year after undergoing hallux valgus correction on his right foot. His medial great toe pain resolved after
surgery, but he reports a persistent feeling that he is walking on a marble under his central forefoot. His
neurovascular examination findings are normal. There is a mild callous under his second metatarsal head.
Tenderness to palpation is isolated to the plantar aspect of the second metatarsophalangeal (MTP) joint.
There is 50% subluxation of the joint with a drawer test. What is the most appropriate next step?

1. Corticosteroid injection in the second web space


2. Foot orthotics with metatarsal pads
3. Nonweight-bearing activity for 3 weeks or until walking is pain free
4. Shaving of the callus and maintenance with a pumice stone
5. Immobilization in a stiff-soled shoe

PREFERRED RESPONSE: 2

RECOMMENDED READINGS
Mizel MS, Yodlowski ML. Disorders of the Lesser Metatarsophalangeal Joints. J Am Acad Orthop Surg.
1995 May;3(3):166-173. PubMed PMID: 10790665.

Doty JF, Coughlin MJ. Metatarsophalangeal joint instability of the lesser toes and plantar plate deficiency.
J Am Acad Orthop Surg. 2014 Apr;22(4):235-45. doi: 10.5435/JAAOS-22-04-235. Review. PubMed
PMID: 24668353.

© 2016 American Academy of Orthopaedic Surgeons 2016 Orthopaedic In-Training Examination


54 • American Academy of Orthopaedic Surgeons

Figure 193a Figure 193b

Figure 193c Figure 193d

Question 193
Figures 193a and 193b are the weight-bearing foot radiographs of a 48-year-old man who has had
progressive, atraumatic posterolateral ankle pain for several years. An examination reveals tenderness
behind his distal fibula, and a Coleman block test result is negative. His subtalar and first tarsometatarsal
joints have normal motion. Figures 193c and 193d are his MRI ankle findings. He has attempted bracing,
activity modification, nonsteroidal anti-inflammatory drugs, and physical therapy without success. What is
the best next step?

1. Debridement and repair of the peroneus brevis tendon and a lateralizing calcaneal osteotomy
2. Debridement and repair of the peroneus longus tendon and a lateralizing calcaneal osteotomy
3. Debridement and repair of the peroneus longus tendon and a dorsiflexion osteotomy of the
first metatarsal
4. Achilles tendon lengthening and triple arthrodesis
5. Lateral column lengthening, flexor digitorum longus transfer, and Achilles tendon lengthening

PREFERRED RESPONSE: 2

© 2016 American Academy of Orthopaedic Surgeons 2016 Orthopaedic In-Training Examination


SECTION 2: Foot and Ankle • 55

RECOMMENDED READINGS
Deben SE, Pomeroy GC. Subtle cavus foot: diagnosis and management. J Am Acad Orthop Surg. 2014
Aug;22(8):512-20. doi: 10.5435/JAAOS-22-08-512. Review. PubMed PMID: 25063749.

Grasset W, Mercier N, Chaussard C, Carpentier E, Aldridge S, Saragaglia D. The surgical treatment of


peroneal tendinopathy (excluding subluxations): a series of 17 patients. J Foot Ankle Surg. 2012 Jan-
Feb;51(1):13-9. doi: 10.1053/j.jfas.2011.10.010. Epub 2011 Nov 23. PubMed PMID: 22112304.

Figure 200

Question 200
Figure 200 is a weight-bearing radiograph of a 70-year-old woman with diabetes mellitus who has a
neuropathic plantar ulcer in the region of her first metatarsal head. An examination reveals peripheral
granulation tissue, neutral dorsiflexion, and exposed tendon but no exposed bone. Which intervention will
allow for ulcer healing, decrease risk for recurrence, and maintain function?

1. Topical application of a honey-impregnated dressing


2. Total-contact casting
3. Tendo-Achilles lengthening with total-contact casting
4. Metatarsal head resection
5. Transmetatarsal amputation

PREFERRED RESPONSE: 3

© 2016 American Academy of Orthopaedic Surgeons 2016 Orthopaedic In-Training Examination


56 • American Academy of Orthopaedic Surgeons

RECOMMENDED READINGS
Mueller MJ, Sinacore DR, Hastings MK, Strube MJ, Johnson JE. Effect of Achilles tendon lengthening on
neuropathic plantar ulcers. A randomized clinical trial. J Bone Joint Surg Am. 2003 Aug;85-A(8):1436-45.
PubMed PMID: 12925622.

Armstrong DG, Stacpoole-Shea S, Nguyen H, Harkless LB. Lengthening of the Achilles tendon in diabetic
patients who are at high risk for ulceration of the foot. J Bone Joint Surg Am. 1999 Apr;81(4):535-8.
Erratum in: J Bone Joint Surg Am. 2000 Oct;82-A(10):1510. PubMed PMID: 10225799.

Figure 211a Figure 211b Figure 211c

Question 211
Figures 211a through 211c are the standing radiographs of a 63-year-old man who has chronic ankle pain
and drainage 1 year after sustaining an open ankle fracture-dislocation. He denies recent fevers, chills, or
constitutional signs. There is clear fluid drainage from an anteromedial wound at the ankle joint line. In
addition to implant removal, definitive treatment should consist of

1. hindfoot arthrodesis.
2. ankle arthrodesis.
3. arthroscopic irrigation and debridement.
4. wound debridement and antibiotic beads.
5. saucerization and staged arthrodesis.

PREFERRED RESPONSE: 5

© 2016 American Academy of Orthopaedic Surgeons 2016 Orthopaedic In-Training Examination


SECTION 2: Foot and Ankle • 57

RECOMMENDED READINGS
Anakwenze OA, Milby AH, Gans I, Stern JJ, Levin LS, Wapner KL. Foot and ankle infections: diagnosis
and management. J Am Acad Orthop Surg. 2012 Nov;20(11):684-93. doi: 10.5435/JAAOS-20-11-684.
Review. PubMed PMID: 23118134.

Baumhauer JF, Lu AP, DiGiovanni BF. Arthodesis of the infected ankle and subtalar joint. Foot Ankle
Clin. 2002 Mar;7(1):175-90. Review. PubMed PMID: 12380388.

© 2016 American Academy of Orthopaedic Surgeons 2016 Orthopaedic In-Training Examination


58 • American Academy of Orthopaedic Surgeons

Figure 216a Figure 216b

Figure 216c

Question 216
Figures 216a through 216c are the anteroposterior and lateral weight-bearing radiographs of a 19-year-old
woman with chronic bilateral hallux valgus deformities. She reports pain over the medial eminence that
limits her shoe wear choices and recreational activities. Examination reveals tenderness over the medial
prominence. Her range of motion is normal. Wide toe box shoes, pads, and oral anti-inflammatory drugs
have not provided relief, and a decision is made to proceed with surgical treatment. The hallux valgus
angle is 34 degrees, and the 1-2 intermetatarsal angle (IMA) is 16 degrees. In addition to some form of
proximal correction, which distal procedure is necessary to achieve adequate correction?

1. Reduction of IMA with a mini-tightrope bunionectomy procedure


2. Medially based closing-wedge osteotomy of the first metatarsal
3. Akin osteotomy at the base of the proximal phalanx
4. Complete lateral soft-tissue release with sesamoid excision
5. Distal Chevron osteotomy of the first metatarsal

PREFERRED RESPONSE: 2

© 2016 American Academy of Orthopaedic Surgeons 2016 Orthopaedic In-Training Examination


SECTION 2: Foot and Ankle • 59

RECOMMENDED READINGS
Smith BW, Coughlin MJ. Treatment of hallux valgus with increased distal metatarsal articular angle: use
of double and triple osteotomies. Foot Ankle Clin. 2009 Sep;14(3):369-82. doi: 10.1016/j.fcl.2009.03.005.
Review. PubMed PMID: 19712880.

Siekmann W, Watson TS, Roggelin M. Correction of moderate to severe hallux valgus with isometric first
metatarsal double osteotomy. Foot Ankle Int. 2014 Nov;35(11):1122-30. doi: 10.1177/1071100714544520.
Epub 2014 Jul 23. PubMed PMID: 25056384.

Question 222
A 41-year-old woman sustained an inversion ankle sprain 6 months ago. She has continued to experience
pain with standing and walking and giving-way sensations on uneven ground. She exhibits tenderness to
palpation along the anterior ankle joint line, normal ankle and hindfoot motion, and stable anterior drawer
and calcaneal inversion test results. What is the best next step?

1. Cortisone injection
2. Initiate nonsteroidal anti-inflammatory drugs (NSAIDs)
3. Proprioception and peroneal strengthening
4. Lace-up ankle bracing
5. Custom arch supports

PREFERRED RESPONSE: 3

RECOMMENDED READINGS
van Ochten JM, van Middelkoop M, Meuffels D, Bierma-Zeinstra SM. Chronic complaints after
ankle sprains: a systematic review on effectiveness of treatments. J Orthop Sports Phys Ther. 2014
Nov;44(11):862-71, C1-23. doi: 10.2519/jospt.2014.5221. Epub 2014 Oct 9. Review. PubMed PMID:
25299494.

Monteleone BJ, Ronsky JL, Meeuwisse WH, Zernicke RF. Ankle kinematics and muscle
activity in functional ankle instability. Clin J Sport Med. 2014 Jan;24(1):62-8. doi: 10.1097/01.
jsm.0000432858.86929.80. PubMed PMID: 24231927.

© 2016 American Academy of Orthopaedic Surgeons 2016 Orthopaedic In-Training Examination


60 • American Academy of Orthopaedic Surgeons

Figure 251a Figure 251b

Figure 251c

Question 251
Figures 251a through 251c are the weight-bearing radiographs of a 19-year-old man who was involved
in a high-speed motor vehicle collision 6 months ago. Radiographs taken at the time of the incident were
read as “negative.” He has continued to experience pain with weight bearing and states that his “foot turns
outward.” What is the best next step?

1. Flatfoot reconstruction with tendon transfer, calcaneal osteotomy, and Achilles lengthening
2. Triple arthrodesis and Achilles lengthening
3. Closed reduction and percutaneous pinning
4. Sinus tarsi implant
5. Open reduction and primary midfoot arthrodesis

PREFERRED RESPONSE: 5

© 2016 American Academy of Orthopaedic Surgeons 2016 Orthopaedic In-Training Examination


SECTION 2: Foot and Ankle • 61

RECOMMENDED READINGS
Ly TV, Coetzee JC. Treatment of primarily ligamentous Lisfranc joint injuries: primary arthrodesis
compared with open reduction and internal fixation. A prospective, randomized study. J Bone Joint Surg
Am. 2006 Mar;88(3):514-20. PubMed PMID: 16510816.

Sangeorzan BJ, Veith RG, Hansen ST Jr. Salvage of Lisfranc's tarsometatarsal joint by arthrodesis. Foot
Ankle. 1990 Feb;10(4):193-200. PubMed PMID: 2307374.

Figure 260a Figure 260b

Question 260
Figures 260a and 260b are the radiographs of a 38-year-old man who fell from his bicycle and now has
ankle pain. His skin is intact; distal pulses are Dopplerable but biphasic. What is the most appropriate next
step?

1. CT scan
2. Vascular surgery consult
3. Closed reduction
4. Ankle brachial index
5. MRI

PREFERRED RESPONSE: 3

© 2016 American Academy of Orthopaedic Surgeons 2016 Orthopaedic In-Training Examination


62 • American Academy of Orthopaedic Surgeons

RECOMMENDED READINGS
Gardner MJ, Boraiah S, Hentel KD, Helfet DL, Lorich DG. The hyperplantarflexion ankle fracture variant.
J Foot Ankle Surg. 2007 Jul-Aug;46(4):256-60. PubMed PMID: 17586438.

White TO, Bugler KE: Ankle fractures. In Court-Brown C, Heckman JD, McKee M, McQueen MM,
Ricci W, Tornetta III P (eds): Rockwood and Green’s Fractures in Adults, 8th ed. Philadelphia, PA, Wolters
Kluwer, 2014: 2541-2592.

Figure 268a Figure 268b Figure 268c

Question 268
Figures 268a through 268c are the current standing radiographs of a 39-year-old woman who wants a
second opinion 6 months after undergoing surgical treatment performed elsewhere for an ankle fracture
dislocation. She has persistent ankle pain with weight bearing and was told by her treating physician that
she would require further surgery. What is the most appropriate revision surgery for this patient?

1. Total ankle arthroplasty


2. Revision fixation of the fibula and medial malleolus
3. Revision fixation of the medial malleolus
4. Revision fixation of the syndesmosis and medial malleolus
5. Ankle arthrodesis

PREFERRED RESPONSE: 2

© 2016 American Academy of Orthopaedic Surgeons 2016 Orthopaedic In-Training Examination


SECTION 2: Foot and Ankle • 63

RECOMMENDED READINGS
Chu A, Weiner L. Distal fibula malunions. J Am Acad Orthop Surg. 2009 Apr;17(4):220-30. Review.
PubMed PMID: 19307671.

Perera A, Myerson M. Surgical techniques for the reconstruction of malunited ankle fractures. Foot Ankle
Clin. 2008 Dec;13(4):737-51, ix. doi: 10.1016/j.fcl.2008.09.005. Review. PubMed PMID: 19013406.

© 2016 American Academy of Orthopaedic Surgeons 2016 Orthopaedic In-Training Examination


64 • American Academy of Orthopaedic Surgeons

SECTION 3: Hand

Question 8
Effective implementation of a surgical checklist program necessitates

1. adequate physician leadership.


2. worldwide adoption of the same checklist.
3. completion while the patient is going to sleep.
4. an available nurse to lead.
5. only hospital administrative support.

PREFERRED RESPONSE: 1

RECOMMENDED READINGS
Consistency for Safety in Orthopaedic Surgery, http://www.aaos.org/about/papers/advistmt/1042.asp
(Accessed November 9, 2015).

Gawande A: The Checklist Manifesto: How to Get Things Right. New York, NY, Metropolitan Books,
2009.

© 2016 American Academy of Orthopaedic Surgeons 2016 Orthopaedic In-Training Examination


SECTION 3: Hand • 65

Figure 15a Figure 15b

Question 15
Figures 15a and 15b are the radiographs of a 62-year-old right-hand-dominant woman who has right wrist
pain. Thirty years ago she underwent scapholunate ligament repair. She never fully regained wrist motion
or strength and reports that her pain and stiffness have progressed substantially during the last 5 years.
An examination reveals decreased wrist flexion and extension and tenderness along the radiocarpal joint.
She attempted bracing and corticosteroid injections without experiencing lasting symptom relief. Which
surgical treatment will most likely provide pain relief and preserve motion?

1. Complete radiocarpal arthrodesis


2. Scaphoid excision and 4-corner arthrodesis
3. Scaphoid distal pole excision
4. Scaphoid excision and radial styloidectomy
5. Proximal row carpectomy

PREFERRED RESPONSE: 2

RECOMMENDED READINGS
Tomaino MM, Miller RJ, Cole I, Burton RI. Scapholunate advanced collapse wrist: proximal row
carpectomy or limited wrist arthrodesis with scaphoid excision? J Hand Surg Am. 1994 Jan;19(1):134-42.
PubMed PMID: 8169358.

Strauch RJ. Scapholunate advanced collapse and scaphoid nonunion advanced collapse arthritis--update
on evaluation and treatment. J Hand Surg Am. 2011 Apr;36(4):729-35. doi: 10.1016/j.jhsa.2011.01.018.
Review. PubMed PMID: 21463735.

© 2016 American Academy of Orthopaedic Surgeons 2016 Orthopaedic In-Training Examination


66 • American Academy of Orthopaedic Surgeons

Figure 28

Question 28
Figure 28 is the ultrasound of a 23-year-old patient who has had a volar radial 1.5-cm tender and painful
wrist mass for 6 months. The additional workup prior to surgery should consist of

1. serum and urine protein electrophoresis.


2. a chest CT scan.
3. positron emission tomography.
4. MRI with intravenous contrast.
5. age-appropriate presurgical laboratory studies.

PREFERRED RESPONSE: 5

RECOMMENDED READINGS
Amrami KK, Bishop AT, Berger RA: Radiology Corner: Imaging Soft-Tissue Tumors of the Hand and
Wrist: Case Presentation and Discussion, Journal of the American Society for Surgery of the Hand,
2005;Volume 5(Issue 4):186-192. http://www.jhandsurg.org/article/S1531-0914(05)00149-X/abstract

Mayerson JL, Scharschmidt TJ, Lewis VO, Morris CD. Diagnosis and Management of Soft-tissue Masses.
J Am Acad Orthop Surg. 2014 Nov;22(11):742-50. doi: 10.5435/JAAOS-22-11-742. PubMed PMID:
25344599.

© 2016 American Academy of Orthopaedic Surgeons 2016 Orthopaedic In-Training Examination


SECTION 3: Hand • 67

Figure 39

Question 39
Figure 39 is the right hand of a 65-year-old man with a history of hypertension and rheumatoid arthritis.
He is taking immunosuppressive disease-modifying antirheumatic drugs (DMARDs) and is seen in the
emergency department with rapid progression of erythema from his right thumb to his right arm during
the last 12 hours. He is confused, lethargic, and has these vital signs: blood pressure 92/40, respiratory
rate 45, temperature 39.7°C, pulse 135, and oxygen saturation 90% on 4 liters of oxygen by face mask.
An examination of his right upper extremity reveals black bulla extending from the metacarpophalangeal
down to the tip and no capillary refill at the pulp. Immediate treatment should consist of

1. intravenous (IV) antibiotics and admission to a medical intensive care unit.


2. emergent radical debridement including thumb amputation.
3. emergent revascularization of the thumb with a vein graft.
4. urgent irrigation of the thumb flexor tendon sheath.
5. urgent debridement of the felon through a midlateral incision.

PREFERRED RESPONSE: 2

RECOMMENDED READINGS
Gonzalez MH, Kay T, Weinzweig N, Brown A, Pulvirenti J. Necrotizing fasciitis of the upper extremity. J
Hand Surg Am. 1996 Jul;21(4):689-92. PubMed PMID: 8842969.

Wong CH, Khin LW, Heng KS, Tan KC, Low CO. The LRINEC (Laboratory Risk Indicator for
Necrotizing Fasciitis) score: a tool for distinguishing necrotizing fasciitis from other soft tissue infections.
Crit Care Med. 2004 Jul;32(7):1535-41. PubMed PMID: 15241098.

© 2016 American Academy of Orthopaedic Surgeons 2016 Orthopaedic In-Training Examination


68 • American Academy of Orthopaedic Surgeons

Question 48
A 16-year-old high school football player injured his ring finger during a game 5 days before his clinic
visit. He reports pain and swelling at the proximal interphalangeal (PIP) joint and difficulty moving his
finger. Which examination finding is most suggestive of a central slip disruption?

1. With the PIP joint flexed to 90 degrees, attempted extension against resistance results in distal
interphalangeal (DIP) hyperextension
2. With the PIP joint flexed to 90 degrees, the patient cannot actively extend the DIP joint
against resistance
3. With the PIP joint in extension, the patient cannot actively extend the DIP joint
against resistance
4. The finger rests in a boutonniere posture that cannot be passively corrected
5. The finger rests in a swan-neck posture that cannot be passively corrected

PREFERRED RESPONSE: 1

RECOMMENDED READINGS
Elson RA. Rupture of the central slip of the extensor hood of the finger. A test for early diagnosis. J Bone
Joint Surg Br. 1986 Mar;68(2):229-31. PubMed PMID: 3958008.

Rubin J, Bozentka DJ, Bora FW. Diagnosis of closed central slip injuries. A cadaveric analysis of non-
invasive tests. J Hand Surg Br. 1996 Oct;21(5):614-6. PubMed PMID: 9230945.

© 2016 American Academy of Orthopaedic Surgeons 2016 Orthopaedic In-Training Examination


SECTION 3: Hand • 69

Video 58 (scan to view)

Question 58
The examination finding shown in Video 58 is consistent with which defect?

1. Trigger finger
2. Flexor digitorum profundus (FDP) incompetence
3. Flexor digitorum sublimis (FDS) incompetence
4. Extensor digitorum communis (EDC) incompetence
5. Extensor digiti quinti (EDQ) incompetence

PREFERRED RESPONSE: 2

RECOMMENDED READINGS
Strickland JW. Flexor Tendon Injuries: I. Foundations of Treatment. J Am Acad Orthop Surg. 1995
Jan;3(1):44-54. PubMed PMID: 10790652.

Wolfe SW, Pederson WC, Hotchkiss RN, Kozin SH, eds. Green’s Operative Hand Surgery, 6th ed.
Philadelphia, PA: Churchill Livingstone Elsevier; 2011:189.

© 2016 American Academy of Orthopaedic Surgeons 2016 Orthopaedic In-Training Examination


70 • American Academy of Orthopaedic Surgeons

Figure 65a Figure 65b

Question 65
Figure 65a is the clinical photograph of a 64-year-old man who crashed while riding his motorcycle. An
examination reveals his long-finger metacarpophalangeal (MP) joint is stuck in extension. He cannot
passively or actively flex at the MP joint. A hand radiograph is seen in Figure 65b. Which interposed
structure is preventing reduction?

1. Flexor tendons
2. Lateral band
3. Extensor hood
4. Lumbrical
5. Volar plate

PREFERRED RESPONSE: 5

RECOMMENDED READINGS
Afifi AM, Medoro A, Salas C, Taha MR, Cheema T. A cadaver model that investigates irreducible
metacarpophalangeal joint dislocation. J Hand Surg Am. 2009 Oct;34(8):1506-11. doi: 10.1016/j.
jhsa.2009.06.001. Epub 2009 Aug 22. PubMed PMID: 19703736.

Bohart PG, Gelberman RH, Vandell RF, Salamon PB. Complex dislocations of the metacarpophalangeal
joint. Clin Orthop Relat Res. 1982 Apr;(164):208-10. PubMed PMID: 7067288.

© 2016 American Academy of Orthopaedic Surgeons 2016 Orthopaedic In-Training Examination


SECTION 3: Hand • 71

Question 75
A router injury leaves a skin and soft-tissue defect over the volar middle phalanx of the long finger for a
32-year-old healthy man. There is a 2.2-cm x 1.2-cm area of exposed flexor tendon without tenosynovium.
What is the best option for defect coverage?

1. Full-thickness skin graft


2. Split-thickness skin graft
3. First dorsal metacarpal artery flap
4. Moberg advancement flap
5. Cross-finger flap

PREFERRED RESPONSE: 5

RECOMMENDED READINGS
Rehim SA, Chung KC. Local flaps of the hand. Hand Clin. 2014 May;30(2):137-51, v. doi: 10.1016/j.
hcl.2013.12.004. Review. PubMed PMID: 24731606.

Eberlin KR, Chang J, Curtin CM, Sammer DM, Saint-Cyr M, Taghinia AH. Soft-tissue coverage
of the hand: a case-based approach. Plast Reconstr Surg. 2014 Jan;133(1):91-101. doi: 10.1097/01.
prs.0000436831.73323.88. PubMed PMID: 24105089.

Question 88
A 70-year-old woman reports decreased thumb strength and intermittent numbness in her index and
long fingers for 1 month. She also has radiographic thumb carpometacarpal (CMC) arthritis and 4/5
thenar muscle strength without atrophy. Nerve conduction studies and electromyography (EMG) reveal
fibrillations within the abductor pollicis brevis and median nerve sensory latency of 3.6 ms (< 3.5 ms is
normal). What is the best next step?

1. Carpal tunnel release


2. Carpal tunnel and Guyon’s canal release
3. Cervical spine MRI and neurologic evaluation
4. Corticosteroid injection in the thumb CMC joint
5. Wrist neutral splint with corticosteroid injection in the carpal tunnel

PREFERRED RESPONSE: 1

RECOMMENDED READINGS
Kronlage SC, Menendez ME. The benefit of carpal tunnel release in patients with electrophysiologically
moderate and severe disease. J Hand Surg Am. 2015 Mar;40(3):438-444.e1. doi: 10.1016/j.
jhsa.2014.12.012. PubMed PMID: 25708432.

Ono S, Clapham PJ, Chung KC. Optimal management of carpal tunnel syndrome. Int J Gen Med. 2010
Aug 30;3:255-61. PubMed PMID: 20830201.

© 2016 American Academy of Orthopaedic Surgeons 2016 Orthopaedic In-Training Examination
72 • American Academy of Orthopaedic Surgeons

Question 96
A 54-year-old laborer has a 6-month history of lateral elbow pain. An elbow examination reveals full
range of motion, tenderness over the lateral epicondyle, and pain with resisted wrist extension with
the elbow in extension. Elbow radiograph findings are normal. You perform a steroid injection and the
patient’s symptoms are decreased 6 weeks later. One year after receiving the injection, this patient—when
compared to a patient who did not have a steroid injection—is likely to

1. have no difference in elbow pain.


2. no longer have elbow pain.
3. need surgery.
4. experience tendon rupture.
5. change occupations.

PREFERRED RESPONSE: 1

RECOMMENDED READINGS
Coombes BK, Bisset L, Brooks P, Khan A, Vicenzino B. Effect of corticosteroid injection, physiotherapy,
or both on clinical outcomes in patients with unilateral lateral epicondylalgia: a randomized controlled
trial. JAMA. 2013 Feb 6;309(5):461-9. doi: 10.1001/jama.2013.129. PubMed PMID: 23385272.

Dong W, Goost H, Lin XB, Burger C, Paul C, Wang ZL, Kong FL, Welle K, Jiang ZC, Kabir K. Injection
therapies for lateral epicondylalgia: a systematic review and Bayesian network meta-analysis. Br J Sports
Med. 2016 Aug;50(15):900-8. doi: 10.1136/bjsports-2014-094387. Epub 2015 Sep 21. PubMed PMID:
26392595.

© 2016 American Academy of Orthopaedic Surgeons 2016 Orthopaedic In-Training Examination


SECTION 3: Hand • 73

Figure 103a Figure 103b

Figure 103c Figure 103d

Question 103
Figures 103a through 103d are the radiographs, sagittal-cut CT scan, and coronal T1 MR image of a
16-year-old boy who has wrist stiffness and pain after sustaining an injury 2 years ago. There is no
bleeding from the proximal pole during surgery. Which procedure will most likely result in restoration of
alignment and healing?

1. 1,2 intercompartmental supraretinacular artery (ICSRA) graft


2. Free-vascularized medial femoral condyle graft
3. Iliac crest corticocancellous graft
4. 4+5 extensor compartmental artery (ECA) vascularized bone graft
5. Cancellous distal radius autograft

PREFERRED RESPONSE: 2

© 2016 American Academy of Orthopaedic Surgeons 2016 Orthopaedic In-Training Examination


74 • American Academy of Orthopaedic Surgeons

RECOMMENDED READINGS
Moon ES, Dy CJ, Derman P, Vance MC, Carlson MG. Management of nonunion following surgical
management of scaphoid fractures: current concepts. J Am Acad Orthop Surg. 2013 Sep;21(9):548-57.
doi: 10.5435/JAAOS-21-09-548. Review. PubMed PMID: 23996986.

Jones DB Jr, Bürger H, Bishop AT, Shin AY. Treatment of scaphoid waist nonunions with an avascular
proximal pole and carpal collapse. A comparison of two vascularized bone grafts. J Bone Joint Surg Am.
2008 Dec;90(12):2616-25. doi: 10.2106/JBJS.G.01503. PubMed PMID: 19047706.

Question 133
A 47-year-old woman has finger pain in all digits of her left hand. She denies a history of trauma or a
connective tissue disorder. Cold exposure causes color changes in her fingers and exacerbates her pain.
An examination reveals 2+ radial and ulnar pulses. The thumb, index, and long fingers display sluggish
capillary refill, and small ulcers appear on her fingertips. There is no erythema or drainage from the ulcers.
Which treatment will most likely decrease her pain and improve finger perfusion?

1. Stellate ganglion block


2. Botulinum toxin A injections
3. Digital amputation
4. Oral gabapentin
5. Oral methotrexate

PREFERRED RESPONSE: 2

RECOMMENDED READINGS
Iorio ML, Masden DL, Higgins JP. Botulinum toxin A treatment of Raynaud’s phenomenon: a review.
Semin Arthritis Rheum. 2012 Feb;41(4):599-603. doi: 10.1016/j.semarthrit.2011.07.006. Epub 2011 Aug
24. Review. PubMed PMID: 21868066.

Neumeister MW, Webb KN, Romanelli M. Minimally invasive treatment of Raynaud phenomenon: the
role of botulinum type A. Hand Clin. 2014 Feb;30(1):17-24. doi: 10.1016/j.hcl.2013.09.006. Review.
PubMed PMID: 24286738.

© 2016 American Academy of Orthopaedic Surgeons 2016 Orthopaedic In-Training Examination


SECTION 3: Hand • 75

Figure 140

Question 140
A 32-year-old man sustained the thumb metacarpophalangeal (MP) joint ulnar collateral ligament (UCL)
injury seen in Figure 140. During repair, which structure blocks reduction of the ligament?

1. Extensor pollicis longus (EPL) tendon


2. Extensor pollicis brevis (EPB) tendon
3. EPB and dorsal capsule
4. Ulnar sesamoid bone and adductor muscle
5. Adductor aponeurosis

PREFERRED RESPONSE: 5

RECOMMENDED READINGS
STENER B. Skeletal injuries associated with rupture of the ulnar collateral ligament of the
metacarpophalangeal joint of the thumb. A clinical and anatomical study. Acta Chir Scand. 1963
Jun;125:583-6. PubMed PMID: 13983826.

Bean CH, Tencer AF, Trumble TE. The effect of thumb metacarpophalangeal ulnar collateral ligament
attachment site on joint range of motion: an in vitro study. J Hand Surg Am. 1999 Mar;24(2):283-7.
PubMed PMID: 10194011.

Carlson MG, Warner KK, Meyers KN, Hearns KA, Kok PL. Anatomy of the thumb metacarpophalangeal
ulnar and radial collateral ligaments. J Hand Surg Am. 2012 Oct;37(10):2021-6. doi: 10.1016/j.
jhsa.2012.06.024. Epub 2012 Aug 31. PubMed PMID: 22939823.

© 2016 American Academy of Orthopaedic Surgeons 2016 Orthopaedic In-Training Examination


76 • American Academy of Orthopaedic Surgeons

Question 175
A 63-year-old woman is seen 3 weeks after sustaining a closed minimally displaced distal radius fracture.
She has been in a short-arm cast and reports minimal pain but notes that she is having difficulty using her
thumb. You suspect she may have an extensor pollicis longus (EPL) rupture. Which examination finding
would confirm lack of EPL function?

1. A positive Froment sign


2. Thenar atrophy
3. Inability to lift her thumb off of the examination table with her palm flat
4. Inability to abduct her thumb from the palm with her palm flat on the table
5. Tenderness over the third dorsal compartment

PREFERRED RESPONSE: 3

RECOMMENDED READINGS
Roth KM, Blazar PE, Earp BE, Han R, Leung A. Incidence of extensor pollicis longus tendon rupture
after nondisplaced distal radius fractures. J Hand Surg Am. 2012 May;37(5):942-7. doi: 10.1016/j.
jhsa.2012.02.006. Epub 2012 Mar 29. PubMed PMID: 22463927.

Kulshreshtha R, Patel S, Arya AP, Hall S, Compson JP. Variations of the extensor pollicis brevis tendon
and its insertion: a study of 44 cadaveric hands. J Hand Surg Eur Vol. 2007 Oct;32(5):550-3. Epub 2007
Jun 12. PubMed PMID: 17950220.

© 2016 American Academy of Orthopaedic Surgeons 2016 Orthopaedic In-Training Examination


SECTION 3: Hand • 77

Figure 228

Question 228
While attempting to recreate the inclination of the distal radius during volar fixation of an intra-articular
sagittal split fracture, use of intraoperative fluoroscopic imaging in the position shown in Figure 228
would be helpful in showing

1. intra-articular screw penetration.


2. alignment of the joint surface.
3. alignment of the sigmoid notch.
4. carpal alignment.
5. dorsal screw penetration.

PREFERRED RESPONSE: 1

RECOMMENDED READINGS
Tweet ML, Calfee RP, Stern PJ. Rotational fluoroscopy assists in detection of intra-articular screw
penetration during volar plating of the distal radius. J Hand Surg Am. 2010 Apr;35(4):619-27. doi:
10.1016/j.jhsa.2009.12.033. Epub 2010 Mar 3. PubMed PMID: 20202762.

Patel A, Culbertson MD, Lahey P, Semenovski M, Choueka J. Improving accuracy and confidence
in distal radius volar plate screw placement through supplemental radiography: examining specialty,
education, and experience levels. Hand (N Y). 2013 Sep;8(3):308-14. doi: 10.1007/s11552-013-9528-3.
PubMed PMID: 24426939.

© 2016 American Academy of Orthopaedic Surgeons 2016 Orthopaedic In-Training Examination


78 • American Academy of Orthopaedic Surgeons

Question 232
A 50-year-old man with a history of tobacco use sustained a traumatic amputation of his dominant
hand index finger through the proximal phalanx while using a table saw. He arrived at the emergency
department 6 hours after the injury with the amputated digit wrapped in moist gauze in a plastic bag inside
a cooler with ice. Replantation should not be performed because of

1. prolonged ischemia time.


2. a history of tobacco use.
3. poor function of the replanted digit.
4. the patient’s age.
5. the table saw mechanism.

PREFERRED RESPONSE: 3

RECOMMENDED READINGS
Urbaniak JR, Roth JH, Nunley JA, Goldner RD, Koman LA. The results of replantation after amputation
of a single finger. J Bone Joint Surg Am. 1985 Apr;67(4):611-9. PubMed PMID: 3980507.

Boulas HJ. Amputations of the fingers and hand: indications for replantation. J Am Acad Orthop Surg.
1998 Mar-Apr;6(2):100-5. PubMed PMID: 9682072.

Question 245
In the setting of a closed zone 3 (over the proximal interphalangeal joint [PIP]) extensor tendon injury
with a central tendon injury of the long finger, which active motion plan is encouraged?

1. PIP joint flexion


2. PIP extension
3. Distal interphalangeal joint (DIPJ) flexion
4. DIPJ flexion and extension
5. Metacarpophalangeal joint flexion and PIP joint extension

PREFERRED RESPONSE: 4

RECOMMENDED READINGS
Posner MA, Green SM. Diagnosis and treatment of finger deformities following injuries to the extensor
tendon mechanism. Hand Clin. 2013 May;29(2):269-81. doi: 10.1016/j.hcl.2013.03.003. PubMed PMID:
23660063.

Scott SC. Closed injuries to the extension mechanism of the digits. Hand Clin.2000 Aug;16(3):367-73,
viii. Review. PubMed PMID: 10955210.

© 2016 American Academy of Orthopaedic Surgeons 2016 Orthopaedic In-Training Examination


SECTION 3: Hand • 79

Question 246
A 41-year-old right-hand-dominant man has a 1-month history of weakness in his right hand. He denies a
history of trauma or change in activity. He specifically notes difficulty with pinch. An examination of his
right hand reveals normal sensation in a radial, median, and ulnar distribution. He is unable to actively
flex his thumb interphalangeal (IP) joint and index distal interphalangeal (DIP) joint but has normal digital
passive motion. What is the most likely diagnosis?

1. Carpal tunnel syndrome


2. Flexor pollicis longus (FPL) and flexor digitorum profundus (FDP) rupture
3. Anterior interosseous nerve (AIN) palsy
4. Cervical radiculopathy
5. IP arthritis of the thumb and finger

PREFERRED RESPONSE: 3

RECOMMENDED READINGS
Rodner CM, Tinsley BA, O'Malley MP. Pronator syndrome and anterior interosseous nerve syndrome.
J Am Acad Orthop Surg. 2013 May;21(5):268-75. doi: 10.5435/JAAOS-21-05-268. Review. PubMed
PMID: 23637145.

Park IJ, Roh YT, Jeong C, Kim HM. Spontaneous anterior interosseous nerve syndrome:
clinical analysis of eleven surgical cases. J Plast Surg Hand Surg. 2013 Dec;47(6):519-23. doi:
10.3109/2000656X.2013.791624. Epub 2013 Apr 30. PubMed PMID: 23627594.

© 2016 American Academy of Orthopaedic Surgeons 2016 Orthopaedic In-Training Examination


80 • American Academy of Orthopaedic Surgeons

Figure 256a Figure 256b

Figure 256c Figure 256d

Question 256
Figures 256a through 256d are the wrist MR images of a 43-year-old right-hand-dominant bricklayer who
reports gradually progressive left hand weakness for 4 months. He describes difficulty gripping objects,
tying his shoes, and holding utensils. He denies any numbness, paresthesias, or a previous injury. An
examination reveals intact sensation in a median, radial, and ulnar nerve distribution. He has atrophy of
hand interossei and a positive Froment sign finding. He has no Tinel sign finding at the wrist or elbow and
no exacerbation of symptoms with elbow hyperflexion. Electromyography shows signs of denervation in
an ulnar nerve distribution distal to the wrist. What is the best next step?

1. Cubital tunnel release


2. Guyon’s canal release
3. Hook-of-hamate excision
4. Chest radiograph and positron emission tomography scan
5. Excision of the ganglion cyst

PREFERRED RESPONSE: 5

© 2016 American Academy of Orthopaedic Surgeons 2016 Orthopaedic In-Training Examination


SECTION 3: Hand • 81

RECOMMENDED READINGS
Wang B, Zhao Y, Lu A, Chen C. Ulnar nerve deep branch compression by a ganglion: a review of nine
cases. Injury. 2014 Jul;45(7):1126-30. doi: 10.1016/j.injury.2014.03.017. Epub 2014 Apr 3. PubMed
PMID: 24774037.

Maroukis BL, Ogawa T, Rehim SA, Chung KC. Guyon canal: the evolution of clinical anatomy. J Hand
Surg Am. 2015 Mar;40(3):560-5. doi: 10.1016/j.jhsa.2014.09.026. Epub 2014 Oct 29. PubMed PMID:
25446410.

Question 263
A 25-year-old man has an acute scaphoid fracture in the proximal third of the scaphoid. The fracture is
displaced 1 mm and has a lateral intrascaphoid angle of 55 degrees (a normal angle is about 45 degrees).
What is the best next step?

1. Long-arm cast with thumb spica


2. Percutaneous in situ screw fixation
3. Closed reduction and pin fixation with casting
4. Open reduction and internal fixation (ORIF) through a volar approach
5. ORIF through a dorsal approach

PREFERRED RESPONSE: 5

RECOMMENDED READINGS
Rettig ME, Kozin SH, Cooney WP. Open reduction and internal fixation of acute displaced scaphoid waist
fractures. J Hand Surg Am. 2001 Mar;26(2):271-6. PubMed PMID: 11279573.

Raskin KB, Parisi D, Baker J, Rettig ME. Dorsal open repair of proximal pole scaphoid fractures. Hand
Clin. 2001 Nov;17(4):601-10, ix. PubMed PMID: 11775471.

Question 267
A 65-year-old woman has severe pain and numbness in her hand. She also has pain radiating from her
neck down to her arm and hand. An examination reveals a positive Tinel sign result at the midforearm.
Electrodiagnostic testing shows a median nerve sensory distal latency of 3.8 ms (normal latency is 3.5
ms). Which intervention or test will be most effective while trying to predict carpal tunnel release success?

1. Trigger-point lidocaine injection


2. Carpal tunnel corticosteroid injection
3. Wrist ultrasound
4. Median nerve exploration
5. More electrodiagnostic testing

PREFERRED RESPONSE: 2
© 2016 American Academy of Orthopaedic Surgeons 2016 Orthopaedic In-Training Examination
82 • American Academy of Orthopaedic Surgeons

RECOMMENDED READINGS
Ponnappan RK, Khan M, Matzon JL, Sheikh ES, Tucker BS, Pepe MD, Tjoumakaris FP, Nassr AN.
Clinical Differentiation of Upper Extremity Pain Etiologies. J Am Acad Orthop Surg. 2015 Aug;23(8):492-
500. doi: 10.5435/JAAOS-D-11-00086. Epub 2015 Jun 26. Review. PubMed PMID: 26116851.

Kane PM, Daniels AH, Akelman E. Double Crush Syndrome. J Am Acad Orthop Surg. 2015
Sep;23(9):558-62. doi: 10.5435/JAAOS-D-14-00176. Review. PubMed PMID: 26306807.

Question 271
A 49-year-old man has painless nodules on his dorsal right index finger and ring finger proximal
interphalangeal (PIP) joints that have been slowly growing for 3 months. The lesions never bleed, and
the overlying skin appears normal. A hand examination reveals well-circumscribed fleshy nodules on the
dorsum of the index and ring fingers, each measuring about 6 mm in diameter. He has full range of motion
of his index finger, but has a 10-degree flexion contracture of his ring finger PIP joint and a 10-degree
contracture of his ring finger metacarpophalangeal joint. A palm examination reveals a palpable cord over
the volar ring finger. He has normal sensation and vascularity to his fingertips. The best treatment for these
nodules is

1. no intervention.
2. marginal excisional biopsy.
3. excisional biopsy with 5-mm margins.
4. a collagenase injection.
5. topical diclofenac gel.

PREFERRED RESPONSE: 1

RECOMMENDED READINGS
Rayan GM, Ali M, Orozco J. Dorsal pads versus nodules in normal population and Dupuytren's disease
patients. J Hand Surg Am. 2010 Oct;35(10):1571-9. doi: 10.1016/j.jhsa.2010.06.001. PubMed PMID:
20800974.

Rayan GM. Dupuytren’s disease: anatomy, pathology, presentation, and treatment. Instr Course Lect.
2007;56:101-11. Review. PubMed PMID: 17472297.

© 2016 American Academy of Orthopaedic Surgeons 2016 Orthopaedic In-Training Examination


SECTION 4: Hip and Knee Reconstruction • 83

SECTION 4: Hip and Knee Reconstruction

Figure 5a Figure 5b

Question 5
Figures 5a and 5b are the anteroposterior (AP) radiographs of a 56-year-old woman who had right total
knee arthroplasty (TKA) 5 years ago. She now has pain only with activity. An examination reveals she
has a well-healed midline skin incision without erythema or drainage. Her knee range of motion is 0 to
125 degrees. She has a stable examination to varus/valgus stress and less than 1 cm of anterior-posterior
translation in 90 degrees of flexion. She underwent knee aspiration and had a cell count of 875 with 20%
neutrophils. Her C-reactive protein (CRP) level is within defined limits. What is the most likely cause for
her knee arthroplasty failure?

1. Septic loosening
2 Aseptic loosening
3. Polyethylene wear
4. Instability
5. Malalignment

PREFERRED RESPONSE: 2

RECOMMENDED READINGS
Schroer WC, Berend KR, Lombardi AV, Barnes CL, Bolognesi MP, Berend ME, Ritter MA, Nunley RM.
Why are total knees failing today? Etiology of total knee revision in 2010 and 2011. J Arthroplasty. 2013
Sep;28(8 Suppl):116-9. doi: 10.1016/j.arth.2013.04.056. Epub 2013 Aug 15. PubMed PMID: 23954423.

Sharkey PF, Lichstein PM, Shen C, Tokarski AT, Parvizi J. Why are total knee arthroplasties failing
today--has anything changed after 10 years? J Arthroplasty. 2014 Sep;29(9):1774-8. doi: 10.1016/j.
arth.2013.07.024. Epub 2014 Jul 5. PubMed PMID: 25007726.

© 2016 American Academy of Orthopaedic Surgeons 2016 Orthopaedic In-Training Examination
84 • American Academy of Orthopaedic Surgeons

Figure 20a Figure 20b

Question 20
A 66-year-old man has right hip pain after undergoing total hip arthroplasty 20 years ago. His erythrocyte
sedimentation rate and C-reactive protein levels are within defined limits. Which molecule is mediating
the issue shown in Figures 20a and 20b?

1. Farnesyl transferase
2. Matrix metalloproteinase-13
3. Activating transcription factor 6
4. Runt-related transcription factor 2
5. Receptor activator of nuclear factor kappa

PREFERRED RESPONSE: 5

RECOMMENDED READINGS
Ramage SC, Urban NH, Jiranek WA, Maiti A, Beckman MJ. Expression of RANKL in osteolytic
membranes: association with fibroblastic cell markers. J Bone Joint Surg Am. 2007 Apr;89(4):841-8.
PubMed PMID: 17403809.

Granchi D, Pellacani A, Spina M, Cenni E, Savarino LM, Baldini N, Giunti A. Serum levels of
osteoprotegerin and receptor activator of nuclear factor-kappaB ligand as markers of periprosthetic
osteolysis. J Bone Joint Surg Am. 2006 Jul;88(7):1501-9. PubMed PMID: 16818976.

© 2016 American Academy of Orthopaedic Surgeons 2016 Orthopaedic In-Training Examination


SECTION 4: Hip and Knee Reconstruction • 85

Question 30
A 75-year-old woman underwent a total knee arthroplasty (TKA). A nerve blockade was performed to
manage her perioperative pain. During her postoperative visit 2 hours after surgery, she has numbness
over the medial aspect of her leg, is able to extend and flex her knee, and can perform a straight-leg raise.
Which type of preoperative nerve block was most likely administered?

1. Femoral nerve
2. Adductor canal
3. Obturator nerve
4. Sciatic nerve
5. Peroneal nerve

PREFERRED RESPONSE: 2

RECOMMENDED READINGS
Shah NA, Jain NP, Panchal KA. Adductor Canal Blockade Following Total Knee Arthroplasty-Continuous
or Single Shot Technique? Role in Postoperative Analgesia, Ambulation Ability and Early Functional
Recovery: A Randomized Controlled Trial. J Arthroplasty. 2015 Aug;30(8):1476-81. doi: 10.1016/j.
arth.2015.03.006. Epub 2015 Mar 16. PubMed PMID: 25824025.

Mudumbai SC, Kim TE, Howard SK, Workman JJ, Giori N, Woolson S, Ganaway T, King R, Mariano
ER. Continuous adductor canal blocks are superior to continuous femoral nerve blocks in promoting early
ambulation after TKA. Clin Orthop Relat Res. 2014 May;472(5):1377-83. doi: 10.1007/s11999-013-
3197-y. PubMed PMID: 23897505.

© 2016 American Academy of Orthopaedic Surgeons 2016 Orthopaedic In-Training Examination


86 • American Academy of Orthopaedic Surgeons

Figure 44

Question 44
The patient in Figure 44 is being seen for knee arthroplasty. During the trialing of components, the knee
is found to be tight in flexion on the lateral side and balanced in extension. To further balance the knee,
which structure should be released to help balance the medial and lateral gaps in flexion?

1. Iliotibial band
2. Popliteus
3. Lateral collateral ligament (LCL)
4. Posterior cruciate ligament
5. Posterior oblique ligament

PREFERRED RESPONSE: 2

RECOMMENDED READINGS
Elkus M, Ranawat CS, Rasquinha VJ, Babhulkar S, Rossi R, Ranawat AS. Total knee arthroplasty
for severe valgus deformity. Five to fourteen-year follow-up. J Bone Joint Surg Am. 2004
Dec;86-A(12):2671-6. PubMed PMID: 15590852.

Peters CL, Jimenez C, Erickson J, Anderson MB, Pelt CE. Lessons learned from selective soft-tissue
release for gap balancing in primary total knee arthroplasty: an analysis of 1216 consecutive total knee
arthroplasties: AAOS exhibit selection. J Bone Joint Surg Am. 2013 Oct 16;95(20):e152. doi: 10.2106/
JBJS.L.01686. PubMed PMID: 24132367.

© 2016 American Academy of Orthopaedic Surgeons 2016 Orthopaedic In-Training Examination


SECTION 4: Hip and Knee Reconstruction • 87

Question 50
A 62-year-old woman with rheumatoid arthritis underwent left total knee arthroplasty and continues
to have pain and swelling. Her presurgical knee alignment was 25-degree valgus. Standard posterior
stabilized implants and soft-tissue releases were used. One year after surgery, she has a 2+ effusion and
2+ lateral laxity at 90 degrees. Nonsurgical treatment options have failed. What is the most appropriate
surgical option?

1. Femur revision with a posterior medial augment


2. Femur revision with a posterior lateral augment
3. Femur revision with a distal augment
4. Lateral retinacular release
5. Medial soft-tissue reefing

PREFERRED RESPONSE: 2

RECOMMENDED READINGS
Favorito PJ, Mihalko WM, Krackow KA. Total knee arthroplasty in the valgus knee. J Am Acad Orthop
Surg. 2002 Jan-Feb;10(1):16-24. Review. PubMed PMID: 11809047.

Ranawat AS, Ranawat CS, Elkus M, Rasquinha VJ, Rossi R, Babhulkar S. Total knee arthroplasty for
severe valgus deformity. J Bone Joint Surg Am. 2005 Sep;87 Suppl 1(Pt 2):271-84. PubMed PMID:
16140800.

Question 59
Soft-tissue releases are performed during total knee arthroplasty (TKA) for a patient with an 8-degree
presurgical varus deformity. During the process of assessing flexion and extension gaps, it is noted that
a lateral tendinous structure was inadvertently completely released when removing posterior lateral
osteophytes. What is the best next step?

1. Posterior stabilized TKA


2. Allograft lateral collateral ligament reconstruction
3. Constrained TKA
4. Anterior-posterior stabilized polyethylene
5. Medial retinacular reefing

PREFERRED RESPONSE: 1

© 2016 American Academy of Orthopaedic Surgeons 2016 Orthopaedic In-Training Examination


88 • American Academy of Orthopaedic Surgeons

RECOMMENDED READINGS
Kesman TJ, Kaufman KR, Trousdale RT. Popliteus tendon resection during total knee arthroplasty: an
observational report. Clin Orthop Relat Res. 2011 Jan;469(1):76-81. doi: 10.1007/s11999-010-1525-z.
PubMed PMID: 20809169.

Ghosh KM, Hunt N, Blain A, Athwal KK, Longstaff L, Amis AA, Rushton S, Deehan DJ. Isolated
popliteus tendon injury does not lead to abnormal laxity in posterior-stabilised total knee arthroplasty.
Knee Surg Sports Traumatol Arthrosc. 2015 Jun;23(6):1763-9. doi: 10.1007/s00167-014-3488-1. Epub
2015 Jan 1. PubMed PMID: 25552404.

Figure 76a Figure 76b

Question 76
A 68-year-old woman fell and sustained the injury shown in Figures 76a and 76b. An intraoperative cell
count of hip synovial fluid yields 5500 total nucleated cells and 11 cells/hpf on 7 fields. What is the best
next step?

1. Femur open reduction and internal fixation (ORIF) with plate and cables/screws
2. Femur ORIF with a revision femoral component
3. Femur ORIF with antibiotic spacer placement
4. Femur ORIF with a head and liner exchange
5. Intravenous (IV) antibiotics with femur ORIF

PREFERRED RESPONSE: 3

© 2016 American Academy of Orthopaedic Surgeons 2016 Orthopaedic In-Training Examination


SECTION 4: Hip and Knee Reconstruction • 89

RECOMMENDED READINGS
Preston S, Somerville L, Lanting B, Howard J. Are Nucleated Cell Counts Useful in the Diagnosis of
Infection in Periprosthetic Fracture? Clin Orthop Relat Res. 2015 Jul;473(7):2238-43. doi: 10.1007/
s11999-015-4162-8. PubMed PMID: 25631172.

Della Valle C, Parvizi J, Bauer TW, Dicesare PE, Evans RP, Segreti J, Spangehl M, Watters WC
3rd, Keith M, Turkelson CM, Wies JL, Sluka P, Hitchcock K; American Academy of Orthopaedic
Surgeons. Diagnosis of periprosthetic joint infections of the hip and knee. J Am Acad Orthop Surg. 2010
Dec;18(12):760-70. PubMed PMID: 21119142.

Question 98
For patients who develop stiffness with decreased flexion following total knee arthroplasty (TKA), the
recommended time frame from date of initial injury for manipulation under anesthesia is within

1. 1 month.
2. 3 months.
3. 6 months.
4. 9 months.
5. 12 months.

PREFERRED RESPONSE: 2

RECOMMENDED READINGS
Issa K, Banerjee S, Kester MA, Khanuja HS, Delanois RE, Mont MA. The effect of timing of
manipulation under anesthesia to improve range of motion and functional outcomes following total knee
arthroplasty. J Bone Joint Surg Am. 2014 Aug 20;96(16):1349-57. doi: 10.2106/JBJS.M.00899. PubMed
PMID: 25143495.

Desai AS, Karmegam A, Dramis A, Board TN, Raut V. Manipulation for stiffness following total knee
arthroplasty: when and how often to do it? Eur J Orthop Surg Traumatol. 2014 Oct;24(7):1291-5. doi:
10.1007/s00590-013-1387-7. Epub 2013 Dec 11. PubMed PMID: 24327007.

© 2016 American Academy of Orthopaedic Surgeons 2016 Orthopaedic In-Training Examination


90 • American Academy of Orthopaedic Surgeons

Question 108
Total hip arthroplasty is planned for a 68-year-old woman who has had ongoing treatment with etanercept
to address rheumatoid arthritis. Appropriate management of this medication to minimize adverse effects
during surgery involves instructing the patient to

1. discontinue using the medication 1 week prior to and 2 weeks after surgery to minimize risk
for infection.
2. discontinue using the medication 4 weeks prior to and 4 weeks after surgery to minimize risk
for infection.
3. discontinue using the medication 2 weeks prior to and 2 weeks after surgery to minimize risk
for component loosening.
4. continue using the medication during the perioperative period because it will not affect
the outcome.
5. increase the medication dosage to prevent stress-induced worsening of the patient’s
polyarticular symptoms.

PREFERRED RESPONSE: 1

RECOMMENDED READINGS
den Broeder AA, Creemers MC, Fransen J, de Jong E, de Rooij DJ, Wymenga A, de Waal-Malefijt M, van
den Hoogen FH. Risk factors for surgical site infections and other complications in elective surgery in
patients with rheumatoid arthritis with special attention for anti-tumor necrosis factor: a large retrospective
study. J Rheumatol. 2007 Apr;34(4):689-95. PubMed PMID: 17117492.

Goodman SM, Figgie M. Lower extremity arthroplasty in patients with inflammatory arthritis:
preoperative and perioperative management. J Am Acad Orthop Surg. 2013 Jun;21(6):355-63. doi:
10.5435/JAAOS-21-06-355. Review. PubMed PMID: 23728960.

Question 121
At which time is it safe for patients to return to driving after undergoing right total hip arthroplasty
(THA)?

1. 2 weeks
2. 4 weeks
3. 6 weeks
4. 8 weeks
5. 10 weeks

PREFERRED RESPONSE: 1

© 2016 American Academy of Orthopaedic Surgeons 2016 Orthopaedic In-Training Examination


SECTION 4: Hip and Knee Reconstruction • 91

RECOMMENDED READINGS
Hernandez VH, Ong A, Orozco F, Madden AM, Post Z. When is it safe for patients to drive after right
total hip arthroplasty? J Arthroplasty. 2015 Apr;30(4):627-30. doi: 10.1016/j.arth.2014.11.015. Epub 2014
Nov 26. PubMed PMID: 25499170.

Marecek GS, Schafer MF. Driving after orthopaedic surgery. J Am Acad Orthop Surg. 2013
Nov;21(11):696-706. doi: 10.5435/JAAOS-21-11-696. Review. PubMed PMID: 24187039.

Figure 127a Video 127b (scan to view)

Question 127
Figure 127a is the radiograph of a patient who is being seen for revision total hip arthroplasty for
infection. Video 127b is the CT scan sequence obtained following removal of the components. Using the
Paprosky classification for acetabular bone deficiency, how should this deficiency be classified?

1. IIA
2. IIB
3. IIC
4. IIIA
5. IIIB

PREFERRED RESPONSE: 5

RECOMMENDED READINGS
Paprosky WG, Perona PG, Lawrence JM. Acetabular defect classification and surgical reconstruction in
revision arthroplasty. A 6-year follow-up evaluation. J Arthroplasty. 1994 Feb;9(1):33-44. PubMed PMID:
8163974.

Dahl B, McNeely C, Robinson BS, Maloney WJ, Paprosky WG, Ries MD, Saleh KJ. Acetabular
reconstruction in revision total hip arthroplasty: maximizing function and outcomes in protrusio and
cavitary defects. Instr Course Lect. 2014;63:219-25. PubMed PMID: 24720308.

© 2016 American Academy of Orthopaedic Surgeons 2016 Orthopaedic In-Training Examination


92 • American Academy of Orthopaedic Surgeons

Question 136
A 76-year-old man underwent right total hip arthroplasty. He fell from a step ladder and is unable to bear
weight on his right side. A radiographic evaluation reveals a Vancouver B2 periprosthetic femur fracture.
What is the best next step?

1. Place a hip abduction brace and allow partial weight bearing with reevaluation in 2 weeks
2. Proceed with surgical treatment using cortical strut graft and plate and cerclage wire fixation
of the fracture
3. Revise the femoral component with a long-stem device and perform fracture stabilization
using a plate and cerclage wire construct
4. Revise the femoral component with a proximal femoral replacement construct
5. Perform a staged procedure with fracture fixation followed by femoral revision once the
fracture has healed

PREFERRED RESPONSE: 3

RECOMMENDED READINGS
Shah RP, Sheth NP, Gray C, Alosh H, Garino JP. Periprosthetic fractures around loose femoral
components. J Am Acad Orthop Surg. 2014 Aug;22(8):482-90. doi: 10.5435/JAAOS-22-08-482. Review.
PubMed PMID: 25063746.

Munro JT, Garbuz DS, Masri BA, Duncan CP. Tapered fluted titanium stems in the management of
Vancouver B2 and B3 periprosthetic femoral fractures. Clin Orthop Relat Res. 2014 Feb;472(2):590-8.
doi: 10.1007/s11999-013-3087-3. PubMed PMID: 23719963.

Question 139
A 67-year-old woman has experienced recurrent posterior hip dislocations that began 2 years after
her index procedure was performed with the anterior approach. Her initial postsurgical course was
uncomplicated and she progressed quickly to normal activities. She has been treated with bracing and
physical therapy that included a comprehensive review of dislocation precautions after each dislocation
occurred. Her C-reactive protein level and erythrocyte sedimentation rate are within defined limits.
Radiographs reveal well-fixed femoral and acetabular components; the cup position abduction angle is 60
degrees with approximately 5 degrees of anteversion. What is the best next step?

1. Revision of the acetabular component


2. Revision arthroplasty of the femoral component
3. Revision to a larger femoral head and liner
4. Constrained liner use
5. Trochanteric advancement

PREFERRED RESPONSE: 1

© 2016 American Academy of Orthopaedic Surgeons 2016 Orthopaedic In-Training Examination


SECTION 4: Hip and Knee Reconstruction • 93

RECOMMENDED READINGS
Lewinnek GE, Lewis JL, Tarr R, Compere CL, Zimmerman JR. Dislocations after total hip-replacement
arthroplasties. J Bone Joint Surg Am. 1978 Mar;60(2):217-20. PubMed PMID: 641088.

Soong M, Rubash HE, Macaulay W. Dislocation after total hip arthroplasty. J Am Acad Orthop Surg. 2004
Sep-Oct;12(5):314-21. Review. PubMed PMID: 15469226.

Figure 144a Figure 144

Question 144
Figures 144a and 144b are the radiographs of a 77-year-old patient who fell on the right hip, which
resulted in a periprosthetic fracture. Which construct design can best fix the fracture?

1. Cables alone
2. Plate with cables
3. Plates with cables and screws
4. Stem revision with fracture fixation with a screw-and-cable construct
5. Stem revision with fracture fixation using strut graft(s) and a cerclage cable construct

PREFERRED RESPONSE: 4

© 2016 American Academy of Orthopaedic Surgeons 2016 Orthopaedic In-Training Examination


94 • American Academy of Orthopaedic Surgeons

RECOMMENDED READINGS
Moazen M, Mak JH, Etchels LW, Jin Z, Wilcox RK, Jones AC, Tsiridis E. Periprosthetic femoral fracture-
-a biomechanical comparison between Vancouver type B1 and B2 fixation methods. J Arthroplasty. 2014
Mar;29(3):495-500. doi: 10.1016/j.arth.2013.08.010. Epub 2013 Sep 12. PubMed PMID: 24035619.

Shah S, Kim SY, Dubov A, Schemitsch EH, Bougherara H, Zdero R. The biomechanics of plate fixation of
periprosthetic femoral fractures near the tip of a total hip implant: cables, screws, or both? Proc Inst Mech
Eng H. 2011 Sep;225(9):845-56. PubMed PMID: 22070022.

Question 154
According to the Paprosky classification for femoral deficiency following total hip arthroplasty, how much
femoral diaphyseal bone must be present to obtain scratch-fit fixation with a fully porous coated stem for
type IIIA deficiencies?

1. 2 cm
2. 4 cm
3. 6 cm
4. 8 cm
5. 10 cm

PREFERRED RESPONSE: 2

RECOMMENDED READINGS
Abdel MP, Lewallen DG, Berry DJ. Periprosthetic femur fractures treated with modular fluted, tapered
stems. Clin Orthop Relat Res. 2014 Feb;472(2):599-603. doi: 10.1007/s11999-013-2936-4. PubMed
PMID: 23529634.

Weeden S, Della Valle C. Revision total hip arthroplasty. In: Glassman A, Lachiewicz PF, Tanzer M, eds.
Orthopaedic Knowledge Update: Hip and Knee Reconstruction. 4th ed. Rosemont, IL: American Academy
of Orthopaedic Surgeons; 2011:333-338.

© 2016 American Academy of Orthopaedic Surgeons 2016 Orthopaedic In-Training Examination


SECTION 4: Hip and Knee Reconstruction • 95

Question 165
A 68-year-old patient underwent a direct lateral total hip arthroplasty (THA) and now has a Trendelenburg
gait. Which nerve most likely is dysfunctional?

1. Inferior gluteal
2. Superior gluteal
3. Sciatic
4. Pudendal
5. Femoral

PREFERRED RESPONSE: 2

RECOMMENDED READINGS
Khan T, Knowles D. Damage to the superior gluteal nerve during the direct lateral approach to the hip: a
cadaveric study. J Arthroplasty. 2007 Dec;22(8):1198-200. PubMed PMID: 18078891.

Ramesh M, O'Byrne JM, McCarthy N, Jarvis A, Mahalingham K, Cashman WF. Damage to the superior
gluteal nerve after the Hardinge approach to the hip. J Bone Joint Surg Br. 1996 Nov;78(6):903-6.
PubMed PMID: 8951004.

© 2016 American Academy of Orthopaedic Surgeons 2016 Orthopaedic In-Training Examination


96 • American Academy of Orthopaedic Surgeons

Figure 177

Question 177
Figure 177 is the radiograph of a 53-year-old woman who underwent left total hip arthroplasty 3 years ago
and has had no issues with her hip since the surgery. Based on these radiographic findings, what is the best
next step?

1. Observation for 1 year


2. Bone scan
3. Hip aspiration
4. Assess serum metal ion levels
5. Assess erythrocyte sedimentation rate (ESR) and C-reactive protein (CRP) levels

PREFERRED RESPONSE: 5

RECOMMENDED READINGS
Yi PH, Cross MB, Moric M, Sporer SM, Berger RA, Della Valle CJ. The 2013 Frank Stinchfield Award:
Diagnosis of infection in the early postoperative period after total hip arthroplasty. Clin Orthop Relat Res.
2014 Feb;472(2):424-9. doi: 10.1007/s11999-013-3089-1. PubMed PMID: 23884798.

Tetreault MW, Wetters NG, Moric M, Gross CE, Della Valle CJ. Is synovial C-reactive protein a useful
marker for periprosthetic joint infection? Clin Orthop Relat Res. 2014 Dec;472(12):3997-4003. doi:
10.1007/s11999-014-3828-y. Epub 2014 Jul 29. PubMed PMID: 25070920.

© 2016 American Academy of Orthopaedic Surgeons 2016 Orthopaedic In-Training Examination


SECTION 4: Hip and Knee Reconstruction • 97

Figure 191

Question 191
Which artery provides the main blood supply to the structure marked by the X in Figure 191 during a left
knee arthroscopy?

1. Descending geniculate artery


2. Middle geniculate artery
3. Anterior tibial recurrent artery
4. Superior geniculate artery
5. Inferior geniculate artery

PREFERRED RESPONSE: 2

RECOMMENDED READINGS
Petersen W, Tillmann B. [Anatomy and function of the anterior cruciate ligament]. Orthopade. 2002
Aug;31(8):710-8. German. PubMed PMID: 12426749.

Warren R, Arnoczky S, Wickiewicz T. Anatomy of the knee. In: Nicholas J, Hershman E, eds. The Lower
Extremity and Spine in Sports Medicine. St Louis, MO: CV Mosby; 1986.

© 2016 American Academy of Orthopaedic Surgeons 2016 Orthopaedic In-Training Examination


98 • American Academy of Orthopaedic Surgeons

Figure 197a Figure 197b

Question 197
Placing the acetabular cup and femoral stem in the templated position shown in Figures 197a and 197b
increases

1. joint reactive force.


2. force requirement for abductors.
3. Increased lever arm for body weight.
4. Increased lever arm for abductors.
5. Increased polyethylene wear.

PREFERRED RESPONSE: 4

RECOMMENDED READINGS
Merle C, Waldstein W, Pegg E, Streit MR, Gotterbarm T, Aldinger PR, Murray DW, Gill HS.
Femoral offset is underestimated on anteroposterior radiographs of the pelvis but accurately assessed
on anteroposterior radiographs of the hip. J Bone Joint Surg Br. 2012 Apr;94(4):477-82. doi:
10.1302/0301-620X.94B4.28067. PubMed PMID: 22434462.

Charles MN, Bourne RB, Davey JR, Greenwald AS, Morrey BF, Rorabeck CH. Soft-tissue balancing of
the hip: the role of femoral offset restoration. Instr Course Lect. 2005;54:131-41. Review. PubMed PMID:
15948440.

© 2016 American Academy of Orthopaedic Surgeons 2016 Orthopaedic In-Training Examination


SECTION 4: Hip and Knee Reconstruction • 99

Figure 206

Question 206
A 64-year-old man with rheumatoid arthritis underwent an uncomplicated total knee arthroplasty (TKA).
He subsequently fell and underwent repair of his patellar tendon 2 weeks after the initial surgery. He now
has left knee pain, instability with ambulation, and an inability to straighten his knee. Figure 206 is the
radiograph taken 9 months after his most recent surgery. What is the best next step?

1. Long-leg cast for 6 weeks


2. Knee arthrodesis and casting
3. Quadriceps V-Y advancement and patellar tendon repair
4. Patellar tendon reconstruction with a hamstring autograft
5. Reconstruction of the extensor mechanism with an allograft

PREFERRED RESPONSE: 5

RECOMMENDED READINGS
Crossett LS, Sinha RK, Sechriest VF, Rubash HE. Reconstruction of a ruptured patellar tendon
with achilles tendon allograft following total knee arthroplasty. J Bone Joint Surg Am. 2002
Aug;84-A(8):1354-61. PubMed PMID: 12177265.

Burnett RS, Berger RA, Della Valle CJ, Sporer SM, Jacobs JJ, Paprosky WG, Rosenberg AG. Extensor
mechanism allograft reconstruction after total knee arthroplasty. J Bone Joint Surg Am. 2005 Sep;87 Suppl
1(Pt 2):175-94. PubMed PMID: 16140793.

© 2016 American Academy of Orthopaedic Surgeons 2016 Orthopaedic In-Training Examination


100 • American Academy of Orthopaedic Surgeons

Question 214
A 52-year-old man underwent left total knee arthroplasty (TKA) 1 year ago and has reported persistent
pain since his surgery. His postsurgical course was notable for persistent wound drainage for more than
14 days after the surgery; the drainage eventually resolved with oral antibiotics. He has not experienced
a recurrence, but notes that his knee feels warm, especially at night. He denies fevers, chills, or sweats.
An examination reveals his range of motion in extension is 10 degrees through 95 degrees of flexion. No
instability is noted, and the skin is without erythema. A moderate-size effusion is present. Radiographs
reveal circumferential radiolucent lines adjacent to each of the components. Further evaluation should
include

1. metal artifact reduction sequence MRI.


2. a CT scan.
3. a bone scan.
4. laboratory studies (complete blood count, erythrocyte sedimentation rate [ESR], and
C-reactive protein [CRP]).
5. allergy skin testing.

PREFERRED RESPONSE: 4

RECOMMENDED READINGS
Garvin KL, Konigsberg BS. Infection following total knee arthroplasty: prevention and management. J
Bone Joint Surg Am. 2011 Jun 15;93(12):1167-75. Review. PubMed PMID: 21776555.

Gonzalez MH, Mekhail AO. The failed total knee arthroplasty: evaluation and etiology. J Am Acad Orthop
Surg. 2004 Nov-Dec;12(6):436-46. Review. PubMed PMID: 15615509.

Question 215
According to the American Academy of Orthopaedic Surgeons clinical practice guideline, Treatment
of Osteoarthritis of the Knee, Evidence-Based Guideline, 2nd Edition, the most highly recommended
intervention for the treatment of symptomatic knee osteoarthritis is

1. tramadol.
2. a valgus knee off-loader brace.
3. a lateral wedge insole.
4. a hyaluronic acid injection.
5. glucosamine and chondroitin.

PREFERRED RESPONSE: 1

© 2016 American Academy of Orthopaedic Surgeons 2016 Orthopaedic In-Training Examination


SECTION 4: Hip and Knee Reconstruction • 101

RECOMMENDED READINGS
Bennell KL, Bowles KA, Payne C, Cicuttini F, Williamson E, Forbes A, Hanna F, Davies-Tuck M, Harris
A, Hinman RS. Lateral wedge insoles for medial knee osteoarthritis: 12 month randomised controlled
trial. BMJ. 2011 May 18;342:d2912. doi: 10.1136/bmj.d2912. PubMed PMID: 21593096.

Brouwer RW, van Raaij TM, Verhaar JA, Coene LN, Bierma-Zeinstra SM. Brace treatment for
osteoarthritis of the knee: a prospective randomized multi-centre trial. Osteoarthritis Cartilage. 2006
Aug;14(8):777-83. Epub 2006 Mar 24. PubMed PMID: 16563810.

Beaulieu AD, Peloso PM, Haraoui B, Bensen W, Thomson G, Wade J, Quigley P, Eisenhoffer J, Harsanyi
Z, Darke AC. Once-daily, controlled-release tramadol and sustained-release diclofenac relieve chronic
pain due to osteoarthritis: a randomized controlled trial. Pain Res Manag. 2008 Mar-Apr;13(2):103-10.
PubMed PMID: 18443672.

Question 225
During total hip arthroplasty (THA), which anatomic structure is crucial when defining the acetabular
quadrants for safe screw placement within the acetabular component?

1. Anterior superior iliac spine


2. Anterior inferior iliac spine
3. Obturator foramen
4. Iliopectineal eminence
5. Greater sciatic notch

PREFERRED RESPONSE: 1

RECOMMENDED READINGS
Wasielewski RC, Cooperstein LA, Kruger MP, Rubash HE. Acetabular anatomy and the transacetabular
fixation of screws in total hip arthroplasty. J Bone Joint Surg Am. 1990 Apr;72(4):501-8. PubMed PMID:
2324135.

Lavernia CJ, Cook CC, Hernandez RA, Sierra RJ, Rossi MD. Neurovascular injuries in acetabular
reconstruction cage surgery: an anatomical study. J Arthroplasty. 2007 Jan;22(1):124-32. PubMed PMID:
17197319.

© 2016 American Academy of Orthopaedic Surgeons 2016 Orthopaedic In-Training Examination


102 • American Academy of Orthopaedic Surgeons

Question 238
In patients with Paget disease, administration of which drug can help to minimize perioperative blood loss
for patients undergoing elective joint arthroplasty?

1. Pamidronate
2. Methylprednisolone
3. Glucosamine chondroitin
4. Aminocaproic acid
5. Fibrin sealer

PREFERRED RESPONSE: 1

RECOMMENDED READINGS
Lewallen DG. Hip arthroplasty in patients with Paget’s disease. Clin Orthop Relat Res. 1999
Dec;(369):243-50. Review. PubMed PMID: 10611879.

Wegrzyn J, Pibarot V, Chapurlat R, Carret JP, Béjui-Hugues J, Guyen O. Cementless total hip arthroplasty
in Paget's disease of bone: a retrospective review. Int Orthop. 2010 Dec;34(8):1103-9. doi: 10.1007/
s00264-009-0853-7. Epub 2009 Aug 11. PubMed PMID: 19669762.

Question 242
A 72-year-old woman underwent total hip arthroplasty (THA) with a metal-on-metal construct 4 years ago
and is now experiencing the insidious onset of groin pain on the affected hip and pain with hip flexion.
Radiographs reveal a 40-degree abduction angle of the acetabular component with appropriate anteversion
and a well-positioned stem with minimal lucency adjacent to the proximal portion of the stem in Gruen
zones 1 and 7. Further evaluation should include

1. CT scan of the hip and pelvis.


2. metal artifact reduction sequence (MARS) MRI.
3. sterile aspiration of the hip for culture and sensitivity.
4. technetium-99 bone scan imaging.
5. indium-111 imaging.

PREFERRED RESPONSE: 2

RECOMMENDED READINGS
Kwon YM, Lombardi AV, Jacobs JJ, Fehring TK, Lewis CG, Cabanela ME. Risk stratification algorithm
for management of patients with metal-on-metal hip arthroplasty: consensus statement of the American
Association of Hip and Knee Surgeons, the American Academy of Orthopaedic Surgeons, and the Hip
Society. J Bone Joint Surg Am. 2014 Jan 1;96(1):e4. doi: 10.2106/JBJS.M.00160. PubMed PMID:
24382732.

Jacobs JJ, Urban RM, Hallab NJ, Skipor AK, Fischer A, Wimmer MA. Metal-on-metal bearing surfaces. J
Am Acad Orthop Surg. 2009 Feb;17(2):69-76. PubMed PMID: 19202120.

© 2016 American Academy of Orthopaedic Surgeons 2016 Orthopaedic In-Training Examination
SECTION 4: Hip and Knee Reconstruction • 103

Figure 243

Question 243
Figure 243 is the postsurgical radiograph of a 65-year-old woman who underwent an uncomplicated right
total hip arthroplasty through a standard posterior approach. At the 2-week follow-up visit, she is having
difficulty bearing weight. What is the best next step?

1. Acetabular component revision


2. Toe-touch weight bearing for 6 weeks
3. Open reduction and internal fixation (ORIF) of the anterior column
4. ORIF of the posterior column with acetabular component revision
5. ORIF of both columns with acetabular component revision

PREFERRED RESPONSE: 4

RECOMMENDED READINGS
Laflamme GY, Belzile EL, Fernandes JC, Vendittoli PA, Hébert-Davies J. Periprosthetic fractures of the
acetabulum during cup insertion: posterior column stability is crucial. J Arthroplasty. 2015 Feb;30(2):265-
9. doi: 10.1016/j.arth.2014.09.013. Epub 2014 Sep 28. PubMed PMID: 25307882.

Peterson CA, Lewallen DG. Periprosthetic fracture of the acetabulum after total hip arthroplasty. J Bone
Joint Surg Am. 1996 Aug;78(8):1206-13. PubMed PMID: 8753713.

© 2016 American Academy of Orthopaedic Surgeons 2016 Orthopaedic In-Training Examination


104 • American Academy of Orthopaedic Surgeons

Question 248
A 53-year-old man underwent a successful total hip arthroplasty revision 2 years ago. Recently he started
to experience recurrent dislocations after a traumatic fall. He underwent revision surgery for instability.
Despite revising femoral and acetabular components and adequately restoring limb length, intraoperative
instability persists. What is the best surgical option?

1. Medialization of the acetabular cup


2. Femoral component revision
3. Increase in neck length
4. Trochanteric advancement
5. Use of an offset acetabular liner

PREFERRED RESPONSE: 4

RECOMMENDED READINGS
Soong M, Rubash HE, Macaulay W. Dislocation after total hip arthroplasty. J AmAcad Orthop Surg. 2004
Sep-Oct;12(5):314-21. Review. PubMed PMID: 15469226.

Kaplan SJ, Thomas WH, Poss R. Trochanteric advancement for recurrent dislocation after total hip
arthroplasty. J Arthroplasty. 1987;2(2):119-24. PubMed PMID: 3612137.

© 2016 American Academy of Orthopaedic Surgeons 2016 Orthopaedic In-Training Examination


SECTION 4: Hip and Knee Reconstruction • 105

Video 258 (scan to view)

Question 258
The patient seen in Video 258 is undergoing a direct anterior approach to total hip arthroplasty. During this
approach, blood vessels are encountered. Which artery is identified?

1. Ascending branch of the highest genicular


2. Ascending branch of the lateral femoral circumflex
3. Ascending branch of the medial femoral circumflex
4. Descending branch of the lateral femoral circumflex
5. Transverse branch of the lateral femoral circumflex

PREFERRED RESPONSE: 2

RECOMMENDED READINGS
Kalhor M, Beck M, Huff TW, Ganz R. Capsular and pericapsular contributions to acetabular and femoral
head perfusion. J Bone Joint Surg Am. 2009 Feb;91(2):409-18. doi: 10.2106/JBJS.G.01679. PubMed
PMID: 19181985.

Chen LH, Huang QW, Wang WJ, He ZR, Ding WL. The applied anatomy of anterior approach for
minimally invasive hip joint surgery. Clin Anat. 2009 Mar;22(2):250-5. doi: 10.1002/ca.20750. PubMed
PMID: 19089989.

© 2016 American Academy of Orthopaedic Surgeons 2016 Orthopaedic In-Training Examination


106 • American Academy of Orthopaedic Surgeons

SECTION 5: Oncology

Figure 4a Figure 4b

Question 4
Figures 4a and 4b are the radiograph and biopsy specimen of a 62-year-old man who notes progressive left
shoulder pain without antecedent trauma. He underwent arthroscopic debridement 10 years ago. What is
the most likely diagnosis?

1. Lymphoma
2. Osteomyelitis
3. Ewing sarcoma
4. Multiple myeloma
5. Metastatic carcinoma

PREFERRED RESPONSE: 4

RECOMMENDED READINGS
Colson K. Treatment-related symptom management in patients with multiple myeloma: a review. Support
Care Cancer. 2015 May;23(5):1431-45. doi: 10.1007/s00520-014-2552-1. Epub 2015 Feb 3. Review.
PubMed PMID: 25646616.

Röllig C, Knop S, Bornhäuser M. Multiple myeloma. Lancet. 2015 May 30;385(9983):2197-208. doi:
10.1016/S0140-6736(14)60493-1. Epub 2014 Dec 23. Review. PubMed PMID: 25540889.

© 2016 American Academy of Orthopaedic Surgeons 2016 Orthopaedic In-Training Examination


SECTION 5: Oncology • 107

Figure 17a Figure 17b

Question 17
Figures 17a and 17b are the MR image and biopsy specimen of a 67-year-old woman who has a slowly
enlarging buttock mass. She is otherwise healthy. She first noticed this change a couple of years ago and
now is reporting mild pain over the mass. What is the most likely diagnosis?

1. Well-differentiated liposarcoma
2. Intramuscular myxoma
3. Myxoid liposarcoma
4. Round-cell liposarcoma
5. Lipoma

PREFERRED RESPONSE: 2

RECOMMENDED READINGS
Prado MA, Miró RL, Leal IM, Vargas J, Dorrego EJ. Intramuscular myxoma: differential diagnosis and
review of the literature. Orthopedics. 2002 Nov;25(11):1297-9. Review. PubMed PMID: 12452352.

Petscavage-Thomas JM, Walker EA, Logie CI, Clarke LE, Duryea DM, Murphey MD. Soft-tissue
myxomatous lesions: review of salient imaging features with pathologic comparison. Radiographics. 2014
Jul-Aug;34(4):964-80. doi: 10.1148/rg.344130110. PubMed PMID: 25019435.

© 2016 American Academy of Orthopaedic Surgeons 2016 Orthopaedic In-Training Examination


108 • American Academy of Orthopaedic Surgeons

Figure 25a Figure 25b

Question 25
Figures 25a and 25b are the hip radiographs of an independent 57-year-old man who has had moderate
hip pain for the past 2 months. He walks unaided with an antalgic gait and has a history of lung cancer. A
biopsy of the lesion reveals metastatic lung adenocarcinoma. He has no other active areas of metastatic
disease. What is the best next step?

1. Protected weight bearing and an oncology referral


2. Placement of a cephalomedullary nail
3. Proximal femoral replacement with a cemented long-stem prosthesis
4. External beam irradiation to the affected bone.
5. Open curettage, cementation, and placement of a proximal femoral locking plate

PREFERRED RESPONSE: 2

RECOMMENDED READINGS
Mirels H. Metastatic disease in long bones: A proposed scoring system for diagnosing impending
pathologic fractures. 1989. Clin Orthop Relat Res. 2003 Oct;(415 Suppl):S4-13. PubMed PMID:
14600587.

Quinn RH, Randall RL, Benevenia J, Berven SH, Raskin KA. Contemporary management of metastatic
bone disease: tips and tools of the trade for general practitioners. Instr Course Lect. 2014;63:431-41.
PubMed PMID: 24720328.

Scolaro JA, Lackman RD. Surgical management of metastatic long bone fractures: principles and
techniques. J Am Acad Orthop Surg. 2014 Feb;22(2):90-100. doi: 10.5435/JAAOS-22-02-90. Review.
PubMed PMID: 24486755.

© 2016 American Academy of Orthopaedic Surgeons 2016 Orthopaedic In-Training Examination


SECTION 5: Oncology • 109

Figure 37a Figure 37b Figure 37c

Figure 37d

Question 37
Figures 37a through 37d are the radiographs, MR image, and biopsy specimen of a 60-year-old man who
had progressive shoulder pain for 3 months. He has pain with activity and at rest. An examination reveals
midhumeral tenderness. A workup shows no other areas of active disease. What is the best next step?

1. Observation
2. Internal fixation
3. Radiation therapy
4. Corticosteroid injection
5. Radiofrequency ablation

PREFERRED RESPONSE: 2

© 2016 American Academy of Orthopaedic Surgeons 2016 Orthopaedic In-Training Examination


110 • American Academy of Orthopaedic Surgeons

RECOMMENDED READINGS
Evans AR, Bottros J, Grant W, Chen BY, Damron TA. Mirels' rating for humerus lesions is both
reproducible and valid. Clin Orthop Relat Res. 2008 Jun;466(6):1279-84. doi: 10.1007/s11999-008-0200-
0. Epub 2008 Mar 21. PubMed PMID: 18357496.

Peabody T. The rodded metastasis is a sarcoma: strategies to prevent inadvertent surgical procedures on
primary bone malignancies. Instr Course Lect. 2004;53:657-61. Review. PubMed PMID: 15116656.

Question 47
A 37-year-old man is experiencing the atraumatic onset of a painless soft-tissue mass in his buttock. After
discussion, the patient opts for an incisional biopsy. How should this biopsy be performed?

1. Transverse incision in an extensile line with drain placement


2. Transverse incision in an extensile line, frozen section, and drain placement after the
diagnosis confirmation
3. Transverse incision in an internervous plane
4. Longitudinal incision through an intramuscular plane
5. Longitudinal incision through an internervous plane

PREFERRED RESPONSE: 4

RECOMMENDED READINGS
Mankin HJ, Springfield DS, Gebhardt MC. Principles of surgical treatment of soft tissue sarcomas. Cancer
Treat Res. 1991;56:37-52. Review. PubMed PMID: 1681878.

Schwab JH, Springfield DS, Raskin KA, Mankin HJ, Hornicek FJ. What's new in primary malignant
musculoskeletal tumors. J Bone Joint Surg Am. 2013 Dec 18;95(24):2240-6. doi: 10.2106/JBJS.M.01226.
Review. PubMed PMID: 24352776.

© 2016 American Academy of Orthopaedic Surgeons 2016 Orthopaedic In-Training Examination


SECTION 5: Oncology • 111

Figure 60a Figure 60b

Figure 60c Figure 60c

Question 60
Figures 60a through 60d are the radiographs, coronal MR image, and bone scan of a 63-year-old
woman who noted the progressive onset of right shoulder pain. She has had physical therapy and has
taken nonsteroidal anti-inflammatory drugs, experiencing minimal relief. The pain began insidiously, is
associated with overhead activities, and is mild. What is the best next step?

1. Biopsy
2. Curettage
3. Observation
4. CT scan of the chest
5. Subacromial steroid injection

PREFERRED RESPONSE: 5

© 2016 American Academy of Orthopaedic Surgeons 2016 Orthopaedic In-Training Examination


112 • American Academy of Orthopaedic Surgeons

RECOMMENDED READINGS
Douis H, Saifuddin A. The imaging of cartilaginous bone tumours. I. Benign lesions. Skeletal Radiol.
2012 Sep;41(10):1195-212. doi: 10.1007/s00256-012-1427-0. Epub 2012 Jun 17. Review. PubMed PMID:
22707094.

Levy JC, Temple HT, Mollabashy A, Sanders J, Kransdorf M. The causes of pain in benign solitary
enchondromas of the proximal humerus. Clin Orthop Relat Res. 2005 Feb;(431):181-6. PubMed PMID:
15685073.

Question 71
Osteoid osteoma can most effectively be differentiated from a stress fracture with which diagnostic
modality?

1. MR image
2. Bone scan
3. Biopsy only
4. Plain radiograph
5. High-resolution CT scan

PREFERRED RESPONSE: 5

RECOMMENDED READINGS
Henshaw RM, Carmody Soni EE. Benign bone-forming tumors. In: Biermann JS, ed. Orthopaedic
Knowledge Update 3: Musculoskeletal Tumors. Rosemont, IL: American Academy of Orthopaedic
Surgeons; 2014:107-121.

Bilchik T, Heyman S, Siegel A, Alavi A. Osteoid osteoma: the role of radionuclide bone imaging,
conventional radiography and computed tomography in its management. J Nucl Med. 1992 Feb;33(2):269-
71. PubMed PMID: 1732454.

© 2016 American Academy of Orthopaedic Surgeons 2016 Orthopaedic In-Training Examination


SECTION 5: Oncology • 113

Question 79
A 24-year-old man underwent a transhumeral amputation 2 years ago for a high-grade sarcoma that
encased the neurovascular bundle. He now has a focally tender mass consistent with a neuroma. Prosthetic
revisions have not relieved the discomfort. What is the most reasonable next step?

1. Targeted muscle reinnervation


2. Neuroma resection
3. Muscle transfer
4. Nerve transposition
5. Further humeral resection

PREFERRED RESPONSE: 1

RECOMMENDED READINGS
Souza JM, Cheesborough JE, Ko JH, Cho MS, Kuiken TA, Dumanian GA. Targeted muscle reinnervation:
a novel approach to postamputation neuroma pain. Clin Orthop Relat Res. 2014 Oct;472(10):2984-90. doi:
10.1007/s11999-014-3528-7. PubMed PMID: 24562875.

Gart MS, Souza JM, Dumanian GA. Targeted Muscle Reinnervation in the Upper Extremity Amputee: A
Technical Roadmap. J Hand Surg Am. 2015 Sep;40(9):1877-88. doi: 10.1016/j.jhsa.2015.06.119. Review.
PubMed PMID: 26314220.

Fitzgibbons P, Medvedev G. Functional and Clinical Outcomes of Upper Extremity Amputation. J Am


Acad Orthop Surg. 2015 Dec;23(12):751-60. doi: 10.5435/JAAOS-D-14-00302. Epub 2015 Nov 2.
Review. PubMed PMID: 26527583.

© 2016 American Academy of Orthopaedic Surgeons 2016 Orthopaedic In-Training Examination


114 • American Academy of Orthopaedic Surgeons

Figure 90

Question 90
A 24-year-old woman has a painless mass in her thigh that has been increasing in size. A biopsy is
planned. Which approach pictured in Figure 90 should be used to perform the biopsy?

1. A
2. B
3. C
4. D
5. E

PREFERRED RESPONSE: 3

RECOMMENDED READINGS
Rougraff BT, Aboulafia A, Biermann JS, Healey J. Biopsy of soft tissue masses: evidence-based medicine
for the musculoskeletal tumor society. Clin Orthop Relat Res. 2009 Nov;467(11):2783-91. doi: 10.1007/
s11999-009-0965-9. Epub 2009 Jul 14. PubMed PMID: 19597901.

Mankin HJ, Mankin CJ, Simon MA. The hazards of the biopsy, revisited. Members of the Musculoskeletal
Tumor Society. J Bone Joint Surg Am. 1996;78:656–663. PubMed PMID: 8642021.

Traina F, Errani C, Toscano A, Pungetti C, Fabbri D, Mazzotti A, Donati D, Faldini C. Current concepts
in the biopsy of musculoskeletal tumors. J Bone Joint Surg Am. 2015 Jan 7;97(1):e7. PubMed PMID:
25723000.

© 2016 American Academy of Orthopaedic Surgeons 2016 Orthopaedic In-Training Examination


SECTION 5: Oncology • 115

Figure 109a Figure 109b

Question 109
A 24-year-old woman with a history of a soft-tissue sarcoma of her left thigh is being monitored for
recurrence. An MR image performed 2 years after her resection is shown in Figure 109a. Bone scan,
positron emission tomography CT, and radiograph findings are all normal. A biopsy of the right femur is
shown in Figure 109b. Which chromosomal translocation is associated with this pathology?

1. X;18
2. 12;16
3. X;17
4. 11;22
5. 2;13

PREFERRED RESPONSE: 2

RECOMMENDED READINGS
Xiang H1, Wang J, Hisaoka M, Zhu X. Characteristic sequence motifs located at the genomic breakpoints
of the translocation t(12;16) and t(12;22) in myxoid liposarcoma. Pathology. 2008 Oct;40(6):547-52. doi:
10.1080/00313020802320424. PubMed PMID: 18752119.

Noble JL1, Moskovic E, Fisher C, Judson I. Imaging of skeletal metastases in myxoid liposarcoma.
Sarcoma. 2010;2010:262361. doi: 10.1155/2010/262361. Epub 2010 Mar 30. PubMed PMID: 20369068.

Schwab JH1, Boland PJ, Antonescu C, Bilsky MH, Healey JH. Spinal metastases from myxoid
liposarcoma warrant screening with magnetic resonance imaging. Cancer. 2007 Oct 15;110(8):1815-22.
PubMed PMID: 17724681.

© 2016 American Academy of Orthopaedic Surgeons 2016 Orthopaedic In-Training Examination


116 • American Academy of Orthopaedic Surgeons

Figure 119a Figure 119b

Figure 119c Figure 119d

Question 119
Figure 119a is the initial radiograph of a 12-year-old girl who plays competitive soccer. She has a 3-week
history of right thigh pain and a limp, and the radiograph was performed 3 weeks ago. Current radiographs
and MR images are shown in Figures 119b through 119d. She has remained afebrile; a white blood cell
count, erythrocyte sedimentation rate, and C-reactive protein level are within defined limits. She has pain
with resisted adduction of her hip, but otherwise her examination findings are benign. Initial treatment
should consist of

1. activity cessation and observation.


2. biopsy.
3. prophylactic fixation with an intramedullary nail.
4. empiric intravenous antibiotics.
5. irrigation and debridement.

PREFERRED RESPONSE: 1

© 2016 American Academy of Orthopaedic Surgeons 2016 Orthopaedic In-Training Examination


SECTION 5: Oncology • 117

RECOMMENDED READINGS
Nishio J, Hara M, Naito M. Adductor insertion avulsion syndrome mimicking neoplastic processes in
a 14-year-old long-distance runner. Orthopedics. 2012 Sep;35(9):e1442-5. doi: 10.3928/01477447-
20120822-37. PubMed PMID: 22955417.

Anderson MW, Kaplan PA, Dussault RG. Adductor insertion avulsion syndrome (thigh splints): spectrum
of MR imaging features. AJR Am J Roentgenol. 2001 Sep;177(3):673-5. PubMed PMID: 11517070.

Mahajan MS. Bone scanning in the adductor insertion avulsion syndrome. World J Nucl Med. 2013
May;12(2):73-5. doi: 10.4103/1450-1147.136698. PubMed PMID: 25126001.

© 2016 American Academy of Orthopaedic Surgeons 2016 Orthopaedic In-Training Examination


118 • American Academy of Orthopaedic Surgeons

Figure 151a Figure 151b Figure 151c

Question 151
Figure 151a is a radiograph of a 40-year-old man with a history of a giant-cell tumor of the femur that was
resected 7 years ago. He has had increasing pain and swelling that is worse with activity for 3 months.
He is afebrile, and his white blood cell count, erythrocyte sedimentation rate, and C-reactive protein level
are all within defined limits. A CT scan of the chest, abdomen, and pelvis demonstrates no evidence of
other disease. He previously had allograft bone placed into the defect. Low- and high-power pathologic
specimens are shown in Figures 151b and 151c. Further treatment should consist of

1. subcutaneous denosumab injections.


2. repeat curettage resection with a local adjuvant, placement of autograft bone, and
prophylactic fixation.
3. repeat curettage resection with a local adjuvant, placement of methylmethacrylate, and
prophylactic fixation.
4. irrigation and debridement, placement of an antibiotic spacer, and intravenous antibiotics.
5. wide resection with reconstruction with a distal femoral replacement.

PREFERRED RESPONSE: 5

RECOMMENDED READINGS
Li J, Zhu Y, Wei Y. Fibrosarcoma development 15 years after curettage and bone grafting of giant cell
tumor of bone. Orthopedics. 2014 May;37(5):e512-6. doi: 10.3928/01477447-20140430-66. PubMed
PMID: 24810832.

Saito T, Mitomi H, Izumi H, Suehara Y, Okubo T, Torigoe T, Takagi T, Kaneko K, Sato K, Matsumoto
T, Yao T. A case of secondary malignant giant-cell tumor of bone with p53 mutation after long-term
follow-up. Hum Pathol. 2011 May;42(5):727-33. doi: 10.1016/j.humpath.2010.08.008. Epub 2011 Jan 14.
PubMed PMID: 21237496.

© 2016 American Academy of Orthopaedic Surgeons 2016 Orthopaedic In-Training Examination


SECTION 5: Oncology • 119

Figure 156a Figure 156b Figure 156c

Question 156
Figures 156a through 156c are the radiograph, MR image, and biopsy specimen of a 47-year-old woman
who underwent total knee arthroplasty and continues to have left groin pain. What is the most appropriate
treatment?

1. Surgical resection and reconstruction


2. Radiation therapy and surgical resection
3. Curettage, adjuvant treatment, and internal fixation
4. Curettage and reconstruction
5. Presurgical chemotherapy, surgical resection, and radiation

PREFERRED RESPONSE: 1

RECOMMENDED READINGS
Mermerkaya MU, Bekmez S, Karaaslan F, Danisman M, Kosemehmetoglu K, Gedikoglu G, Ayvaz M,
Tokgozoglu AM. Intralesional curettage and cementation for low-grade chondrosarcoma of long bones:
retrospective study and literature review. World J Surg Oncol. 2014 Nov 10;12:336. doi: 10.1186/1477-
7819-12-336. Review. PubMed PMID: 25382793.

Angelini A, Calabrò T, Pala E, Trovarelli G, Maraldi M, Ruggieri P. Resection and reconstruction of pelvic
bone tumors. Orthopedics. 2015 Feb;38(2):87-93. doi: 10.3928/01477447-20150204-51. PubMed PMID:
25665107.

© 2016 American Academy of Orthopaedic Surgeons 2016 Orthopaedic In-Training Examination


120 • American Academy of Orthopaedic Surgeons

Figure 172a Figure 172b

Question 172
Figures 172a and 172b are the MR image and biopsy specimen of a 37-year-old runner who injured his
calf 2 weeks ago while training for a marathon. He describes a plantar flexion eversion position of his
ankle and now has pain and swelling on his calf. What is the most likely diagnosis?

1. Lipoma
2. Synovial sarcoma
3. Hematoma
4. Intramuscular arteriovenous malformation (AVM)
5. Medial gastrocnemius avulsion injury

PREFERRED RESPONSE: 2

RECOMMENDED READINGS
Deshmukh R, Mankin HJ, Singer S. Synovial sarcoma: the importance of size and location for survival.
Clin Orthop Relat Res. 2004 Feb;(419):155-61. PubMed PMID: 15021147.

Karakousis G. Update for ASCO 2015 sarcoma sessions. Surg Oncol. 2015 Dec;24(4):369-70. doi:
10.1016/j.suronc.2015.08.006. Epub 2015 Aug 14. PubMed PMID: 26298199.

© 2016 American Academy of Orthopaedic Surgeons 2016 Orthopaedic In-Training Examination


SECTION 5: Oncology • 121

Figure 187a Figure 187b

Figure 187c Figure 187d

Figure 187e

Question 187
A 58-year-old woman with a history of nonmetastatic breast cancer sustains a pathologic displaced
humeral fracture. An intraoperative biopsy and frozen section is completed; after a pathology review, you
determine that proceeding with intramedullary fixation of the fracture is appropriate. Which pathology
specimen (Figures 187a through 187e) helps to confirm that it is safe to proceed with intramedullary
fixation?

1. 187a
2. 187b
3. 187c
4. 187d
5. 187e

PREFERRED RESPONSE: 5

© 2016 American Academy of Orthopaedic Surgeons 2016 Orthopaedic In-Training Examination


122 • American Academy of Orthopaedic Surgeons

RECOMMENDED READINGS
Weber KL. Evaluation of the adult patient (aged >40 years) with a destructive bone lesion. J Am Acad
Orthop Surg. 2010 Mar;18(3):169-79. Review. PubMed PMID: 20190107.

Beebe KS: Prediction of impending fractures and treatment considerations in patients with metastatic bone
disease. In: Biermann JS, ed. Orthopaedic Knowledge Update Musculoskeletal Tumors. 3rd ed. Rosemont,
IL: American Academy of Orthopaedic Surgeons; 2014:331-338.

Adams SC, Potter BK, Mahmood Z, Pitcher JD, Temple HT. Consequences and prevention of inadvertent
internal fixation of primary osseous sarcomas. Clin Orthop Relat Res. 2009 Feb;467(2):519-25. doi:
10.1007/s11999-008-0546-3. Epub 2008 Oct 21. PubMed PMID: 18937020.

© 2016 American Academy of Orthopaedic Surgeons 2016 Orthopaedic In-Training Examination


SECTION 5: Oncology • 123

Figure 217a Figure 217b Figure 217c

Figure 217d Figure 217e

Question 217
Figures 217a through 217e are the plain radiographs, T1- and T2-weighted MR images, and biopsy
specimen of an 18-year-old woman who has noticed a painless posterior thigh mass for several months.
She believes the mass may have increased in size, but cannot describe any symptoms in the area. What is
the most likely diagnosis?

1. Desmoid
2. Lipoma
3. Hemangioma
4. Clear-cell sarcoma
5. Alveolar soft-part sarcoma

PREFERRED RESPONSE: 5

© 2016 American Academy of Orthopaedic Surgeons 2016 Orthopaedic In-Training Examination


124 • American Academy of Orthopaedic Surgeons

RECOMMENDED READINGS
Zarrin-Khameh N, Kaye KS. Alveolar soft part sarcoma. Arch Patho-l Lab Med. 2007 Mar;131(3):488-91.
Review. PubMed PMID: 17516754.

Stacy GS, Nair L. Magnetic resonance imaging features of extremity sarcomas of uncertain differentiation.
Clin Radiol. 2007 Oct;62(10):950-8. Epub 2007 May 15. Review. PubMed PMID: 17765459.

© 2016 American Academy of Orthopaedic Surgeons 2016 Orthopaedic In-Training Examination


SECTION 5: Oncology • 125

Figure 241a Figure 241b

Figure 241c Figure 241d Figure 241e

Figure 241f Figure 241g

Question 241
Figures 241a and 241b are arthroscopic views of a 36-year-old woman who undergoes a knee arthroscopy
for pain and recurrent effusions. Which figure depicts the pathology specimen that correlates with the
surgical findings?

1. 241c
2. 241d
3. 241e
4. 241f
5. 241g

PREFERRED RESPONSE: 2

© 2016 American Academy of Orthopaedic Surgeons 2016 Orthopaedic In-Training Examination


126 • American Academy of Orthopaedic Surgeons

RECOMMENDED READINGS
Aurégan JC, Bohu Y, Lefevre N, Klouche S, Naouri JF, Herman S, Hardy P. Primary arthroscopic
synovectomy for pigmented villo-nodular synovitis of the knee: recurrence rate and functional outcomes
after a mean follow-up of seven years. Orthop Traumatol Surg Res. 2013 Dec;99(8):937-43. doi:
10.1016/j.otsr.2013.08.004. Epub 2013 Oct 23. PubMed PMID: 24161841.

Murphey MD, Rhee JH, Lewis RB, Fanburg-Smith JC, Flemming DJ, Walker EA. Pigmented villonodular
synovitis: radiologic-pathologic correlation. Radiographics. 2008 Sep-Oct;28(5):1493-518. doi: 10.1148/
rg.285085134. PubMed PMID: 18794322.

Question 254
A 57-year-old patient who smokes cigarettes has a dual-energy x-ray absorptiometry hip scan. Her T-score
is -1.2, and her 10-year hip fracture probability is 3%. According to World Health Organization (WHO)
Guidelines, what is the best next step?

1. Recommend a minimum of 1000-mg daily calcium and vitamin D dosing and repeat the scan
in 1 year
2. Recommend a minimum of 1200-mg daily calcium and repeat the scan in 2 years
3. Recommend a minimum of 1200-mg daily calcium and repeat the scan in 5 years
4. Initiate pharmacologic treatment with a repeat scan in 1 to 2 years.
5. Initiate a smoking cessation program, reinforce vitamin supplementation of 1200 mg of
calcium daily, and repeat the scan in 2 years

PREFERRED RESPONSE: 4

RECOMMENDED READINGS
Favus MJ. Bisphosphonates for osteoporosis. N Engl J Med. 2010 Nov 18;363(21):2027-35. doi: 10.1056/
NEJMct1004903. Review. PubMed PMID: 21083387.

Choma TJ, Rechtine GR, McGuire RA Jr, Brodke DS. Treating the Aging Spine. J Am Acad Orthop
Surg. 2015 Dec;23(12):e91-e100. doi: 10.5435/JAAOS-D-15-00245. Epub 2015 Oct 28. PubMed PMID:
26510625.

© 2016 American Academy of Orthopaedic Surgeons 2016 Orthopaedic In-Training Examination


SECTION 5: Oncology • 127

Figure 257a Figure 257b

Question 257
Figure 257a is an MR image of a 73-year-old woman who has hip pain and no known history of cancer.
Staging studies demonstrate a mass in her kidney with no other lesions noted. A biopsy specimen is shown
in Figure 257b. In regard to her femur, which intervention will provide the best long-term outcome and
lowest risk for revision surgery?

1. Presurgical radiation
2. Presurgical embolization
3. Intralesional resection of the tumor with placement of methylmethacrylate and a long
cephalomedullary nail
4. Intralesional resection of the tumor with placement of methylmethacrylate and a dynamic
hip screw
5. Resection of the tumor and placement of an endoprosthesis

PREFERRED RESPONSE: 5

RECOMMENDED READINGS
Laitinen M, Parry M, Ratasvuori M, Wedin R, Albergo JI, Jeys L, Abudu A, Carter S, Gaston L, Tillman
R, Grimer R. Survival and complications of skeletal reconstructions after surgical treatment of bony
metastatic renal cell carcinoma. Eur J Surg Oncol. 2015 Jul;41(7):886-92. doi: 10.1016/j.ejso.2015.04.008.
Epub 2015 Apr 29. PubMed PMID: 25964050.

Hwang N, Nandra R, Grimer RJ, Carter SR, Tillman RM, Abudu A, Jeys LM. Massive endoprosthetic
replacement for bone metastases resulting from renal cell carcinoma: factors influencing patient survival.
Eur J Surg Oncol. 2014 Apr;40(4):429-34. doi: 10.1016/j.ejso.2013.08.001. Epub 2013 Sep 21. PubMed
PMID: 24063967.

Miller BJ, Soni EE, Gibbs CP, Scarborough MT. Intramedullary nails for long bone metastases: why
do they fail? Orthopedics. 2011 Apr 11;34(4). doi: 10.3928/01477447-20110228-12. PubMed PMID:
21469628.

© 2016 American Academy of Orthopaedic Surgeons 2016 Orthopaedic In-Training Examination


128 • American Academy of Orthopaedic Surgeons

Figure 262

Question 262
Figure 262 is the radiograph of a 46-year-old healthy woman with a 3-week history of vague thigh pain
that began when she sustained an injury while stepping out of the shower. Laboratory study findings are
normal, and a positron emission tomography (PET)/CT body scan shows activity isolated only to the
fracture site. What is the best next step?

1. Closed reduction and internal fixation with a locked intramedullary rod


2. Above-knee amputation
3. Open biopsy and splint application
4. Open reduction and internal fixation with a locked intramedullary rod
5. Placement of skeletal traction and an MRI

PREFERRED RESPONSE: 3

RECOMMENDED READINGS
Quinn RH, Randall RL, Benevenia J, Berven SH, Raskin KA. Contemporary management of metastatic
bone disease: tips and tools of the trade for general practitioners. Instr Course Lect. 2014;63:431-41.
PubMed PMID: 24720328.

Scolaro JA, Lackman RD. Surgical management of metastatic long bone fractures: principles and
techniques. J Am Acad Orthop Surg. 2014 Feb;22(2):90-100. doi: 10.5435/JAAOS-22-02-90. Review.
PubMed PMID: 24486755.

© 2016 American Academy of Orthopaedic Surgeons 2016 Orthopaedic In-Training Examination


SECTION 5: Oncology • 129

Figure 270a Figure 270b

Figure 270c

Figure 270d Figure 270e

Question 270
Figures 270a through 270e are the radiograph, CT scans, and low- and high-power biopsy specimens of a
9-year-old girl who has progressive pain and swelling over her medial clavicle. An examination reveals a
swollen, tender left medial clavicle. What is the most likely diagnosis?

1. Lymphoma
2. Osteomyelitis
3. Ewing sarcoma
4. Fibrous dysplasia
5. Aneurysmal bone cyst

PREFERRED RESPONSE: 2
© 2016 American Academy of Orthopaedic Surgeons 2016 Orthopaedic In-Training Examination
130 • American Academy of Orthopaedic Surgeons

RECOMMENDED READINGS
Ceroni D, Belaieff W, Cherkaoui A, Lascombes P, Schrenzel J, de Coulon G, Dubois-Ferrière V, Dayer R.
Primary epiphyseal or apophyseal subacute osteomyelitis in the pediatric population: a report of fourteen
cases and a systematic review of the literature. J Bone Joint Surg Am. 2014 Sep 17;96(18):1570-5. doi:
10.2106/JBJS.M.00791. Review. PubMed PMID: 25232082.

Street M, Puna R, Huang M, Crawford H. Pediatric Acute Hematogenous Osteomyelitis. J Pediatr Orthop.
2015 Sep;35(6):634-9. doi: 10.1097/BPO.0000000000000332. PubMed PMID: 25333907.

© 2016 American Academy of Orthopaedic Surgeons 2016 Orthopaedic In-Training Examination


SECTION 5: Oncology • 131

Figure 273a Figure 273b

Figure 273c Figure 273d

Question 273
Figures 273a and 273b are thigh MR images of a 26-year-old man who has noted right thigh pain and a
progressive foot drop for 2 months along with numbness in his sciatic nerve distribution. A CT scan of
the chest, abdomen, and pelvis demonstrates no other sites of disease. The patient undergoes a marginal
resection of the tumor with preservation of the sciatic nerve. After surgery, the foot drop remains. Low-
and high-power pathology images are shown in Figures 273c and 273d. To optimize his outcome, the next
step should involve

1. an ankle-foot orthosis.
2. an ankle arthrodesis.
3. interfascicular neurolysis.
4. tendon transfers.
5. wide re-excision of the tumor bed.

PREFERRED RESPONSE: 5

© 2016 American Academy of Orthopaedic Surgeons 2016 Orthopaedic In-Training Examination


132 • American Academy of Orthopaedic Surgeons

RECOMMENDED READINGS
Thway K, Jones RL, Noujaim J, Fisher C. Epithelioid Sarcoma: Diagnostic Features and Genetics. Adv
Anat Pathol. 2016 Jan;23(1):41-9. doi: 10.1097/PAP.0000000000000102. PubMed PMID: 26645461.

Asano N, Yoshida A, Ogura K, Kobayashi E, Susa M, Morioka H, Iwata S, Ishii T, Hiruma T, Chuman
H, Kawai A. Prognostic Value of Relevant Clinicopathologic Variables in Epithelioid Sarcoma: A Multi-
Institutional Retrospective Study of 44 Patients. Ann Surg Oncol. 2015 Aug;22(8):2624-32. doi: 10.1245/
s10434-014-4294-1. Epub 2015 Feb 7. PubMed PMID: 25663591

© 2016 American Academy of Orthopaedic Surgeons 2016 Orthopaedic In-Training Examination


SECTION 6: Pediatrics • 133

SECTION 6: Pediatrics

Figure 3a Figure 3b

Question 3
Figures 3a and 3b are the radiographs of a patient who has a painful right leg without any sign of infection.
The original injury to the leg occurred 2 years ago when an external fixator was placed. Treatment should
now address the tibia with

1. observation.
2. surgery and no procedure to the fibula.
3. surgery and guided growth of the fibula.
4. surgery and osteotomy of the fibula.
5. amputation and prosthetic fitting.

PREFERRED RESPONSE: 4

RECOMMENDED READINGS
Teitz CC, Carter DR, Frankel VH. Problems associated with tibial fractures with intact fibulae. J Bone
Joint Surg Am. 1980 Jul; 62(5): 770-776. PubMed PMID: 7391100.

DeLee JC, Heckman JD, Lewis AG. Partial fibulectomy for ununited fractures of the tibia. J Bone Joint
Surg Am. 1981 Dec; 63(9): 1390-1395. PubMed PMID: 7320030.

© 2016 American Academy of Orthopaedic Surgeons 2016 Orthopaedic In-Training Examination


134 • American Academy of Orthopaedic Surgeons

Figure 13a Figure 13b

Question 13
Figures 13a and 13b are the radiographs of a 12-year-old girl who is referred for scoliosis. What is the best
treatment option?

1. Continued observation with follow-up in 4 months for repeat radiographs


2. Full-time thoracolumbosacral brace wear at least 13 hours per day
3. Full-time serial body casting for 3 months
4. Anterior thoracoscopic tethering procedure
5. Posterior spinal fusion with instrumentation

PREFERRED RESPONSE: 2

RECOMMENDED READINGS
Weinstein SL, Dolan LA, Wright JG, Dobbs MB. Effects of bracing in adolescents with idiopathic
scoliosis. N Engl J Med. 2013 Oct 17;369(16):1512-21. doi: 10.1056/NEJMoa1307337. Epub 2013 Sep
19. PubMed PMID: 24047455.

Sanders JO, Khoury JG, Kishan S, Browne RH, Mooney JF 3rd, Arnold KD, McConnell SJ, Bauman JA,
Finegold DN. Predicting scoliosis progression from skeletal maturity: a simplified classification during
adolescence. J Bone Joint Surg Am. 2008 Mar;90(3):540-53. doi: 10.2106/JBJS.G.00004. PubMed PMID:
18310704.

© 2016 American Academy of Orthopaedic Surgeons 2016 Orthopaedic In-Training Examination


SECTION 6: Pediatrics • 135

Question 21
A cast index higher than 0.8 can help to predict which complication following closed reduction and
casting of a pediatric distal radius fracture?

1. Skin breakdown under the cast


2. Neurologic injury
3. Growth arrest
4. Redisplacement of the fracture in a cast
5. Compartment syndrome

PREFERRED RESPONSE: 4

RECOMMENDED READINGS
Kamat AS, Pierse N, Devane P, Mutimer J, Horne G. Redefining the cast index: the optimum technique to
reduce redisplacement in pediatric distal forearm fractures. J Pediatr Orthop. 2012 Dec;32(8):787-91. doi:
10.1097/BPO.0b013e318272474d. PubMed PMID: 23147621.

McQuinn AG, Jaarsma RL. Risk factors for redisplacement of pediatric distal forearm and distal radius
fractures. J Pediatr Orthop. 2012 Oct-Nov;32(7):687-92. doi: 10.1097/BPO.0b013e31824b7525. PubMed
PMID: 22955532.

© 2016 American Academy of Orthopaedic Surgeons 2016 Orthopaedic In-Training Examination


136 • American Academy of Orthopaedic Surgeons

Figure 29a Figure 29b Figure 29c

Question 29
Figures 29a through 29c are the pelvic and knee radiographs of an 11-year-old girl who has pain on the
inner aspect of her thigh. She walks with a limp and has decreased range of motion of the right hip and
knee secondary to her pain. Her right knee is stable, and there is no joint line tenderness. What is the best
next step?

1. Arthroscopy of the right knee


2. Rest, ice, and elevation of the right limb
3. Injection of the right knee
4. Surgery on the right hip
5. Nerve block of the right limb

PREFERRED RESPONSE: 4

RECOMMENDED READINGS
Hosseinzadeh P, Iwinski HJ, Salava J, Oeffinger D. Delay in the Diagnosis of Stable Slipped Capital
Femoral Epiphysis. J Pediatr Orthop. 2015 Oct 21. [Epub ahead of print] PubMed PMID: 26491912.

Aronsson DD, Loder RT, Breur GJ, Weinstein SL. Slipped capital femoral epiphysis: current concepts. J
Am Acad Orthop Surg. 2006 Nov;14(12):666-79. Review. PubMed PMID: 17077339.

© 2016 American Academy of Orthopaedic Surgeons 2016 Orthopaedic In-Training Examination


SECTION 6: Pediatrics • 137

Question 33
A 2-week-old infant has minimal use of the right upper extremity. The child was delivered after a
prolonged and difficult labor. Which examination findings are associated with 90% recovery of function?

1. Flail extremity, no elbow flexion and ptosis


2. Torticollis with no shoulder, elbow, or wrist function
3. No shoulder abduction or external rotation, no elbow flexion or supination, intact wrist flexion
and extension, hand intrinsics intact
4. No shoulder abduction or external rotation, no elbow flexion or supination, no wrist flexion or
extension, hand intrinsics intact
5. No shoulder abduction or external rotation, no elbow flexion or supination, no wrist flexion or
extension, no hand intrinsic function

PREFERRED RESPONSE: 3

RECOMMENDED READINGS
Waters PM. Update on management of pediatric brachial plexus palsy. J Pediatr Orthop. 2005 Jan-
Feb;25(1):116-26. Review. PubMed PMID: 15614072.

Ho Christine. Disorders of the upper extremity. In: Herring JA ed. Tachdjian’s Pediatric Orthopaedics. Vol
1. 5th ed. Philadelphia, PA: Elsevier-Saunders; 2014:464-471.

Question 38
What is the biochemical effect of an intramuscular injection of botulinum toxin A?

1. Competitively blocks postsynaptic acetylcholine receptors


2. Inhibits synaptic acetylcholinesterase activity
3. Inhibits the presynaptic release of acetylcholine
4. Decreases the number of neuromuscular junctions
5. Disassociates the action of actin and myosin in targeted muscles

PREFERRED RESPONSE: 3

RECOMMENDED READINGS
Molenaers G, Fagard K, Van Campenhout A, Desloovere K. Botulinum toxin A treatment of the lower
extremities in children with cerebral palsy. J Child Orthop. 2013 Nov;7(5):383-7. doi: 10.1007/s11832-
013-0511-x. Epub 2013 Aug 28. Review. PubMed PMID: 24432099.

Aoki KR. Pharmacology and immunology of botulinum toxin type A. Clin Dermatol. 2003 Nov-
Dec;21(6):476-80. Review. PubMed PMID: 14759579.

© 2016 American Academy of Orthopaedic Surgeons 2016 Orthopaedic In-Training Examination


138 • American Academy of Orthopaedic Surgeons

Figure 43a

Question 43
Figure 43a is a standing lower extremity anteroposterior radiograph of a 2½-year-old boy who has bowed
legs. An examination reveals a 6-cm distance between the medial condyles of his knees when his feet are
touching and his legs are extended. A gait evaluation reveals a moderate varus thrust while walking. He is
slightly overweight. What is the best next step?

1. Observation
2. Bilateral knee-ankle-foot orthoses
3. Bilateral medial tibial hemiepiphysiodesis using 2-hole plate and screws
4. Bilateral medial femoral and tibial hemiepiphysiodesis using 2-hole plate and screws
5. Bilateral valgus-producing tibial osteotomies

PREFERRED RESPONSE: 1

RECOMMENDED READINGS
Bowen RE, Dorey FJ, Moseley CF. Relative tibial and femoral varus as a predictor of progression of varus
deformities of the lower limbs in young children. J Pediatr Orthop. 2002 Jan-Feb;22(1):105-11. PubMed
PMID: 11744864.

Arazi M, Oğün TC, Memik R. Normal development of the tibiofemoral angle in children: a clinical study
of 590 normal subjects from 3 to 17 years of age. J Pediatr Orthop. 2001 Mar-Apr;21(2):264-7. PubMed
PMID: 11242264.

© 2016 American Academy of Orthopaedic Surgeons 2016 Orthopaedic In-Training Examination


SECTION 6: Pediatrics • 139

Figure 49a Figure 49b Figure 49c

Figure 49d Figure 49e Figure 49f

Question 49
Figures 49a through 49f are the MR images of a 12-year-old boy who has severe back and radiating right
leg pain after falling from the back of a pickup truck 4 days ago. He can walk only short distances with
his hips in extension and his knees flexed. Straight-leg raise and crossed straight-leg raise findings are
positive; motor and sensory examination findings are otherwise within normal limits. Radiographs taken
in the emergency department are negative for fracture or dislocation. What is the best next step?

1. Observation
2. Rest, activity modification, and bracing
3. L5-S1 diskectomy with foraminotomy
4. L5-S1 decompression with instrumented posterior spinal fusion
5. Direct L5 pars repair

PREFERRED RESPONSE: 2

© 2016 American Academy of Orthopaedic Surgeons 2016 Orthopaedic In-Training Examination


140 • American Academy of Orthopaedic Surgeons

RECOMMENDED READINGS
Pizzutillo PD, Hummer CD 3rd. Nonoperative treatment for painful adolescent spondylolysis or
spondylolisthesis. J Pediatr Orthop. 1989 Sep-Oct;9(5):538-40. PubMed PMID: 2529267.

Cavalier R, Herman MJ, Cheung EV, Pizzutillo PD. Spondylolysis and spondylolisthesis in children and
adolescents: I. Diagnosis, natural history, and nonsurgical management. J Am Acad Orthop Surg. 2006
Jul;14(7):417-24. Review. PubMed PMID: 16822889.

Cheung EV, Herman MJ, Cavalier R, Pizzutillo PD. Spondylolysis and spondylolisthesis in children and
adolescents: II. Surgical management. J Am Acad Orthop Surg. 2006 Aug;14(8):488-98. Review. PubMed
PMID: 16885480.

Question 56
A 12-year-old girl has L3-L4–level myelomeningocele. She is a full community ambulator with ankle-
foot orthoses. She has had swelling and erythema of her right proximal tibia for 1 week. Her temperature
is 37.7°C. Radiographs show a mild periosteal reaction. Her erythrocyte sedimentation rate (ESR) is 40
mm/h (reference range [rr], 0-20 mm/h), and her C-reactive protein (CRP) level is 0.8 mg/L ([rr], 0.08-3.1
mg/L). What is the best next step?

1. Aspiration of the knee to rule out infection


2. Empiric antibiotic treatment
3. Open incision and drainage of osteomyelitis
4. Immobilization in a splint for 3 weeks
5. Immobilization in long-leg cast for 12 weeks

PREFERRED RESPONSE: 4

RECOMMENDED READINGS
Kumar SJ, Cowell HR, Townsend P. Physeal, metaphyseal, and diaphyseal injuries of the lower
extremities in children with myelomeningocele. J Pediatr Orthop. 1984 Jan;4(1):25-7. PubMed PMID:
6693564.

Lock TR, Aronson DD. Fractures in patients who have myelomeningocele. J Bone Joint Surg Am. 1989
Sep;71(8):1153-7. PubMed PMID: 2777841.

© 2016 American Academy of Orthopaedic Surgeons 2016 Orthopaedic In-Training Examination


SECTION 6: Pediatrics • 141

Question 64
A patient has been prepped for surgery; while working in the operating room, a resident notices an alcohol
odor while speaking to the attending surgeon. The procedure has not yet begun. What is the assisting
resident’s responsibility in this scenario?

1. Assume that the odor is from the skin prep


2. Ask the surgeon to take a break for 5 minutes
3. Report the finding immediately
4. Address the issue after the procedure because the patient has been prepped and is ready
5. Be prepared to perform the procedure without guidance

PREFERRED RESPONSE: 3

RECOMMENDED READINGS
Oreskovich MR, Shanafelt T, Dyrbye LN, Tan L, Sotile W, Satele D, West CP, Sloan J, Boone S. The
prevalence of substance use disorders in American physicians. Am J Addict 2015 Jan; 24(1): 30-38.
PubMed PMID: 25823633.

Dyrbye LN, West CP, Satele D, Boone S, Sloan J, Shanafelt TD. A national study of medical students'
attitudes toward self-prescribing and responsibility to report impaired colleagues. Acad Med. 2015
Apr;90(4):485-93. doi: 10.1097/ACM.0000000000000604. PubMed PMID: 25539515.

Question 70
An 8-year-old boy is struck by a motor vehicle. An examination in the emergency department reveals
a closed-head injury, pulmonary contusion with rib fractures, hypotension, and a displaced transverse
femur fracture. After initial resuscitation, he is stabilized but has persistent drops in blood pressure.
Treatment should consist of

1. placement of a traction pin and balanced traction for 4 weeks followed by spica casting.
2. immediate spica casting and admission to the pediatric intensive care unit (PICU) for
additional stabilization.
3. immediate open reduction and internal fixation of the femur fracture.
4. admission to the PICU for hemodynamic resuscitation and fracture fixation within 48 hours.
5. admission to the PICU and fracture fixation within 2 weeks of injury.

PREFERRED RESPONSE: 4

© 2016 American Academy of Orthopaedic Surgeons 2016 Orthopaedic In-Training Examination


142 • American Academy of Orthopaedic Surgeons

RECOMMENDED READINGS
Brundage SI, McGhan R, Jurkovich GJ, Mack CD, Maier RV. Timing of femur fracture fixation: effect on
outcome in patients with thoracic and head injuries. J Trauma. 2002 Feb;52(2):299-307. PubMed PMID:
11834992.

Mendelson SA, Dominick TS, Tyler-Kabara E, Moreland MS, Adelson PD. Early versus late femoral
fracture stabilization in multiply injured pediatric patients with closed head injury. J Pediatr Orthop. 2001
Sep-Oct;21(5):594-9. PubMed PMID: 11521025.

Figure 74

Question 74
An 8-year-old child underwent the surgical procedure on the hand as seen in Figure 74. This condition is
associated with

1. Blount disease.
2. Down syndrome.
3. achondroplasia.
4. Poland syndrome.
5. pseudoachondroplasia.

PREFERRED RESPONSE: 4

RECOMMENDED READINGS
Catena N, Divizia MT, Calevo MG, Baban A, Torre M, Ravazzolo R, Lerone M, Sénès FM. Hand and
upper limb anomalies in Poland syndrome: a new proposal of classification. J Pediatr Orthop. 2012 Oct-
Nov;32(7):727-31. doi: 10.1097/BPO.0b013e318269c898. PubMed PMID: 22955538.

Ireland DC, Takayama N, Flatt AE. Poland's syndrome. J Bone Joint Surg Am. 1976 Jan;58(1):52-8.
PubMed PMID: 175070.

© 2016 American Academy of Orthopaedic Surgeons 2016 Orthopaedic In-Training Examination


SECTION 6: Pediatrics • 143

Figure 81

Question 81
Figure 81 is the anteroposterior elbow radiograph of a 12-year-old child. What is the best treatment
option?

1. Long-arm cast for 6 weeks followed by physical therapy


2. Sling with early range of motion to prevent stiffness
3. Open reduction and internal fixation with plating and immediate range of motion
4. Closed reduction and casting accepting residual angulation of 60 degrees
5. Percutaneous reduction and immobilization for 3 weeks, followed by protected
range of motion

PREFERRED RESPONSE: 5

RECOMMENDED READINGS
Radomisli TE, Rosen AL. Controversies regarding radial neck fractures in children. Clin Orthop Relat
Res. 1998 Aug;(353):30-9. Review. PubMed PMID: 9728157.

D'souza S, Vaishya R, Klenerman L. Management of radial neck fractures in children: a retrospective


analysis of one hundred patients. J Pediatr Orthop. 1993 Mar-Apr;13(2):232-8. PubMed PMID: 8459018.

© 2016 American Academy of Orthopaedic Surgeons 2016 Orthopaedic In-Training Examination


144 • American Academy of Orthopaedic Surgeons

Question 86
An increase in which complication(s) has been seen in children with ipsilateral forearm fractures
associated with supracondylar humerus fractures?

1. Ipsilateral nerve palsy and compartment syndrome


2. Open injury and compartment syndrome
3. Pulseless extremity and ipsilateral nerve palsy
4. Associated rupture of the biceps tendon
5. Infection and iatrogenic ulnar nerve injury

PREFERRED RESPONSE: 1

RECOMMENDED READINGS
Muchow RD, Riccio AI, Garg S, Ho CA, Wimberly RL. Neurological and vascular injury associated with
supracondylar humerus fractures and ipsilateral forearm fractures in children. J Pediatr Orthop. 2015
Mar;35(2):121-5. doi: 10.1097/BPO.0000000000000230. PubMed PMID: 24919135.

Blakemore LC, Cooperman DR, Thompson GH, Wathey C, Ballock RT. Compartment syndrome in
ipsilateral humerus and forearm fractures in children. Clin Orthop Relat Res. 2000 Jul;(376):32-8.
PubMed PMID: 10906855.

Question 92
Which femur fracture pattern is most associated with nonaccidental injury?

1. Transverse
2. Spiral
3. Oblique
4. Butterfly
5. Comminuted

PREFERRED RESPONSE: 1

RECOMMENDED READINGS
Murphy R, Kelly DM, Moisan A, Thompson NB, Warner WC Jr, Beaty JH, Sawyer JR. Transverse
fractures of the femoral shaft are a better predictor of nonaccidental trauma in young children than spiral
fractures are. J Bone Joint Surg Am. 2015 Jan 21;97(2):106-11. doi: 10.2106/JBJS.N.00664. PubMed
PMID: 25609436.

Scherl SA, Miller L, Lively N, Russinoff S, Sullivan CM, Tornetta P 3rd. Accidental and nonaccidental
femur fractures in children. Clin Orthop Relat Res. 2000 Jul;(376):96-105. PubMed PMID: 10906863.

© 2016 American Academy of Orthopaedic Surgeons 2016 Orthopaedic In-Training Examination


SECTION 6: Pediatrics • 145

Figure 95

Question 95
Figure 95 is the radiograph of a 20-month-old girl who has a nonpainful limp on her left side. What is the
best treatment option?

1. Pavlik harness full time for 3 months


2. Hip abduction bracing and physical therapy
3. Closed reduction, an arthrogram with at least a 10-mm medial dye pool, and spica casting
4. Open reduction, osteotomy as needed, and spica casting
5. Observation with a shoe lift as needed

PREFERRED RESPONSE: 4

RECOMMENDED READINGS
Vitale MG, Skaggs DL. Developmental dysplasia of the hip from six months to four years of age. J Am
Acad Orthop Surg. 2001 Nov-Dec;9(6):401-11. Review. PubMed PMID: 11730331.

Race C, Herring JA. Congenital dislocation of the hip: an evaluation of closed reduction. J Pediatr Orthop.
1983 May;3(2):166-72. PubMed PMID: 6863522.

© 2016 American Academy of Orthopaedic Surgeons 2016 Orthopaedic In-Training Examination


146 • American Academy of Orthopaedic Surgeons

Figure 104a Figure 104b

Question 104
Figures 104a and 104b are the follow-up radiographs of a 6-year-old child who sustained a proximal tibia
fracture 1 year ago. Treatment of this deformity should involve

1. observation.
2. osteotomy of the tibia alone.
3. osteotomy of the tibia and fibula.
4. guided growth of the tibia alone.
5. guided growth of the tibia and fibula.

PREFERRED RESPONSE: 1

RECOMMENDED READINGS
Zionts LE, MacEwen GD. Spontaneous improvement of post-traumatic tibia valga. J Bone Joint Surg Am
1986 June; 68(5): 680-687. PubMed PMID: 3722224.

Jackson DW, Cozen L. Genu valgum as a complication of proximal tibial metaphyseal fractures in
children. J Bone and Joint Surg Am 1971 Dec; 53(8): 1571-1578. PubMed PMID: 5121797.

© 2016 American Academy of Orthopaedic Surgeons 2016 Orthopaedic In-Training Examination


SECTION 6: Pediatrics • 147

Figure 113a Figure 113b

Question 113
Figures 113a and 113b are the radiographs of a 14-year-old boy who is seen in the emergency department
after injuring his knee while playing basketball. He has substantial anterior knee swelling, an inability
to actively extend his knee, and normal sensory examination findings of the lower leg. What is the best
treatment option?

1. Closed reduction and immobilization in a cylinder cast


2. Closed reduction and percutaneous fixation with cannulated screws
3. Open reduction and internal fixation (ORIF) with multiple cannulated screws
4. Direct repair of the patella tendon
5. Spanning-knee external fixation

PREFERRED RESPONSE: 3

RECOMMENDED READINGS
Pretell-Mazzini J, Kelly DM, Sawyer JR, Esteban EM, Spence DD, Warner WC Jr, Beaty JH. Outcomes
and Complications of Tibial Tubercle Fractures in Pediatric Patients: A Systematic Review of the
Literature. J Pediatr Orthop. 2015 Apr 10. [Epub ahead of print] PubMed PMID: 25887827.

Jakoi A, Freidl M, Old A, Javandel M, Tom J, Realyvasquez J. Tibial tubercle avulsion fractures in
adolescent basketball players. Orthopedics. 2012 Aug 1;35(8):692-6. doi: 10.3928/01477447-20120725-
07. PubMed PMID: 22868593.

© 2016 American Academy of Orthopaedic Surgeons 2016 Orthopaedic In-Training Examination


148 • American Academy of Orthopaedic Surgeons

Figure 122a Figure 122b

Question 122
Figures 122a and 122b are the radiographs of a 6-year-old child who fell from a tree. The most common
neurologic injury associated with this fracture could result in which functional deficit?

1. Loss of sensation to the thumb and long finger


2. Inability to extend the wrist and fingers
3. Inability to adduct fingers
4. Inability to flex the wrist
5. Inability to flex the thumb interphalangeal joint

PREFERRED RESPONSE: 5

RECOMMENDED READINGS
Abzug JM, Herman MJ. Management of supracondylar humerus fractures in children: current concepts. J
Am Acad Orthop Surg. 2012 Feb;20(2):69-77. doi: 10.5435/JAAOS-20-02-069. Review. PubMed PMID:
22302444.

Babal JC, Mehlman CT, Klein G. Nerve injuries associated with pediatric supracondylar
humeral fractures: a meta-analysis. J Pediatr Orthop. 2010 Apr-May;30(3):253-63. doi: 10.1097/
BPO.0b013e3181d213a6. PubMed PMID: 20357592.

© 2016 American Academy of Orthopaedic Surgeons 2016 Orthopaedic In-Training Examination


SECTION 6: Pediatrics • 149

Figure 129a Figure 129b

Question 129
Figures 129a and 129b are the radiographs of a 5-year-old boy who has a deformity of the lower leg. His
parents note that the deformity has been slowly improving since he began walking. At skeletal maturity,
this boy may expect to have

1. limb-length discrepancy.
2. scoliosis.
3. skin lesions.
4. an increased risk for bone tumors.
5. an increased risk for neurologic issues.

PREFERRED RESPONSE: 1

RECOMMENDED READINGS
Shah HH, Doddabasappa SN, Joseph B. Congenital posteromedial bowing of the tibia: a retrospective
analysis of growth abnormalities in the leg. J Pediatr Orthop B. 2009 May;18(3):120-8. doi: 10.1097/
BPB.0b013e328329dc86. PubMed PMID: 19339901.

Pappas AM. Congenital posteromedial bowing of the tibia and fibula. J Pediatr Orthop. 1984
Sep;4(5):525-31. PubMed PMID: 6490868.

© 2016 American Academy of Orthopaedic Surgeons 2016 Orthopaedic In-Training Examination


150 • American Academy of Orthopaedic Surgeons

Question 138
A 6-year-old boy underwent closed reduction and pinning of a type 3 supracondylar humerus fracture.
The pins were removed at 3 weeks; his radiographs at that time showed an abundance of callus. Six weeks
later, he has limited elbow motion. His current range of motion is 45 to 100 degrees. What is the best next
step?

1. Recast for an additional 3 weeks because of a possible delayed union


2. Return to the operating room for manipulation and possible arthroscopic contracture release
3. Reassure and observe for the return of motion over 3 to 6 months
4. Order a dynamic elbow brace and a gradual increase in motion
5. Initiate indomethacin treatment

PREFERRED RESPONSE: 3

RECOMMENDED READINGS
Spencer HT, Wong M, Fong YJ, Penman A, Silva M. Prospective longitudinal evaluation of elbow motion
following pediatric supracondylar humeral fractures. J Bone Joint Surg Am. 2010 Apr;92(4):904-10. doi:
10.2106/JBJS.I.00736. PubMed PMID: 20360514.

Keppler P, Salem K, Schwarting B, Kinzl L. The effectiveness of physiotherapy after operative treatment
of supracondylar humeral fractures in children. J Pediatr Orthop. 2005 May-Jun;25(3):314-6. PubMed
PMID: 15832145.

Question 143
For a patient with achondroplasia, the thoracolumbar kyphosis seen in infancy will most likely

1. resolve with independent walking.


2. increase with independent walking.
3. increase with skeletal maturity.
4. remain unchanged with independent walking.
5. remain unchanged with skeletal maturity.

PREFERRED RESPONSE: 1

RECOMMENDED READINGS
Shirley ED, Ain MC. Achondroplasia: manifestations and treatment. J Am Acad Orthop Surg. 2009
Apr;17(4):231-41. Review. PubMed PMID: 19307672.

Engberts AC, Jacobs WC, Castelijns SJ, Castelein RM, Vleggeert-Lankamp CL. The prevalence of
thoracolumbar kyphosis in achondroplasia: a systematic review. J Child Orthop. 2012 Mar;6(1):69-73.
doi: 10.1007/s11832-011-0378-7. Epub 2011 Dec 3. PubMed PMID: 22442656.

© 2016 American Academy of Orthopaedic Surgeons 2016 Orthopaedic In-Training Examination


SECTION 6: Pediatrics • 151

Figure 152

Question 152
Figure 152 is the radiograph of a 3-year-old child who has a foot deformity. At this age, treatment should
consist of

1. observation.
2. bracing.
3. nonsurgical treatment using the Ponseti method alone.
4. serial casting followed by surgery.
5. anterior tibial tendon transfer surgery.

PREFERRED RESPONSE: 4

RECOMMENDED READINGS
Miller M, Dobbs MB. Congenital Vertical Talus: Etiology and Management. J Am Acad Orthop Surg.
2015 Oct;23(10):604-11. doi: 10.5435/JAAOS-D-14-00034. Epub 2015 Sep 3. Review. PubMed PMID:
26337950.

Yang JS, Dobbs MB. Treatment of Congenital Vertical Talus: Comparison of Minimally Invasive and
Extensive Soft-Tissue Release Procedures at Minimum Five-Year Follow-up. J Bone Joint Surg Am. 2015
Aug 19;97(16):1354-65. doi: 10.2106/JBJS.N.01002. PubMed PMID: 26290087.

© 2016 American Academy of Orthopaedic Surgeons 2016 Orthopaedic In-Training Examination


152 • American Academy of Orthopaedic Surgeons

Figure 162a Figure 162b

Question 162
Figures 162a and 162b are the pelvic radiographs of a 13-year-old boy who is seen in the emergency
department. He has a 1-year history of pain in his right hip with weight bearing. An examination reveals a
shortened right leg that is held in external rotation and has limited flexion with obligate external rotation
with maximal hip flexion. An examination of his left hip reveals full and normal motion without pain.
What is the best next step?

1. Immediate in situ fixation of the right hip with a cannulated screw


2. Immediate right surgical hip dislocation, proximal femoral osteotomy, and relative lengthening
of the femoral neck
3. Immediate bilateral hip fixation with cannulated screws
4. Physical therapy for improved right hip range of motion
5. Delayed in situ fixation of the right hip with a cannulated screw after physical therapy for
improved hip range of motion

PREFERRED RESPONSE: 1

RECOMMENDED READINGS
Wensaas A, Svenningsen S, Terjesen T. Long-term outcome of slipped capital femoral epiphysis: a 38-year
follow-up of 66 patients. J Child Orthop. 2011 Apr;5(2):75-82. doi: 10.1007/s11832-010-0308-0. Epub
2010 Dec 12. PubMed PMID: 21594079.

Thawrani DP, Feldman DS, Sala DA. Current Practice in the Management of Slipped Capital Femoral
Epiphysis. J Pediatr Orthop. 2015 Apr 15. [Epub ahead of print] PubMed PMID: 25929770.

© 2016 American Academy of Orthopaedic Surgeons 2016 Orthopaedic In-Training Examination


SECTION 6: Pediatrics • 153

Question 169
A 4-year-old child cannot fully extend the small finger. An examination reveals a 45-degree flexion
contracture at the little finger proximal interphalangeal joint with soft-tissue webbing. The extensor and
flexion tendons are intact. There is no history of trauma. What is the best initial treatment?

1. Reassurance; the natural history is near-complete resolution


2. Z-plasty of the webbing with extension casting
3. Full excision of the fibrous band with intrinsic rebalancing
4. Digital stretching with progressive nighttime splinting
5. Proximal phalanx extension osteotomy with shortening

PREFERRED RESPONSE: 4

RECOMMENDED READINGS
Comer GC, Ladd AL. Management of complications of congenital hand disorders. Hand Clin. 2015
May;31(2):361-75. doi: 10.1016/j.hcl.2015.01.011. Review. PubMed PMID: 25934210.

Zancolli E. Congenital abnormalities of the retinacular system of the hand. In: Gupta A, Kay SPJ, Scheker
LR, eds. The Growing Hand: Diagnosis and Management of the Upper Extremity in Children. London,
England: Mosby; 2000:367-374.

© 2016 American Academy of Orthopaedic Surgeons 2016 Orthopaedic In-Training Examination


154 • American Academy of Orthopaedic Surgeons

Figure 174

Question 174
Figure 174 is a picture of a 3-year-old boy who has a foot deformity. The parents are concerned about
the shape of his foot. He walked at 12 months, has no limp, and runs and plays without difficulty. An
examination reveals the foot is flexible in the hindfoot, no pain is elicited with palpation or motion, and
his gait is normal for a 3-year-old. What is the best next step?

1. Obtain an MRI of the foot


2. Obtain a CT scan of the foot
3. Reassure and educate the parents
4. Prescribe inserts for medial foot support
5. Prescribe reverse last shoes until the deformity improves

PREFERRED RESPONSE: 3

RECOMMENDED READINGS
Jane MacKenzie A, Rome K, Evans AM. The efficacy of nonsurgical interventions for pediatric flexible
flat foot: a critical review. J Pediatr Orthop. 2012 Dec;32(8):830-4. doi: 10.1097/BPO.0b013e3182648c95.
Review. PubMed PMID: 23147627.

Pfeiffer M, Kotz R, Ledl T, Hauser G, Sluga M. Prevalence of flat foot in preschool-aged children.
Pediatrics. 2006 Aug;118(2):634-9. PubMed PMID: 16882817.

© 2016 American Academy of Orthopaedic Surgeons 2016 Orthopaedic In-Training Examination


SECTION 6: Pediatrics • 155

Figure 183a Figure 183b

Question 183
Figures 183a and 183b are the radiographs of a 12-year-old boy who sustained an injury. After reduction,
what is the most likely cause of physeal arrest?

1. The amount of initial displacement


2. The size of the metaphyseal fragment
3. A postreduction angulation exceeding 20 degrees
4. A postreduction physeal gap exceeding 3 mm
5. A concomitant fibular fracture

PREFERRED RESPONSE: 4

RECOMMENDED READINGS
Barmada A, Gaynor T, Mubarak SJ. Premature physeal closure following distal tibia physeal fractures: a
new radiographic predictor. J Pediatr Orthop. 2003 Nov-Dec;23(6):733-9. PubMed PMID: 14581776.

Wuerz TH, Gurd DP. Pediatric physeal ankle fracture. J Am Acad Orthop Surg. 2013 Apr;21(4):234-44.
doi: 10.5435/JAAOS-21-04-234. Review. PubMed PMID: 23545729.

© 2016 American Academy of Orthopaedic Surgeons 2016 Orthopaedic In-Training Examination


156 • American Academy of Orthopaedic Surgeons

Question 189
A 4-year-old girl has bilateral ankle swelling and a swollen right knee. She has a mild limp but no pain
with walking or with joint range of motion. Her temperature is 37.7°C, her erythrocyte sedimentation rate
is 45 mm/h (reference range [rr], 0-20 mm/h), and her C-reactive protein level is 1.5 mg/L (rr, 0.08-3.1
mg/L). After treatment initiation, what is the best next step?

1. Echocardiogram to rule out the source of multiple infection


2. Hematology consultation
3. MRI of the lower extremities
4. Ophthalmologic consultation
5. Infectious disease consultation

PREFERRED RESPONSE: 4

RECOMMENDED READINGS
Punaro M. Rheumatologic conditions in children who may present to the orthopaedic surgeon. J Am Acad
Orthop Surg. 2011 Mar;19(3):163-9. Review. PubMed PMID: 21368097.

Herring JA. Arthritis. In: Herring. ed. Tachdjian’s Pediatric Orthopaedics. Vol 1. 5th ed. Philadelphia, PA:
Elsevier-Saunders; 2014:1007-1017.

Question 195
A 4-year-old boy is seen with his parents in the emergency department after an unwitnessed fall. He has
pain in his right thigh and no other obvious injury. A radiograph reveals a spiral right femur fracture with
no comminution and 1 cm of shortening. What is best next step?

1. Skeletal survey
2. CT scan of the head
3. Child protective services notification
4. Hip spica casting
5. Flexible intramedullary nailing

PREFERRED RESPONSE: 4

© 2016 American Academy of Orthopaedic Surgeons 2016 Orthopaedic In-Training Examination


SECTION 6: Pediatrics • 157

RECOMMENDED READINGS
Flaherty EG, Perez-Rossello JM, Levine MA, Hennrikus WL; American Academy of Pediatrics
Committee on Child Abuse and Neglect; Section on Radiology, American Academy of Pediatrics; Section
on Endocrinology, American Academy of Pediatrics; Section on Orthopaedics, American Academy of
Pediatrics; Society for Pediatric Radiology. Evaluating children with fractures for child physical abuse.
Pediatrics. 2014 Feb;133(2):e477-89. doi: 10.1542/peds.2013-3793. Epub 2014 Jan 27. PubMed PMID:
24470642.

Kocher MS, Sink EL, Blasier RD, Luhmann SJ, Mehlman CT, Scher DM, Matheney T, Sanders JO,
Watters WC 3rd, Goldberg MJ, Keith MW, Haralson RH 3rd, Turkelson CM, Wies JL, Sluka P, McGowan
R; American Academy of Orthopaedic Surgeons. American Academy of Orthopaedic Surgeons clinical
practice guideline on treatment of pediatric diaphyseal femur fracture. J Bone Joint Surg Am. 2010 Jul
21;92(8):1790-2. doi: 10.2106/JBJS.J.00137. PubMed PMID: 20660244.

Figure 210a Figure 210b

Question 210
Figures 210a and 210b are the radiographs of a 5-year-old child who sustained an injury after a fall from a
monkey bar. Treatment should include reduction of the

1. ulna and ulnohumeral joint.


2. ulna and distal radioulnar joint.
3. ulna alone.
4. ulna and radiocapitellar joint.
5. ulnohumeral joint alone.

PREFERRED RESPONSE: 4

© 2016 American Academy of Orthopaedic Surgeons 2016 Orthopaedic In-Training Examination


158 • American Academy of Orthopaedic Surgeons

RECOMMENDED READINGS
Kozin SH, Abzug JM, Safier S, Herman MJ. Complications of pediatric elbow dislocations and monteggia
fracture-dislocations. Instr Course Lect. 2015; 64: 493-498. PubMed PMID: 25745932.

Ring D. Monteggia fractures. Orthop Clin North Am. 2013 Jan; 44(1): 59-66. PubMed PMID: 23174326.

Figure 223

Question 223
Figure 223 is the anteroposterior weight-bearing radiograph of a 10-year-old boy who has had intermittent
ankle pain for 6 months. What is the best treatment option?

1. An ankle brace and initiation of physical therapy with an ankle stabilization protocol.
2. Distal tibial osteotomy
3. Medial hemiepiphysiodesis with a screw or plate
4. Excision of osteochondromas and a fibular osteotomy with tenodesis tendon transfer to the
distal fibula
5. Complete epiphysiodesis and osteotomy with gradual correction with an external fixator

PREFERRED RESPONSE: 3

RECOMMENDED READINGS
Stieber JR, Dormans JP. Manifestations of hereditary multiple exostoses. J Am Acad Orthop Surg. 2005
Mar-Apr;13(2):110-20. Review. PubMed PMID: 15850368.

Rupprecht M, Spiro AS, Rueger JM, Stücker R. Temporary screw epiphyseodesis of the distal tibia: a
therapeutic option for ankle valgus in patients with hereditary multiple exostosis. J Pediatr Orthop. 2011
Jan-Feb;31(1):89-94. doi: 10.1097/BPO.0b013e318202c20e. PubMed PMID: 21150737.

© 2016 American Academy of Orthopaedic Surgeons 2016 Orthopaedic In-Training Examination


SECTION 6: Pediatrics • 159

Figure 229a

Question 229
Figure 229a is the radiograph of a 22-month-old boy who has infantile idiopathic scoliosis. What is the
preferred initial treatment to minimize curve progression?

1. Observation
2. Serial casting
3. Rib-to-spine growing rods
4. Spine-to-spine growing rods
5. Posterior spinal fusion with instrumentation

PREFERRED RESPONSE: 2

RECOMMENDED READINGS
Sanders JO, D’Astous J, Fitzgerald M, Khoury JG, Kishan S, Sturm PF. Derotational casting
for progressive infantile scoliosis. J Pediatr Orthop. 2009 Sep;29(6):581-7. doi: 10.1097/
BPO.0b013e3181b2f8df. PubMed PMID: 19700987.

Tis JE, Karlin LI, Akbarnia BA, Blakemore LC, Thompson GH, McCarthy RE, Tello CA, Mendelow MJ,
Southern EP; Growing Spine Committee of the Scoliosis Research Society. Early onset scoliosis: modern
treatment and results. J Pediatr Orthop. 2012 Oct-Nov;32(7):647-57. Review. PubMed PMID: 22955526.

Waldron SR, Poe-Kochert C, Son-Hing JP, Thompson GH. Early onset scoliosis: the value of serial risser
casts. J Pediatr Orthop. 2013 Dec;33(8):775-80. doi: 10.1097/BPO.0000000000000072. PubMed PMID:
23965912.

© 2016 American Academy of Orthopaedic Surgeons 2016 Orthopaedic In-Training Examination


160 • American Academy of Orthopaedic Surgeons

Video 239 (scan to view)

Question 239
Video 239 is the CT scan of an 8-year-old boy who has neck pain and torticollis. He reports waking up
from sleep 4 weeks ago with neck pain and noticed his head was stuck in a position in which it was tilted
to the left and turned to the right. His mother mentions a recent upper respiratory tract infection, but no
other recent medical history. What is the best next step?

1. Nonsteroidal anti-inflammatory drugs (NSAIDs) and a soft cervical collar


2. Cervical traction followed by use of a soft cervical collar for 4 weeks
3. Cervical traction followed by halo-vest placement to maintain reduction
4. Posterior C1-C2 fusion using the Gallie technique
5. Observation

PREFERRED RESPONSE: 3

RECOMMENDED READINGS
Neal KM, Mohamed AS. Atlantoaxial rotatory subluxation in children. J Am Acad Orthop Surg. 2015
Jun;23(6):382-92. doi: 10.5435/JAAOS-D-14-00115. Review. PubMed PMID: 26001430.

Rahimi SY, Stevens EA, Yeh DJ, Flannery AM, Choudhri HF, Lee MR. Treatment of atlantoaxial
instability in pediatric patients. Neurosurg Focus. 2003 Dec 15;15(6):ECP1. Review. PubMed PMID:
15305843.

© 2016 American Academy of Orthopaedic Surgeons 2016 Orthopaedic In-Training Examination


SECTION 6: Pediatrics • 161

Figure 247

Question 247
After treatment, the child seen in Figure 247 is expected to

1. walk and run with a prosthesis.


2. walk only with a prosthesis.
3. walk only.
4. run and walk.
5. not run and not walk.

PREFERRED RESPONSE: 4

RECOMMENDED READINGS
Ziont LE. What's New in Idiopathic Clubfoot? J Pediatr Orthop. 2015 Sep; 35(6): 547-50. PubMed PMID:
25298569.

Lohle-Akkerskijk JJ, Rameckers EA, Adriesse H, de Reus I, van Erve RH. Walking capacity of children
with clubfeet in primary school: something to worry about? J Pediatr Orthop B. 2015 Jan; 24(1): 18-23.
PubMed PMID: 25350905.

© 2016 American Academy of Orthopaedic Surgeons 2016 Orthopaedic In-Training Examination


162 • American Academy of Orthopaedic Surgeons

Question 265
An 8-year-boy with hemiplegic cerebral palsy is evaluated for toe-walking gait on his right leg. An
examination reveals he has no hip or knee flexion contracture and he cannot place his foot flat during the
stance phase of gait. He has maximal ankle dorsiflexion of 5 degrees with his knee flexed and 10 degrees
short of neutral with his knee extended. What is the best surgical treatment?

1. Open Achilles lengthening


2. Complete release of the Achilles tendon
3. Complete anterior tibial tendon transfer
4. Gastrocnemius recession
5. Split anterior tibial tendon transfer

PREFERRED RESPONSE: 4

RECOMMENDED READINGS
Barouk P, Barouk LS. Clinical diagnosis of gastrocnemius tightness. Foot Ankle Clin. 2014
Dec;19(4):659-67. doi: 10.1016/j.fcl.2014.08.004. Epub 2014 Sep 26. Review. PubMed PMID: 25456715.

Karol LA. Surgical management of the lower extremity in ambulatory children with cerebral palsy. J Am
Acad Orthop Surg. 2004 May-Jun;12(3):196-203. Review. PubMed PMID: 15161173.

© 2016 American Academy of Orthopaedic Surgeons 2016 Orthopaedic In-Training Examination


SECTION 7: Shoulder and Elbow • 163

SECTION 7: Shoulder and Elbow

Figure 12a Figure 12b

Question 12
Figures 12a and 12b are the postsurgical radiographs of a 22-year-old man who underwent an open
coracoid transfer to address recurrent right shoulder instability 5 days ago. He has had an occasional low-
grade fever (37.2°C) since surgery. His surgical site is completely benign. Palpation of the axilla shows
no fullness. He demonstrates an inability to abduct his shoulder against resistance and decreased sensation
on the lateral aspect of his shoulder. He also has decreased sensation along the volar lateral aspect of his
ipsilateral forearm. What is the best next step?

1. Continued observation with follow-up in 2 to 3 weeks


2. Urgent electromyography (EMG)/nerve conduction velocity studies of the right
upper extremity
3. CT scan to evaluate for surgical-site hematoma
4. Laboratory studies (erythrocyte sedimentation rate, C-reactive protein, and complete
blood count)
5. Graft position revision and possible neurolysis

PREFERRED RESPONSE: 1

© 2016 American Academy of Orthopaedic Surgeons 2016 Orthopaedic In-Training Examination


164 • American Academy of Orthopaedic Surgeons

RECOMMENDED READINGS
Shah AA, Butler RB, Romanowski J, Goel D, Karadagli D, Warner JJ. Short-term complications of the
Latarjet procedure. J Bone Joint Surg Am. 2012 Mar 21;94(6):495-501. doi: 10.2106/JBJS.J.01830.
PubMed PMID: 22318222.

Delaney RA, Freehill MT, Janfaza DR, Vlassakov KV, Higgins LD, Warner JJ. 2014 Neer Award Paper:
neuromonitoring the Latarjet procedure. J Shoulder Elbow Surg. 2014 Oct;23(10):1473-80. doi: 10.1016/j.
jse.2014.04.003. Epub 2014 Jun 18. PubMed PMID: 24950948.

Freehill MT, Srikumaran U, Archer KR, McFarland EG, Petersen SA. The Latarjet coracoid
process transfer procedure: alterations in the neurovascular structures. J Shoulder Elbow Surg. 2013
May;22(5):695-700. doi: 10.1016/j.jse.2012.06.003. Epub 2012 Sep 1. PubMed PMID: 22947236.

Boardman ND 3rd, Cofield RH. Neurologic complications of shoulder surgery. Clin Orthop Relat Res.
1999 Nov;(368):44-53. Review. PubMed PMID: 10613152.

Figure 26a*
*Used with permission from Petilon J, Carr DR, Sekiya JK, Unger DV. Pectoralis major muscle injuries: evaluation and management. J Am
Acad Orthop Surg. 2005 Jan-Feb;13(1):59-68. Review.

Question 26
Figure 26a is the clinical photograph of a 31-year-old recreational athlete who felt a pop in his right
shoulder while performing a bench press. Appropriate treatment involves reinsertion of the involved
tendon onto the

1. footprint area of the greater tuberosity.


2. lesser tuberosity at the area of avulsion.
3. humeral shaft lateral to the bicipital groove.
4. humeral shaft medial to the bicipital groove.
5. humeral shaft on the inferior edge of the quadrilateral space.

PREFERRED RESPONSE: 3

© 2016 American Academy of Orthopaedic Surgeons 2016 Orthopaedic In-Training Examination


SECTION 7: Shoulder and Elbow • 165

RECOMMENDED READINGS
Petilon J, Carr DR, Sekiya JK, Unger DV. Pectoralis major muscle injuries: evaluation and management. J
Am Acad Orthop Surg. 2005 Jan-Feb;13(1):59-68. Review. PubMed PMID: 15712983.

Butt U, Mehta S, Funk L, Monga P. Pectoralis major ruptures: a review of current management. J Shoulder
Elbow Surg. 2015 Apr;24(4):655-62. doi: 10.1016/j.jse.2014.10.024. Epub 2015 Jan 1. Review. PubMed
PMID: 25556808.

Figure 34a Figure 34b Figure 34c

Question 34
Figures 34a through 34c are the radiograph and MR images of a 62-year-old left-hand-dominant woman
who has had left shoulder pain for 6 months. The pain bothers her mostly at night. She has been taking
nonsteroidal anti-inflammatory drugs (NSAIDs) as needed and received a cortisone injection several years
ago in the same shoulder. An examination reveals mildly diminished range of motion in elevation and
external rotation, and she reports severe pain with forced shoulder abduction. Her strength in elevation is
4/5 (limited by pain) and external rotation strength is 5/5. What is the most appropriate next step?

1. Ultrasound-guided intra-articular cortisone injection


2. Subacromial cortisone injection followed by formal physical therapy
3. Arthroscopic capsular release with manipulation under anesthesia
4. Arthroscopic subacromial decompression with release of the coracoacromial ligament
5. Arthroscopic takedown of the supraspinatus, followed by repair and decompression

PREFERRED RESPONSE: 2

© 2016 American Academy of Orthopaedic Surgeons 2016 Orthopaedic In-Training Examination


166 • American Academy of Orthopaedic Surgeons

RECOMMENDED READINGS
Ketola S, Lehtinen J, Arnala I, Nissinen M, Westenius H, Sintonen H, Aronen P, Konttinen YT,
Malmivaara A, Rousi T. Does arthroscopic acromioplasty provide any additional value in the treatment
of shoulder impingement syndrome?: a two-year randomised controlled trial. J Bone Joint Surg Br. 2009
Oct;91(10):1326-34. doi: 10.1302/0301-620X.91B10.22094. PubMed PMID: 19794168.

Pedowitz RA, Yamaguchi K, Ahmad CS, Burks RT, Flatow EL, Green A, Iannotti JP, Miller BS, Tashjian
RZ, Watters WC 3rd, Weber K, Turkelson CM, Wies JL, Anderson S, St Andre J, Boyer K, Raymond
L, Sluka P, McGowan R; American Academy of Orthopaedic Surgeons. Optimizing the management of
rotator cuff problems. J Am Acad Orthop Surg. 2011 Jun;19(6):368-79. PubMed PMID: 21628648.

Ketola S, Lehtinen J, Rousi T, Nissinen M, Huhtala H, Konttinen YT, Arnala I. No evidence of long-term
benefits of arthroscopicacromioplasty in the treatment of shoulder impingement syndrome: Five-year
results of a randomised controlled trial. Bone Joint Res. 2013 Jul 1;2(7):132-9. doi: 10.1302/2046-
3758.27.2000163. Print 2013. PubMed PMID: 23836479.

Haahr JP, Østergaard S, Dalsgaard J, Norup K, Frost P, Lausen S, Holm EA, Andersen JH. Exercises
versus arthroscopic decompression in patients with subacromial impingement: a randomised, controlled
study in 90 cases with a one year follow up. Ann Rheum Dis. 2005 May;64(5):760-4. PubMed PMID:
15834056.

© 2016 American Academy of Orthopaedic Surgeons 2016 Orthopaedic In-Training Examination


SECTION 7: Shoulder and Elbow • 167

Video 41 (scan to view)

Question 41
Video 41 features a 45-year-old man who has long-standing right shoulder pain that is refractory to
physical therapy. The figure provides intraoperative arthroscopic images taken while viewing from the
lateral portal in the beach-chair position. After undergoing this isolated procedure, which upper extremity
motion should be avoided during the initial 2 weeks following surgery?

1. Supine passive shoulder-forward elevation


2. Supine active abduction and external rotation of the shoulder
3. Seated active shoulder internal rotation
4. Seated active forearm supination with the elbow flexed 90 degrees
5. Seated active forearm pronation with the elbow flexed 90 degrees

PREFERRED RESPONSE: 4

RECOMMENDED READINGS
Shank JR, Singleton SB, Braun S, Kissenberth MJ, Ramappa A, Ellis H, Decker MJ, Hawkins RJ, Torry
MR. A comparison of forearm supination and elbow flexion strength in patients with long head of the
biceps tenotomy or tenodesis. Arthroscopy. 2011 Jan;27(1):9-16. doi: 10.1016/j.arthro.2010.06.022. Epub
2010 Oct 29. PubMed PMID: 21035992.

Levy AS, Kelly BT, Lintner SA, Osbahr DC, Speer KP. Function of the long head of the biceps at the
shoulder: electromyographic analysis. J Shoulder Elbow Surg. 2001 May-Jun;10(3):250-5. PubMed
PMID: 11408907.

Yamaguchi K, Riew KD, Galatz LM, Syme JA, Neviaser RJ. Biceps activity during shoulder motion: an
electromyographic analysis. Clin Orthop Relat Res. 1997 Mar;(336):122-9. PubMed PMID: 9060495.

© 2016 American Academy of Orthopaedic Surgeons 2016 Orthopaedic In-Training Examination


168 • American Academy of Orthopaedic Surgeons

Figure 53a Figure 53b Figure 53c

Question 53
The condition seen in Figures 53a through 53c likely represents progression of

1. unrecognized Propionibacterium acnes (P. acnes) infection.


2. an untreated rotator cuff tear with a long-head biceps tendon rupture.
3. inflammatory synovitis associated with rheumatoid arthritis.
4. joint breakdown secondary to acquired hemophilia.
5. chronic hypertrophic nonunion of a proximal humeral fracture.

PREFERRED RESPONSE: 2

RECOMMENDED READINGS
Ecklund KJ, Lee TQ, Tibone J, Gupta R. Rotator cuff tear arthropathy. J Am Acad Orthop Surg. 2007
Jun;15(6):340-9. Review. PubMed PMID: 17548883.

Feeley BT, Gallo RA, Craig EV. Cuff tear arthropathy: current trends in diagnosis and surgical
management. J Shoulder Elbow Surg. 2009 May-Jun;18(3):484-94. doi: 10.1016/j.jse.2008.11.003. Epub
2009 Feb 8. Review. PubMed PMID: 19208484.

Question 66
A 78-year-old man has a 4-part fracture of the proximal humerus. He is scheduled for a reverse total
shoulder arthroplasty (TSA). Objective improvement in active external rotation will most consistently be
achieved with which surgical technique?

1. Inferior placement of the baseplate


2. Increased retroversion of the humeral stem
3. Placement of a thicker polyethylene insert
4. Suture fixation of the greater tuberosity
5. Use of a more valgus neck-shaft angled stem

PREFERRED RESPONSE: 4

© 2016 American Academy of Orthopaedic Surgeons 2016 Orthopaedic In-Training Examination


SECTION 7: Shoulder and Elbow • 169

RECOMMENDED READINGS
Jobin CM, Galdi B, Anakwenze OA, Ahmad CS, Levine WN. Reverse shoulder arthroplasty for the
management of proximal humerus fractures. J Am Acad Orthop Surg. 2015 Mar;23(3):190-201. doi:
10.5435/JAAOS-D-13-00190. Epub 2015 Jan 28. Review. PubMed PMID: 25630370.

Anakwenze OA, Zoller S, Ahmad CS, Levine WN. Reverse shoulder arthroplasty for acute proximal
humerus fractures: a systematic review. J Shoulder Elbow Surg. 2014 Apr;23(4):e73-80. doi: 10.1016/j.
jse.2013.09.012. Epub 2014 Jan 7. Review. PubMed PMID: 24406120.

Gallinet D, Adam A, Gasse N, Rochet S, Obert L. Improvement in shoulder rotation in complex shoulder
fractures treated by reverse shoulder arthroplasty. J Shoulder Elbow Surg. 2013 Jan;22(1):38-44. doi:
10.1016/j.jse.2012.03.011. Epub 2012 Jun 15. PubMed PMID: 22705317.

Question 85
A 45-year-old laborer has pain and stiffness of his dominant right elbow that has gradually worsened for 6
months. He has received several injections, experienced diminishing success, and is now taking narcotic
pain medication to relieve his symptoms. He is still working despite the pain and reports that he cannot
take much time off from work or he will lose his job. An examination reveals full strength with extension
of 30 degrees, flexion of 120 degrees, pronation of 90 degrees, and supination of 85 degrees. He has mild
crepitus with motion. Radiographs reveal moderate osteoarthritis with osteophytes of the humerus and
ulna. His radiocapitellar joint has minimal arthritic change. What is the best option for treatment?

1. Substitution of narcotic pain medication for prescription nonsteroidal anti-inflammatory


drugs (NSAIDs)
2. Injection of the ulnohumeral joint with corticosteroids followed by intense physical therapy
3. Arthroscopic fenestration of the olecranon fossa and osteophyte debridement
4. Open debridement of osteophytes on the humerus and ulna with radial head resection
5. Total elbow arthroplasty with preservation of the nonarthritic native radial head using a linked
ulnar prosthesis

PREFERRED RESPONSE: 3

RECOMMENDED READINGS
Beingessner DM, Dunning CE, Gordon KD, Johnson JA, King GJ. The effect of radial head excision and
arthroplasty on elbow kinematics and stability. J Bone Joint Surg Am. 2004 Aug;86-A(8):1730-9. PubMed
PMID: 15292422.

Krishnan SG, Harkins DC, Pennington SD, Harrison DK, Burkhead WZ. Arthroscopic ulnohumeral
arthroplasty for degenerative arthritis of the elbow in patients under fifty years of age. J Shoulder Elbow
Surg. 2007 Jul-Aug;16(4):443-8. Epub 2007 Jan 24. PubMed PMID: 17254810.

© 2016 American Academy of Orthopaedic Surgeons 2016 Orthopaedic In-Training Examination


170 • American Academy of Orthopaedic Surgeons

Question 89
When performing a primary anatomic total shoulder arthroplasty for glenohumeral osteoarthritis,
reproducing the center of rotation of the glenohumeral joint is important to achieve symmetric joint
balancing. Relative to the center of the humeral intramedullary canal, the offset of the humeral head center
of rotation (COR) generally lies

1. 8 mm posterior and 4 mm medial.


2. 8 mm posterior and 4 mm lateral.
3. 8 mm anterior and 4 mm lateral.
4. 4 mm anterior and 4 mm medial.
5. 4 mm medial only.

PREFERRED RESPONSE: 1

RECOMMENDED READINGS
Boileau P, Walch G. The three-dimensional geometry of the proximal humerus. Implications for surgical
technique and prosthetic design. J Bone Joint Surg Br. 1997 Sep;79(5):857-65. PubMed PMID: 9331050.

Robertson DD, Yuan J, Bigliani LU, Flatow EL, Yamaguchi K. Three-dimensional analysis
of the proximal part of the humerus: relevance to arthroplasty. J Bone Joint Surg Am. 2000
Nov;82-A(11):1594-602. PubMed PMID: 11097450.

© 2016 American Academy of Orthopaedic Surgeons 2016 Orthopaedic In-Training Examination


SECTION 7: Shoulder and Elbow • 171

Video 101 (scan to view)

Question 101
A 45-year-old woman underwent arthroscopic capsular release for recalcitrant adhesive capsulitis. Release
of the tissue using cautery as shown in Video 101 results primarily in increased

1. external rotation with the shoulder abducted 90 degrees.


2. terminal internal rotation of the humeral head.
3. inferior translation of the humeral head in neutral rotation.
4. posterior translation of the shoulder forward flexed and internally rotated.
5. anterior translation with the shoulder abducted 45 degrees.

PREFERRED RESPONSE: 3

RECOMMENDED READINGS
Tetro AM, Bauer G, Hollstien SB, Yamaguchi K. Arthroscopic release of the rotator interval and
coracohumeral ligament: An anatomic study in cadavers. Arthroscopy. 2002 Feb;18(2):145-50. PubMed
PMID: 11830807.

Jost B, Koch PP, Gerber C. Anatomy and functional aspects of the rotator interval. J Shoulder Elbow Surg.
2000 Jul-Aug;9(4):336-41. PubMed PMID: 10979532.

© 2016 American Academy of Orthopaedic Surgeons 2016 Orthopaedic In-Training Examination


172 • American Academy of Orthopaedic Surgeons

Figure 116a Figure 116b

Question 116
Figures 116a and 116b are the radiographs of a 71-year-old man who underwent uncomplicated right
total shoulder arthroplasty for glenohumeral osteoarthritis 1 year ago. He now reports the sudden onset
of severe pain in his right shoulder over the weekend and says that he feels somewhat “feverish.” He
cannot move his arm without a substantial increase in pain, and he is having trouble sleeping and with
activities of daily living. He denies nausea, vomiting, or specific trauma to his shoulder and is worried
that his shoulder is infected (he also reports attending a recent family reunion and getting food poisoning).
Radiograph findings are negative, and laboratory values include a normal white blood count, C-reactive
protein of 1.2 mg/L (rr, 0.08-3.1 mg/L), and erythrocyte sedimentation rate of 17 mm/h (rr, 0-20 mm/h).
Treatment will likely involve

1. reassurance and observation.


2. diagnostic arthroscopy with lavage and 48 hours of intravenous antibiotics.
3. immediate aspiration with a fluid culture.
4. open reduction and internal fixation (ORIF).
5. revision shoulder arthroplasty.

PREFERRED RESPONSE: 5

RECOMMENDED READINGS
Pinkas D, Wiater B, Wiater JM. The Glenoid Component in Anatomic Shoulder Arthroplasty. J Am Acad
Orthop Surg. 2015 May;23(5):317-326. Epub 2015 Mar 31. Review. PubMed PMID: 25829449.

Papadonikolakis A, Neradilek MB, Matsen FA 3rd. Failure of the glenoid component in anatomic total
shoulder arthroplasty: a systematic review of the English-language literature between 2006 and 2012. J
Bone Joint Surg Am. 2013 Dec 18;95(24):2205-12. doi: 10.2106/JBJS.L.00552. Review. PubMed PMID:
24352774.

© 2016 American Academy of Orthopaedic Surgeons 2016 Orthopaedic In-Training Examination


SECTION 7: Shoulder and Elbow • 173

Figure 131a Figure 131b

Figure 131c Figure 131d

Question 131
Figure 131a is the radiograph of a 60-year-old right-hand-dominant woman who has persistent right
shoulder pain. She has received several cortisone injections to the shoulder, which helped temporarily.
Physical therapy has exacerbated her pain. An examination reveals 120 degrees of active forward
elevation with scapulothoracic substitution and 15 degrees of external rotation. Her strength in elevation is
4/5, and external rotation strength is 5/5. MR images are shown in Figures 131b through 131d. What is the
most appropriate surgical intervention?

1. Arthroscopic capsular release with debridement of the glenohumeral joint


2. Arthroscopic rotator cuff repair with biceps tenotomy
3. Hemiarthroplasty with an enlarged prosthetic humeral head
4. Unconstrained total shoulder arthroplasty (TSA) with rotator cuff repair
5. Reverse (TSA)

PREFERRED RESPONSE: 5

© 2016 American Academy of Orthopaedic Surgeons 2016 Orthopaedic In-Training Examination


174 • American Academy of Orthopaedic Surgeons

RECOMMENDED READINGS
Izquierdo R, Voloshin I, Edwards S, Freehill MQ, Stanwood W, Wiater JM, Watters WC 3rd, Goldberg
MJ, Keith M, Turkelson CM, Wies JL, Anderson S, Boyer K, Raymond L, Sluka P; American Academy
of Orthopedic Surgeons. Treatment of glenohumeral osteoarthritis. J Am Acad Orthop Surg. 2010
Jun;18(6):375-82. PubMed PMID: 20511443.

Edwards TB, Boulahia A, Kempf JF, Boileau P, Nemoz C, Walch G. The influence of rotator cuff disease
on the results of shoulder arthroplasty for primary osteoarthritis: results of a multicenter study. J Bone
Joint Surg Am. 2002 Dec;84-A(12):2240-8. PubMed PMID: 12473715.

Question 149
A 46-year-old woman sustained a simple elbow dislocation with no fracture after a fall. After undergoing
closed reduction, she has substantial pain and does not want to move her arm because it “hurts too much.”
Together, you and the patient agree to pursue a course of directed physical therapy. What is the best
recommendation for the therapist regarding her rehabilitation protocol?

1. Use aggressive passive stretching into extension to prevent flexion contracture from
prolonged immobilization.
2. Encourage early, gentle active range of motion to improve stabilizing compressive forces
across the joint.
3. Encourage early strengthening of the flexor wad musculature to prevent atrophy and
recurrent instability.
4. Allow only passive range of motion by the therapist to decrease risk for recurrent posterior
dislocation of the joint through active firing of the triceps.
5. After a 4-week period of immobilization to allow for bony healing, allow both passive
and active range of motion in a hinged brace to protect the joint followed by an early
progression to strengthening.

PREFERRED RESPONSE: 2

RECOMMENDED READINGS
Seiber K, Gupta R, McGarry MH, Safran MR, Lee TQ. The role of the elbow musculature, forearm
rotation, and elbow flexion in elbow stability: an in vitro study. J Shoulder Elbow Surg. 2009 Mar-
Apr;18(2):260-8. doi: 10.1016/j.jse.2008.08.004. Epub 2008 Nov 30. PubMed PMID: 19046641.

Wyrick JD, Dailey SK, Gunzenhaeuser JM, Casstevens EC. Management of Complex Elbow Dislocations:
A Mechanistic Approach. J Am Acad Orthop Surg. 2015 May;23(5):297-306. Review. PubMed PMID:
25911662.

© 2016 American Academy of Orthopaedic Surgeons 2016 Orthopaedic In-Training Examination


SECTION 7: Shoulder and Elbow • 175

Question 167
An 82-year-old woman underwent an uncomplicated reverse total shoulder arthroplasty performed under
general anesthesia. Her history includes diabetes and hypertension, both controlled by oral medication, as
well as renal insufficiency. On the second postsurgical day, she is conversant when evaluated on rounds,
but she appears more tired than she was at baseline. She has been prescribed 2.5 mg of oxycodone every
4 hours as needed. A review of her visual analog scores since surgery reveal a range between 9/10 and
10/10, and her vital signs include tachycardia in the 90s. Her complete blood count findings are similar to
her baseline values. The most appropriate next step is to

1. reassure the patient and provide follow-up later that afternoon.


2. proceed with discharge and expedite outpatient orthopaedic follow-up.
3. perform repeat shoulder radiographs.
4. consult with the geriatric service for medical co-management.
5. initiate intravenous meperidine patient-controlled analgesia.

PREFERRED RESPONSE: 4

RECOMMENDED READINGS
American Geriatrics Society Expert Panel on Postoperative Delirium in Older Adults. Postoperative
delirium in older adults: best practice statement from the American Geriatrics Society. J Am Coll Surg.
2015 Feb;220(2):136-48.e1. doi: 10.1016/j.jamcollsurg.2014.10.019. Epub 2014 Nov 14. PubMed PMID:
25535170.

Mendelson DA, Friedman SM. Principles of comanagement and the geriatric fracture center. Clin Geriatr
Med. 2014 May;30(2):183-9. doi: 10.1016/j.cger.2014.01.016. Epub 2014 Mar 14. Review. PubMed
PMID: 24721359.

Friedman SM, Mendelson DA, Bingham KW, Kates SL. Impact of a comanaged Geriatric Fracture
Center on short-term hip fracture outcomes. Arch Intern Med. 2009 Oct 12;169(18):1712-7. doi: 10.1001/
archinternmed.2009.321. PubMed PMID: 19822829.

© 2016 American Academy of Orthopaedic Surgeons 2016 Orthopaedic In-Training Examination


176 • American Academy of Orthopaedic Surgeons

Figure 178a Figure 178b

Question 178
A morbidly obese 63-year-old man sustained a fall and fractured his right shoulder and right ankle.
His history included diabetes, hypertension, coronary artery disease, peripheral vascular disease, and
gastroesophageal reflux. His medications include verapamil, insulin, metformin, hydrochlorothiazide, and
omeprazole. Simultaneous open reduction and internal fixation (ORIF) of the ankle in the supine position
and ORIF with bone grafting of the shoulder in the beach-chair position (BCP) under general anesthesia
with supplemental regional blocks were performed as shown in Figures 178a and 178b. After surgery,
he has a sense of severe burning and tingling in his right leg. An examination reveals diminished light-
touch sensation in the proximal lateral right thigh but full strength with challenge of hip flexion and knee
extension. What is the most likely cause of his symptoms?

1. Undiagnosed pelvic fracture resulting from the initial fall


2. Neurologic complication from the lower extremity regional block
3. Failure to appropriately cushion the lateral abdominal support post during positioning
4. Exacerbation of preexisting diabetic peripheral neuropathy
5. Interaction of halothane (used for general anesthesia) with verapamil and hydrochlorothiazide

PREFERRED RESPONSE: 3

RECOMMENDED READINGS
Guss D, Bhattacharyya T. Perioperative management of the obese orthopaedic patient. J Am Acad Orthop
Surg. 2006 Jul;14(7):425-32. Review. PubMed PMID: 16822890.

Moen TC, Rudolph GH, Caswell K, Espinoza C, Burkhead WZ Jr, Krishnan SG. Complications of
shoulder arthroscopy. J Am Acad Orthop Surg. 2014 Jul;22(7):410-9. doi: 10.5435/JAAOS-22-07-410.
Review. PubMed PMID: 24966247.

© 2016 American Academy of Orthopaedic Surgeons 2016 Orthopaedic In-Training Examination


SECTION 7: Shoulder and Elbow • 177

Figure 185a Figure 185b

Question 185
A 19-year-old man sustained an anterior shoulder dislocation during a wrestling match and undergoes
surgical repair of the injury. At arthroscopy, you encounter the finding shown in Figure 185a and perform
the repair shown in Figure 185b. These images are taken from the posterior portal with the patient in the
lateral decubitus position. Failure to completely evaluate the extent of damage to the structure marked as 1
will result in

1. recurrent anterior instability with repair failure.


2. excess tightening and stiffness.
3. subluxation of the long head of the biceps.
4. subscapularis insufficiency.
5. symptomatic posterior subluxation when lifting weight.

PREFERRED RESPONSE: 1

RECOMMENDED READINGS
Burkhart SS, De Beer JF. Traumatic glenohumeral bone defects and their relationship to failure of
arthroscopic Bankart repairs: significance of the inverted-pear glenoid and the humeral engaging Hill-
Sachs lesion. Arthroscopy. 2000 Oct;16(7):677-94. PubMed PMID: 11027751.

Bushnell BD, Creighton RA, Herring MM. Bony instability of the shoulder. Arthroscopy. 2008
Sep;24(9):1061-73. doi: 10.1016/j.arthro.2008.05.015. Epub 2008 Jun 30. Review. PubMed PMID:
18760215.

© 2016 American Academy of Orthopaedic Surgeons 2016 Orthopaedic In-Training Examination


178 • American Academy of Orthopaedic Surgeons

Figure 202a Figure 202b

Question 202
Figures 202a and 202b are the anteroposterior and lateral radiographs of an obese 58-year-old woman who
sustained an elbow injury after a fall. What is the best course of treatment for the radial head component
of this injury?

1. Open reduction and internal fixation (ORIF)


2. Open fragment excision and lateral ligament repair
3. Radial head arthroplasty
4. Radial head excision
5. Arthroscopic fragment excision

PREFERRED RESPONSE: 3

RECOMMENDED READINGS
Acevedo DC, Paxton ES, Kukelyansky I, Abboud J, Ramsey M. Radial Head Arthroplasty: State of the
Art. J Am Acad Orthop Surg. 2014 Oct;22(10):633-642. Review. PubMed PMID: 25281258.

Beingessner DM, Dunning CE, Gordon KD, Johnson JA, King GJ. The effect of radial head excision and
arthroplasty on elbow kinematics and stability. J Bone Joint Surg Am. 2004 Aug;86-A(8):1730-9. PubMed
PMID: 15292422.

© 2016 American Academy of Orthopaedic Surgeons 2016 Orthopaedic In-Training Examination


SECTION 7: Shoulder and Elbow • 179

Figure 213a Figure 213b Figure 213c

Figure 213d Figure 213e Figure 213f

Question 213
Figures 213a through 213e are the MR images of a 42-year-old man who underwent right shoulder
arthroscopy for recalcitrant shoulder pain 6 months ago. He received 2 subacromial cortisone injections
that provided temporary pain relief and underwent physical therapy. While arthroscopically viewing from
the posterior portal, the material shown in Video 213f was encountered. What is the most appropriate next
step?

1. Arthroscopic debridement alone with a specimen for culture and sensitivity


2. Arthroscopic debridement alone with administration of intravenous colchicine
3. Arthroscopic debridement of the lesion with concomitant rotator cuff repair
4. Conversion to open excision of the lesion with drain placement
5. Immediate wound closure and referral to orthopaedic oncology

PREFERRED RESPONSE: 3

© 2016 American Academy of Orthopaedic Surgeons 2016 Orthopaedic In-Training Examination


180 • American Academy of Orthopaedic Surgeons

RECOMMENDED READINGS
Suzuki K, Potts A, Anakwenze O, Singh A. Calcific Tendinitis of the Rotator Cuff: Management Options.
J Am Acad Orthop Surg. 2014 Nov;22(11):707-717. Review. PubMed PMID: 25344596.

El Shewy MT. Arthroscopic removal of calcium deposits of the rotator cuff: a 7-year follow-up. Am
J Sports Med. 2011 Jun;39(6):1302-5. doi: 10.1177/0363546510396320. Epub 2011 Feb 24. PubMed
PMID: 21350066.

Figure 221a Figure 221b

Question 221
Figures 221a and 221b are the current radiographs of a 43-year-old right-hand-dominant man who
has severe left shoulder pain 2 years after undergoing left shoulder hemiarthroplasty with biological
resurfacing of the glenoid. An examination reveals 130 degrees of painful elevation and 30 degrees of
external rotation. Elevation and external rotation strength are both 5/5. The best next step is

1. intra-articular cortisone injection with manipulation under anesthesia.


2. arthroscopic capsular release and lysis of adhesions.
3. revision shoulder hemiarthroplasty with Achilles allograft resurfacing.
4. revision to unconstrained total shoulder arthroplasty (TSA).
5. revision to reverse TSA.

PREFERRED RESPONSE: 4

© 2016 American Academy of Orthopaedic Surgeons 2016 Orthopaedic In-Training Examination


SECTION 7: Shoulder and Elbow • 181

RECOMMENDED READINGS
Lee BK, Vaishnav S, Rick Hatch GF 3rd, Itamura JM. Biologic resurfacing of the glenoid with meniscal
allograft: long-term results with minimum 2-year follow-up. J Shoulder Elbow Surg. 2013 Feb;22(2):253-
60. doi: 10.1016/j.jse.2012.04.019. Epub 2012 Aug 25. PubMed PMID: 22929583.

Hammond LC, Lin EC, Harwood DP, Juhan TW, Gochanour E, Klosterman EL, Cole BJ, Nicholson GP,
Verma NN, Romeo AA. Clinical outcomes of hemiarthroplasty and biological resurfacing in patients aged
younger than 50 years. J Shoulder Elbow Surg. 2013 Oct;22(10):1345-51. doi: 10.1016/j.jse.2013.04.015.
Epub 2013 Jun 22. PubMed PMID: 23796385.

Bois AJ, Whitney IJ, Somerson JS, Wirth MA. Humeral Head Arthroplasty and Meniscal Allograft
Resurfacing of the Glenoid: A Concise Follow-up of a Previous Report and Survivorship Analysis. J Bone
Joint Surg Am. 2015 Oct 7;97(19):1571-7. doi: 10.2106/JBJS.N.01079. PubMed PMID: 26446964.

© 2016 American Academy of Orthopaedic Surgeons 2016 Orthopaedic In-Training Examination


182 • American Academy of Orthopaedic Surgeons

Figure 234a Figure 234b

Figure 234c Figure 234d

Question 234
Figures 234a through 234d are the radiographs and CT scans of an 86-year-old woman who fell and
sustained a left elbow fracture. She has considerable pain in her elbow, an inability to flex or extend her
elbow, and numbness and tingling in the ring and small fingers of her left hand. She lives in an assisted
living facility and reports no problems with her elbow before the fall. Her history includes mild chronic
lung disease, hypertension, diabetes, hyperlipidemia, peripheral neuropathy, gout, cardiomyopathy, renal
failure, and a heart attack. What is the best treatment option to maximize function?

1. Observation with early mobilization


2. A 6- to 8-week casting period to allow the fracture to heal, followed by isolated ulnar nerve
transposition if the patient is still symptomatic
3. Open reduction and internal fixation (ORIF) with ulnar nerve transposition
4. Radial head excision with lateral ligamentous reconstruction and ulnar nerve transposition
5. Total elbow arthroplasty (TEA) with ulnar nerve transposition

PREFERRED RESPONSE: 5

© 2016 American Academy of Orthopaedic Surgeons 2016 Orthopaedic In-Training Examination


SECTION 7: Shoulder and Elbow • 183

RECOMMENDED READINGS
Choo A, Ramsey ML. Total elbow arthroplasty: current options. J Am Acad Orthop Surg. 2013
Jul;21(7):427-37. doi: 10.5435/JAAOS-21-07-427. Review. PubMed PMID: 23818030.

Galano GJ, Ahmad CS, Levine WN. Current treatment strategies for bicolumnar distal humerus fractures.
J Am Acad Orthop Surg. 2010 Jan;18(1):20-30. Review. PubMed PMID: 20044489.

Question 253
During an anterior approach to the shoulder for a reverse total shoulder arthroplasty (TSA) with a
concomitant latissimus dorsi/teres major transfer, retractors are placed along the superficial surface of the
latissimus dorsi. Which nerve is most at risk during exposure?

1. Musculocutaneous
2. Axillary
3. Radial
4. Median
5. Ulnar

PREFERRED RESPONSE: 3

RECOMMENDED READINGS
Pearle AD, Kelly BT, Voos JE, Chehab EL, Warren RF. Surgical technique and anatomic study of
latissimus dorsi and teres major transfers. J Bone Joint Surg Am. 2006 Jul;88(7):1524-31. PubMed PMID:
16818978.

Morelli M, Nagamori J, Gilbart M, Miniaci A. Latissimus dorsi tendon transfer for massive irreparable
cuff tears: an anatomic study. J Shoulder Elbow Surg. 2008 Jan-Feb;17(1):139-43. PubMed PMID:
18069017.

© 2016 American Academy of Orthopaedic Surgeons 2016 Orthopaedic In-Training Examination


184 • American Academy of Orthopaedic Surgeons

Figure 272a Figure 272b

Figure 272c Figure 272d Figure 272e

Question 272
Figures 272a through 272e are the clinical photograph, radiograph, and MR images of a 40-year-old right-
hand-dominant man who has severe pain in his right shoulder that began acutely 3 days ago while he was
arm wrestling. He is an avid weight trainer and has a history of an open instability repair on the left and
an arthroscopic instability repair on the right. He has been taking anti-inflammatory drugs, experiencing
minimal pain relief. An examination reveals his active and passive motion above shoulder level is
markedly restricted by pain, and rotator cuff strength testing is not tolerated. The most appropriate next
step is

1. referral to pain management with continued sling use for 6 weeks.


2. referral to physical therapy for immediate shoulder mobilization.
3. arthroscopic subscapularis tendon repair.
4. open repair of the sternal head of the pectoralis major.
5. subpectoral long head biceps tenodesis.

PREFERRED RESPONSE: 4

© 2016 American Academy of Orthopaedic Surgeons 2016 Orthopaedic In-Training Examination


SECTION 7: Shoulder and Elbow • 185

RECOMMENDED READINGS
Petilon J, Carr DR, Sekiya JK, Unger DV. Pectoralis major muscle injuries: evaluation and management. J
Am Acad Orthop Surg. 2005 Jan-Feb;13(1):59-68. Review. PubMed PMID: 15712983.

de Castro Pochini A, Andreoli CV, Belangero PS, Figueiredo EA, Terra BB, Cohen C, Andrade
Mdos S, Cohen M, Ejnisman B. Clinical considerations for the surgical treatment of pectoralis major
muscle ruptures based on 60 cases: a prospective study and literature review. Am J Sports Med. 2014
Jan;42(1):95-102. doi: 10.1177/0363546513506556. Epub 2013 Nov 5. Review. PubMed PMID:
24192390.

Lee J, Brookenthal KR, Ramsey ML, Kneeland JB, Herzog R. MR imaging assessment of the pectoralis
major myotendinous unit: an MR imaging-anatomic correlative study with surgical correlation. AJR Am J
Roentgenol. 2000 May;174(5):1371-5. PubMed PMID: 10789797.

© 2016 American Academy of Orthopaedic Surgeons 2016 Orthopaedic In-Training Examination


186 • American Academy of Orthopaedic Surgeons

SECTION 8: Spine

Video 7 (scan to view)

Question 7
In a well-designed prospective randomized study that provides level II evidence in favor of surgical
treatment for the injury shown in Video 7, the primary outcome measure is the neck disability index (NDI)
score, which shows an improvement of 5.5 points on a scale of 100 for patients treated with surgery (vs
those who received nonsurgical treatment). This difference is statistically significant with a P value of
0.0001. The magnitude of difference and the P value suggests this difference

1. is highly significant and should change treatment patterns in favor of surgery.


2. is irrelevant because the study has only level II evidence.
3. may or may not be a clinically important difference.
4. does not reflect the number of patients enrolled in the study.
5. has been miscalculated.

PREFERRED RESPONSE: 3

RECOMMENDED READINGS
Vaccaro AR, Kepler CK, Kopjar B, Chapman J, Shaffrey C, Arnold P, Gokaslan Z, Brodke D, France J,
Dekutoski M, Sasso R, Yoon ST, Bono C, Harrop J, Fehlings MG. Functional and quality-of-life outcomes
in geriatric patients with type-II dens fracture. J Bone Joint Surg Am. 2013 Apr 17;95(8):729-35. doi:
10.2106/JBJS.K.01636. PubMed PMID: 23595072.

Young BA, Walker MJ, Strunce JB, Boyles RE, Whitman JM, Childs JD. Responsiveness of the Neck
Disability Index in patients with mechanical neck disorders. Spine J. 2009 Oct;9(10):802-8. doi: 10.1016/j.
spinee.2009.06.002. Epub 2009 Jul 25. PubMed PMID: 19632904.

Gatchel RJ, Lurie JD, Mayer TG. Minimal clinically important difference. Spine (Phila Pa 1976). 2010
Sep 1;35(19):1739-43. doi: 10.1097/BRS.0b013e3181d3cfc9. PubMed PMID: 20700084

© 2016 American Academy of Orthopaedic Surgeons 2016 Orthopaedic In-Training Examination


SECTION 8: Spine • 187

Figure 19

Question 19
Figure 19 is the sagittal CT scan of a 57-year-old woman with symptomatic cervical myelopathy. The
single factor favoring an anterior approach is

1. a congenitally narrow spinal canal.


2. multiple levels of pathology.
3. kyphotic spinal alignment.
4. degenerative disk disease.
5. cervical spondylolisthesis.

PREFERRED RESPONSE: 3

RECOMMENDED READINGS
Lebl DR, Bono CM. Update on the Diagnosis and Management of Cervical Spondylotic Myelopathy. J
Am Acad Orthop Surg. 2015 Nov;23(11):648-60. doi: 10.5435/JAAOS-D-14-00250. Review. PubMed
PMID: 26498584.

Emery SE. Cervical spondylotic myelopathy: diagnosis and treatment. J Am Acad Orthop Surg. 2001
Nov-Dec;9(6):376-88. Review. PubMed PMID: 11767723.

© 2016 American Academy of Orthopaedic Surgeons 2016 Orthopaedic In-Training Examination


188 • American Academy of Orthopaedic Surgeons

Video 27 (scan to view)

Question 27
Video 27 is the CT scan of a neurologically intact patient who refuses surgical treatment for the injury
shown, instead opting for treatment with a brace. Evidence-based medicine suggests that if this injury is
treated with a brace rather than surgery, the result will be

1. painful kyphosis, which will necessitate future surgery.


2. better in the long term.
3. better sagittal alignment.
4. quicker recovery.
5. an outcome comparable to surgery.

PREFERRED RESPONSE: 5

RECOMMENDED READINGS
Bailey CS, Urquhart JC, Dvorak MF, Nadeau M, Boyd MC, Thomas KC, Kwon BK, Gurr KR,
Bailey SI, Fisher CG. Orthosis versus no orthosis for the treatment of thoracolumbar burst fractures
without neurologic injury: a multicenter prospective randomized equivalence trial. Spine J. 2014 Nov
1;14(11):2557-64. doi: 10.1016/j.spinee.2013.10.017. Epub 2013 Oct 31. PubMed PMID: 24184649.

Alcalá-Cerra G, Paternina-Caicedo AJ, Díaz-Becerra C, Moscote-Salazar LR, Fernandes-Joaquim A.


Orthosis for thoracolumbar burst fractures without neurologic deficit: A systematic review of prospective
randomized controlled trials. J Craniovertebr Junction Spine. 2014 Jan;5(1):25-32. doi: 10.4103/0974-
8237.135213. PubMed PMID: 25013344.

© 2016 American Academy of Orthopaedic Surgeons 2016 Orthopaedic In-Training Examination


SECTION 8: Spine • 189

Figure 35

Question 35
A lateral cervical radiograph is shown in Figure 35. Which abnormal laboratory value usually is associated
with these findings?

1. Rheumatoid factor (RF) or anti-citrullinated peptide/protein antibody (anti-CCP)


2. Human leukocyte antigen B27 (HLA-B27)
3. C-reactive protein (CRP)
4. Trisomy 21
5. Hemoglobin A1c (HgbA1c)

PREFERRED RESPONSE: 5

RECOMMENDED READINGS
Belanger TA, Rowe DE. Diffuse idiopathic skeletal hyperostosis: musculoskeletal manifestations. J Am
Acad Orthop Surg. 2001 Jul-Aug;9(4):258-67. Review. PubMed PMID: 11476536.

Smith LL, Burnet SP, McNeil JD. Musculoskeletal manifestations of diabetes mellitus. Br J Sports Med.
2003 Feb;37(1):30-5. Review. PubMed PMID: 12547740.

Denko CW, Malemud CJ. Body mass index and blood glucose: correlations with serum insulin, growth
hormone, and insulin-like growth factor-1 levels in patients with diffuse idiopathic skeletal hyperostosis
(DISH). Rheumatol Int. 2006 Feb;26(4):292-7. Epub 2005 Feb 10. PubMed PMID: 15703952.

© 2016 American Academy of Orthopaedic Surgeons 2016 Orthopaedic In-Training Examination


190 • American Academy of Orthopaedic Surgeons

Figure 45

Question 45
Figure 45 is the MR image of a 79-year-old woman with hypertension who has a 3-month history of
bilateral buttock pain while walking more than 200 feet. An evaluation should include

1. examination of peripheral pulses.


2. electromyography and nerve conduction studies.
3. peripheral muscle biopsy.
4. a serum coagulopathy profile.
5. a CT myelogram.

PREFERRED RESPONSE: 1

RECOMMENDED READINGS
Grimm BD, Blessinger BJ, Darden BV, Brigham CD, Kneisl JS, Laxer EB. Mimickers of lumbar
radiculopathy. J Am Acad Orthop Surg. 2015 Jan;23(1):7-17. doi: 10.5435/JAAOS-23-01-7. PubMed
PMID: 25538126.

Issack PS, Cunningham ME, Pumberger M, Hughes AP, Cammisa FP Jr. Degenerative lumbar spinal
stenosis: evaluation and management. J Am Acad Orthop Surg. 2012 Aug;20(8):527-35. doi: 10.5435/
JAAOS-20-08-527. Review. PubMed PMID: 22855855.

© 2016 American Academy of Orthopaedic Surgeons 2016 Orthopaedic In-Training Examination


SECTION 8: Spine • 191

Question 55
The use of recombinant human bone morphogenetic protein-2 (rhBMP-2) in spine fusions greatly
decreased after which event?

1. Published studies on 100% spine fusion with rhBMP-2 were redacted following an
investigation by the United States Department of Justice
2. Iliac crest bone graft (ICBG) was shown to be better than rhBMP-2
3. The Affordable Care Act (ACA) was enacted
4. The Sunshine Act was enacted
5. The original U.S. Food and Drug Administration (FDA) studies were subject to closer scrutiny

PREFERRED RESPONSE: 5

RECOMMENDED READINGS
Laine C, Guallar E, Mulrow C, Taichman DB, Cornell JE, Cotton D, Griswold ME, Localio AR, Meibohm
AR, Stack CB, Williams SV, Goodman SN. Closing in on the truth about recombinant human bone
morphogenetic protein-2: evidence synthesis, data sharing, peer review, and reproducible research. Ann
Intern Med. 2013 Jun 18;158(12):916-8. doi: 10.7326/0003-4819-158-12-201306180-00012. PubMed
PMID: 23778911.

Carragee EJ, Hurwitz EL, Weiner BK. A critical review of recombinant human bone morphogenetic
protein-2 trials in spinal surgery: emerging safety concerns and lessons learned. Spine J. 2011
Jun;11(6):471-91. doi: 10.1016/j.spinee.2011.04.023. Review. PubMed PMID: 21729796.

Martin BI, Lurie JD, Tosteson AN, Deyo RA, Farrokhi FR, Mirza SK. Use of bone morphogenetic
protein among patients undergoing fusion for degenerative diagnoses in the United States, 2002 to 2012.
Spine J. 2015 Apr 1;15(4):692-9. doi: 10.1016/j.spinee.2014.12.010. Epub 2014 Dec 15. PubMed PMID:
25523380.

Question 68
For patients with rheumatoid arthritis who are undergoing surgery for symptomatic cervical myelopathy,
neurologic recovery after decompression is best predicted by presurgical

1. basilar invagination less than 0.3 cm.


2. subaxial subluxation less than 3.5 mm.
3. atlanto-dens interval less than 2 mm.
4. posterior atlanto-dens interval exceeding 13 mm.
5. rotatory subluxation less than 10 degrees.

PREFERRED RESPONSE: 4

© 2016 American Academy of Orthopaedic Surgeons 2016 Orthopaedic In-Training Examination


192 • American Academy of Orthopaedic Surgeons

RECOMMENDED READINGS
Kim DH, Hilibrand AS. Rheumatoid arthritis in the cervical spine. J Am Acad Orthop Surg. 2005
Nov;13(7):463-74. Review. PubMed PMID: 16272271.

Garfin SR. Rheumatoid Arthritis of Cervical Spine Overview of Rheumatoid Spondylitis. Updated Dec 6,
2015. http://emedicine.medscape.com/article/1266195-overview (Accessed June 1, 2016).

© 2016 American Academy of Orthopaedic Surgeons 2016 Orthopaedic In-Training Examination


SECTION 8: Spine • 193

Video 77 (scan to view)

Question 77
Video 77 is the CT scan of a patient who fell and hit his head. He does not have any upper or lower
extremity weakness or numbness. He is ambulatory and has normal bowel and bladder function. Definitive
treatment should include

1. a rigid cervical collar.


2. single-level anterior cervical diskectomy and fusion.
3. combined anterior and posterior cervical surgery with a pedicle subtraction osteotomy.
4. posterior cervical instrumentation and fusion.
5. cervical disk arthroplasty to preserve motion and prevent adjacent segment degeneration.

PREFERRED RESPONSE: 4

RECOMMENDED READINGS
Robinson Y, Robinson AL, Olerud C. Complications and survival after long posterior instrumentation
of cervical and cervicothoracic fractures related to ankylosing spondylitis or diffuse idiopathic skeletal
hyperostosis. Spine (Phila Pa 1976). 2015 Feb 15;40(4):E227-33. doi: 10.1097/BRS.0000000000000726.
PubMed PMID: 25494322.

Mathews M, Bolesta MJ. Treatment of spinal fractures in ankylosing spondylitis. Orthopedics. 2013
Sep;36(9):e1203-8. doi: 10.3928/01477447-20130821-25. PubMed PMID: 24025014.

Chaudhary SB, Hullinger H, Vives MJ. Management of acute spinal fractures in ankylosing spondylitis.
ISRN Rheumatol. 2011;2011:150484. doi: 10.5402/2011/150484. Epub 2011 Jun 30. PubMed PMID:
22389792.

Caron T, Bransford R, Nguyen Q, Agel J, Chapman J, Bellabarba C. Spine fractures in patients with
ankylosing spinal disorders. Spine (Phila Pa 1976). 2010 May 15;35(11):E458-64. doi: 10.1097/
BRS.0b013e3181cc764f. PubMed PMID: 20421858.

© 2016 American Academy of Orthopaedic Surgeons 2016 Orthopaedic In-Training Examination


194 • American Academy of Orthopaedic Surgeons

Question 94
A patient with a smoking history has pain in her legs after walking. You are trying to distinguish a vascular
etiology from a neurogenic etiology. Which response is associated with a vascular etiology?

1. When walking, the pain will improve if she is leaning over a grocery cart
2. Sitting down is necessary to relieve the pain
3. Riding a bicycle does not cause any claudication
4. Standing still relieves the pain
5. Washing dishes at the sink aggravates the pain

PREFERRED RESPONSE: 4

RECOMMENDED READINGS
Katz JN, Harris MB. Clinical practice. Lumbar spinal stenosis. N Engl J Med. 2008 Feb 21;358(8):818-25.
doi: 10.1056/NEJMcp0708097. Review. PubMed PMID: 18287604.

Binder DK, Schmidt MH, Weinstein PR. Lumbar spinal stenosis. Semin Neurol. 2002 Jun;22(2):157-66.
Review. PubMed PMID: 12524561.

© 2016 American Academy of Orthopaedic Surgeons 2016 Orthopaedic In-Training Examination


SECTION 8: Spine • 195

Figure 110a Figure 110b Figure 110c

Question 110
Based on the radiographic findings in Figures 110a through 110c, what is the most likely diagnosis?

1. Diffuse interstitial skeletal hyperostosis (DISH)


2. Ankylosing spondylitis (AS)
3. Down syndrome with C1-C2 instability
4. Rheumatoid arthritis (RA)
5. Osteomyelitis

PREFERRED RESPONSE: 2

RECOMMENDED READINGS
Kubiak EN, Moskovich R, Errico TJ, Di Cesare PE. Orthopaedic management of ankylosing spondylitis. J
Am Acad Orthop Surg. 2005 Jul-Aug;13(4):267-78. PubMed PMID: 16112983.

Haroon N. Ankylosis in ankylosing spondylitis: current concepts. Clin Rheumatol. 2015 Jun;34(6):1003-7.
doi: 10.1007/s10067-015-2956-4. PubMed PMID: 25935456.

Smith JA. Update on ankylosing spondylitis: current concepts in pathogenesis. Curr Allergy Asthma Rep.
2015 Jan;15(1):489. doi: 10.1007/s11882-014-0489-6. Review. PubMed PMID: 25447326.

© 2016 American Academy of Orthopaedic Surgeons 2016 Orthopaedic In-Training Examination


196 • American Academy of Orthopaedic Surgeons

Video 124 (scan to view)

Question 124
A study shows improved 1-month and 1-year survival rates for patients older than age 60 who undergo
surgery for the injury shown in Video 124 vs survival for patients treated without surgery. If the study is
a retrospective review and surgery was performed per surgeon preference, which type of bias is the most
pressing concern?

1. Selection
2. Confirmation
3. Recall
4. Publication
5. Interviewer

PREFERRED RESPONSE: 1

RECOMMENDED READINGS
Chapman J, Smith JS, Kopjar B, Vaccaro AR, Arnold P, Shaffrey CI, Fehlings MG. The AOSpine North
America Geriatric Odontoid Fracture Mortality Study: a retrospective review of mortality outcomes for
operative versus nonoperative treatment of 322 patients with long-term follow-up. Spine (Phila Pa 1976).
2013 Jun 1;38(13):1098-104. doi: 10.1097/BRS.0b013e31828-6f0cf. PubMed PMID: 23354104.

Guyatt G, Rennie D, Meade MO, Cook DJ. Users' Guides to the Medical Literature: A Manual for
Evidence-Based Clinical Practice. 3rd edition. New York, NY: McGraw Hill; 2015.

© 2016 American Academy of Orthopaedic Surgeons 2016 Orthopaedic In-Training Examination


SECTION 8: Spine • 197

Figure 141a Figure 141b

Video 141c (scan to view)

Question 141
Figures 141a through 141c are radiographs and an axial MRI scan. When discussing this patient’s
condition, which statement is most accurate regarding the benefits and limitations associated with both
surgery and continued nonsurgical care?

1. Continued nonsurgical treatment will produce the same long-term results as surgery
2. Fusion is absolutely required when considering surgery
3. Fusion should be done using bone morphogenetic protein (BMP)
4. Surgery will provide better short-term and long-term improvement
5. An instrumented fusion with pedicle screws and rods will provide a better result.

PREFERRED RESPONSE: 4

© 2016 American Academy of Orthopaedic Surgeons 2016 Orthopaedic In-Training Examination


198 • American Academy of Orthopaedic Surgeons

RECOMMENDED READINGS
Herkowitz HN, Kurz LT. Degenerative lumbar spondylolisthesis with spinal stenosis. A prospective study
comparing decompression with decompression and intertransverse process arthrodesis. J Bone Joint Surg
Am. 1991 Jul;73(6):802-8. PubMed PMID: 2071615.

Pearson AM, Lurie JD, Tosteson TD, Zhao W, Abdu WA, Weinstein JN. Who should undergo surgery for
degenerative spondylolisthesis? Treatment effect predictors in SPORT. Spine (Phila Pa 1976). 2013 Oct
1;38(21):1799-811. doi: 10.1097/BRS.0b013e3182a314d0. PubMed PMID: 23846502.

Sigmundsson FG, Jönsson B, Strömqvist B. Outcome of decompression with and without fusion in spinal
stenosis with degenerative spondylolisthesis in relation to preoperative pain pattern: a register study of
1,624 patients. Spine J. 2015 Apr 1;15(4):638-46. doi: 10.1016/j.spinee.2014.11.020. Epub 2014 Nov 29.
PubMed PMID: 25450653.

Kepler CK, Hilibrand AS, Sayadipour A, Koerner JD, Rihn JA, Radcliff KE, Vaccaro AR, Albert TJ,
Anderson DG. Clinical and radiographic degenerative spondylolisthesis (CARDS) classification. Spine
J. 2015 Aug 1;15(8):1804-11. doi: 10.1016/j.spinee.2014.03.045. Epub 2014 Apr 3. PubMed PMID:
24704503.

© 2016 American Academy of Orthopaedic Surgeons 2016 Orthopaedic In-Training Examination


SECTION 8: Spine • 199

Figure 163a Figure 163b

Question 163
Figures 163a and 163b are the MR images of a 45-year-old woman who has neck pain that is radiating into
her right shoulder and arm after 3 weeks of nonsurgical treatment. There is no neurologic deficit. What is
the most appropriate treatment?

1. Anterior cervical diskectomy and fusion


2. Continued nonsurgical treatment
3. Chiropractic manipulation
4. Posterior cervical foraminotomy
5. Cervical epidural injection

PREFERRED RESPONSE: 2

RECOMMENDED READINGS
Ponnappan RK, Khan M, Matzon JL, Sheikh ES, Tucker BS, Pepe MD, Tjoumakaris FP, Nassr AN.
Clinical Differentiation of Upper Extremity Pain Etiologies. J Am Acad Orthop Surg. 2015 Aug;23(8):492-
500. doi: 10.5435/JAAOS-D-11-00086. Epub 2015 Jun 26. PubMed PMID: 26116851.

Levine MJ, Albert TJ, Smith MD. Cervical Radiculopathy: Diagnosis and Nonoperative Management. J
Am Acad Orthop Surg. 1996 Nov;4(6):305-316. PubMed PMID: 10797198.

© 2016 American Academy of Orthopaedic Surgeons 2016 Orthopaedic In-Training Examination


200 • American Academy of Orthopaedic Surgeons

Question 180
Although initial studies revealed no adverse events associated with rhBMP-2 and also demonstrated a
clear benefit to its use when compared with iliac crest bone graft, what is now known about rhBMP-2?

1. It represents the best alternative to iliac crest autograft that is currently available to stimulate
bone growth
2. It is no longer available in the United States; it has been withdrawn because of the
manufacturer’s liability concerns
3. It is associated with substantial cancer risk
4. It is not permitted by the U.S. Food and Drug Administration (FDA) for off-label applications
5. It is always inappropriate for use in orthopaedic surgery.

PREFERRED RESPONSE: 1

RECOMMENDED READINGS
Laine C, Guallar E, Mulrow C, Taichman DB, Cornell JE, Cotton D, Griswold ME, Localio AR, Meibohm
AR, Stack CB, Williams SV, Goodman SN. Closing in on the truth about recombinant human bone
morphogenetic protein-2: evidence synthesis, data sharing, peer review, and reproducible research. Ann
Intern Med. 2013 Jun 18;158(12):916-8. doi: 10.7326/0003-4819-158-12-201306180-00012. PubMed
PMID: 23778911.

Carragee EJ, Hurwitz EL, Weiner BK. A critical review of recombinant human bone morphogenetic
protein-2 trials in spinal surgery: emerging safety concerns and lessons learned. Spine J. 2011
Jun;11(6):471-91. doi: 10.1016/j.spinee.2011.04.023. Review. PubMed PMID: 21729796.

North American Spine Coverage Committee. Recombinant Human bone morphogenic protein 2: defining
appropriate coverage positions. https://www.spine.org/PolicyPractice/CoverageRecommendations/e-
book#p=1 Accessed 10-26-2015.

Martin BI, Lurie JD, Tosteson AN, Deyo RA, Farrokhi FR, Mirza SK. Use of bone morphogenetic
protein among patients undergoing fusion for degenerative diagnoses in the United States, 2002 to 2012.
Spine J. 2015 Apr 1;15(4):692-9. doi: 10.1016/j.spinee.2014.12.010. Epub 2014 Dec 15. PubMed PMID:
25523380.

© 2016 American Academy of Orthopaedic Surgeons 2016 Orthopaedic In-Training Examination


SECTION 8: Spine • 201

Video 194 (scan to view)

Question 194
A 45-year-old patient who was involved in a motor vehicle collision sustained the injury shown in Video
194. What is the most likely mechanism of injury?

1. Distraction
2. Extension
3. Cannot determine without MR imaging
4. Compression
5. Rotational

PREFERRED RESPONSE: 4

RECOMMENDED READINGS
Langrana NA, Harten RD RD, Lin DC, Reiter MF, Lee CK. Acute thoracolumbar burst fractures: a
new view of loading mechanisms. Spine (Phila Pa 1976). 2002 Mar 1;27(5):498-508. PubMed PMID:
11880835.

Denis F. The three column spine and its significance in the classification of acute thoracolumbar spinal
injuries. Spine (Phila Pa 1976). 1983 Nov-Dec;8(8):817-31. PubMed PMID: 6670016.

© 2016 American Academy of Orthopaedic Surgeons 2016 Orthopaedic In-Training Examination


202 • American Academy of Orthopaedic Surgeons

Video 199 (scan to view)

Question 199
Video 199 is the CT scan of a man with an open femur fracture, a subdural hematoma, and a spine injury.
After the orthopaedic trauma team stabilizes the fracture and completes an examination, which finding
most strongly favors spine surgery?

1. Thoracolumbar injury classification system (TLICs) score of 5 or higher


2. Presence of bilateral lower extremity weakness and loss of rectal tone
3. MRI evidence of posterior ligamentous complex disruption
4. Kyphosis angle exceeding 15 degrees
5. Increase of 5 degrees kyphosis on upright radiographs

PREFERRED RESPONSE: 2

RECOMMENDED READINGS
Vaccaro AR, Lehman RA Jr, Hurlbert RJ, Anderson PA, Harris M, Hedlund R, Harrop J, Dvorak M,
Wood K, Fehlings MG, Fisher C, Zeiller SC, Anderson DG, Bono CM, Stock GH, Brown AK, Kuklo
T, Oner FC. A new classification of thoracolumbar injuries: the importance of injury morphology, the
integrity of the posterior ligamentous complex, and neurologic status. Spine (Phila Pa 1976). 2005 Oct
15;30(20):2325-33. PubMed PMID: 16227897.

Joaquim AF, Daubs MD, Lawrence BD, Brodke DS, Cendes F, Tedeschi H, Patel AA. Retrospective
evaluation of the validity of the Thoracolumbar Injury Classification System in 458 consecutively treated
patients. Spine J. 2013 Dec;13(12):1760-5. doi: 10.1016/j.spinee.2013.03.014. Epub 2013 Apr 18.
PubMed PMID: 23602328.

© 2016 American Academy of Orthopaedic Surgeons 2016 Orthopaedic In-Training Examination


SECTION 8: Spine • 203

Question 205
A patient with neurogenic claudication caused by spinal stenosis and spondylolisthesis has not responded
to nonsurgical treatment. She underwent a complication-free surgery that corrected her underlying spine
pathology, but she is unhappy with the outcome. Postsurgical imaging shows a well-healed fusion without
junctional pathology and overall good sagittal alignment. This patient’s dissatisfaction is most likely
related to

1. improved pain and disability scores.


2. smoking.
3. diabetes.
4. depression.
5. hypertension.

PREFERRED RESPONSE: 4

RECOMMENDED READINGS
Adogwa O, Parker SL, Shau DN, Mendenhall SK, Bydon A, Cheng JS, Asher AL, McGirt MJ.
Preoperative Zung depression scale predicts patient satisfaction independent of the extent of improvement
after revision lumbar surgery. Spine J. 2013 May;13(5):501-6. doi: 10.1016/j.spinee.2013.01.017. Epub
2013 Feb 16. PubMed PMID: 23422730.

Fenton JJ, Jerant AF, Bertakis KD, Franks P. The cost of satisfaction: a national study of patient
satisfaction, health care utilization, expenditures, and mortality. Arch Intern Med. 2012 Mar
12;172(5):405-11. doi: 10.1001/archinternmed.2011.1662. Epub 2012 Feb 13. PubMed PMID: 22331982.

© 2016 American Academy of Orthopaedic Surgeons 2016 Orthopaedic In-Training Examination


204 • American Academy of Orthopaedic Surgeons

Video 220 (scan to view)

Question 220
In a well-designed prospective randomized study that provides level II evidence in favor of surgical
treatment for the injury shown in Video 220, the primary outcome measure is the neck disability index
(NDI) score, which shows an improvement of 5.5 points on a scale of 100 for patients treated with surgery
(vs those who received nonsurgical treatment). This difference is statistically significant with a P value of
0.0001. The magnitude of difference and the P value suggests this difference

1. is highly significant and should change treatment patterns in favor of surgery.


2. is irrelevant because the study has only level II evidence.
3. may or may not be a clinically significant difference in outcome.
4. does not reflect the number of patients enrolled in the study.
5. has been miscalculated.

PREFERRED RESPONSE: 1

RECOMMENDED READINGS
Vaccaro AR, Kepler CK, Kopjar B, Chapman J, Shaffrey C, Arnold P, Gokaslan Z, Brodke D, France J,
Dekutoski M, Sasso R, Yoon ST, Bono C, Harrop J, Fehlings MG. Functional and quality-of-life outcomes
in geriatric patients with type-II dens fracture. J Bone Joint Surg Am. 2013 Apr 17;95(8):729-35. doi:
10.2106/JBJS.K.01636. PubMed PMID: 23595072.

Young BA, Walker MJ, Strunce JB, Boyles RE, Whitman JM, Childs JD. Responsiveness of the Neck
Disability Index in patients with mechanical neck disorders. Spine J. 2009 Oct;9(10):802-8. doi: 10.1016/j.
spinee.2009.06.002. Epub 2009 Jul 25. PubMed PMID: 19632904.

Gatchel RJ, Lurie JD, Mayer TG. Minimal clinically important difference. Spine (Phila Pa 1976). 2010
Sep 1;35(19):1739-43. doi: 10.1097/BRS.0b013e3181d3cfc9. PubMed PMID: 20700084.

© 2016 American Academy of Orthopaedic Surgeons 2016 Orthopaedic In-Training Examination


SECTION 8: Spine • 205

Question 233
A 60-year-old man with a history of neck pain sustained an extension injury to his cervical spine after
falling while riding a bicycle. The incident resulted in an incomplete spinal cord injury. An examination
will most likely reveal

1. that his motor deficit is worse in the upper extremities than the lower extremities.
2. that lower extremities are affected more than upper extremities.
3. an ipsilateral deficit in motor function and a contralateral deficit in pain and temperature.
4. a loss of proprioception with preserved motor, pain, and light touch.
5. a complete loss of motor and sensory function below the level of the injury.

PREFERRED RESPONSE: 1

RECOMMENDED READINGS
Gupta R, Bathen ME, Smith JS, Levi AD, Bhatia NN, Steward O. Advances in the management of spinal
cord injury. J Am Acad Orthop Surg. 2010 Apr;18(4):210-22. Review. PubMed PMID: 20357230.

Nowak DD, Lee JK, Gelb DE, Poelstra KA, Ludwig SC. Central cord syndrome. J Am Acad Orthop Surg.
2009 Dec;17(12):756-65. Review. PubMed PMID: 19948700.

Question 249
Two weeks after undergoing laminectomy and fusion for spinal stenosis, a patient is evaluated for a
possible infection. Which laboratory test is most sensitive for identification of postsurgical infection?

1. Erythrocyte sedimentation rate (ESR)


2. C-reactive protein (CRP)
3. White blood cell count (WBC) and differential
4. Hemoglobin A1c (HgbA1c)
5. Serum albumin level

PREFERRED RESPONSE: 2

RECOMMENDED READINGS
Radcliff KE, Neusner AD, Millhouse PW, Harrop JD, Kepler CK, Rasouli MR, Albert TJ, Vaccaro
AR. What is new in the diagnosis and prevention of spine surgical site infections. Spine J. 2015 Feb
1;15(2):336-47. doi: 10.1016/j.spinee.2014.09.022. Epub 2014 Sep 28. Review. PubMed PMID:
25264181.

Kang DG, Holekamp TF, Wagner SC, Lehman RA Jr. Intrasite vancomycin powder for the prevention of
surgical site infection in spine surgery: a systematic literature review. Spine J. 2015 Apr 1;15(4):762-70.
doi: 10.1016/j.spinee.2015.01.030. Epub 2015 Jan 28. Review. PubMed PMID: 25637469.

© 2016 American Academy of Orthopaedic Surgeons 2016 Orthopaedic In-Training Examination


206 • American Academy of Orthopaedic Surgeons

SECTION 9: Sports Medicine

Figure 11a Figure 11b Figure 11c

Question 11
Figures 11a through 11c are the radiograph and MR images of a 10-year-old boy who has had 4 weeks
of lateral knee pain that began while he was playing soccer. The pain is only present with physical
activity, and he does not have any catching symptoms. An examination reveals minimal symptoms with
provocative maneuvers and no effusion. What is the most appropriate initial treatment?

1. Activity modification and decreased weight bearing


2. Arthroscopic removal of the fragment
3. Arthroscopic retrograde drilling
4. Arthroscopic open reduction and internal fixation
5. Observation with no change in athletic activity

PREFERRED RESPONSE: 1

RECOMMENDED READINGS
Crawford DC, Safran MR. Osteochondritis dissecans of the knee. J Am Acad Orthop Surg. 2006
Feb;14(2):90-100. Review. PubMed PMID: 16467184.

Polousky JD. Juvenile osteochondritis dissecans. Sports Med Arthrosc. 2011 Mar;19(1):56-63. doi:
10.1097/JSA.0b013e31820b94b9. Review. PubMed PMID: 21293239.

Carey JL, Grimm NL. Treatment algorithm for osteochondritis dissecans of the knee. Orthop Clin North
Am. 2015 Jan;46(1):141-6. doi: 10.1016/j.ocl.2014.09.010. Review. PubMed PMID: 25435043.


© 2016 American Academy of Orthopaedic Surgeons 2016 Orthopaedic In-Training Examination
SECTION 9: Sports Medicine • 207

Question 23
Which anterior cruciate ligament (ACL) bundle likely is responsible for rotatory stability of the knee?

1. Anteromedial
2. Anterolateral
3. Central
4. Posteromedial
5. Posterolateral

PREFERRED RESPONSE: 5

RECOMMENDED READINGS
Amis AA. The functions of the fibre bundles of the anterior cruciate ligament in anterior drawer, rotational
laxity and the pivot shift. Knee Surg Sports Traumatol Arthrosc. 2012 Apr;20(4):613-20. doi: 10.1007/
s00167-011-1864-7. Epub 2012 Jan 26. Review. PubMed PMID: 22278656.

Duthon VB, Barea C, Abrassart S, Fasel JH, Fritschy D, Ménétrey J. Anatomy of the anterior cruciate
ligament. Knee Surg Sports Traumatol Arthrosc. 2006 Mar;14(3):204-13. Epub 2005 Oct 19. Review.
PubMed PMID: 16235056.

Question 42
Which computer-based neurocognitive test assesses users’ attention, memory, and processing speed?

1. Immediate Post-Concussion Assessment and Cognitive Testing Battery (ImPACT)


2. Sport Concussion Assessment Tool II (SCAT-3)
3. King-Devick (K-D) Test
4. Standardized Assessment of Concussion (SAC)
5. Safe Concussion Outcome Recovery & Education (SCORE) Program

PREFERRED RESPONSE: 1

RECOMMENDED READINGS
Schatz P. Long-term test-retest reliability of baseline cognitive assessments using ImPACT. Am J Sports
Med. 2010 Jan;38(1):47-53. doi: 10.1177/0363546509343805. Epub 2009 Sep 29. PubMed PMID:
19789333.

Okonkwo DO, Tempel ZJ, Maroon J. Sideline assessment tools for the evaluation of concussion in
athletes: a review. Neurosurgery. 2014 Oct;75 Suppl 4:S82-95. doi: 10.1227/NEU.0000000000000493.
Review. PubMed PMID: 25232887.

© 2016 American Academy of Orthopaedic Surgeons 2016 Orthopaedic In-Training Examination


208 • American Academy of Orthopaedic Surgeons

Figure 51a Figure 51b

Question 51
Figures 51a and 51b are the radiographs of a 40-year-old recreational basketball player who injured his
knee while jumping for a rebound. He felt a pop and developed immediate swelling. What is the best next
step?

1. MR imaging
2. Rest, ice, and observation
3. Physical therapy to regain motion
4. Knee arthroscopy and medial patellofemoral ligament (MPFL) repair
5. Open surgical repair

PREFERRED RESPONSE: 5

RECOMMENDED READINGS
Brooks P. Extensor mechanism ruptures. Orthopedics. 2009 Sep;32(9). pii: orthosupersite.com/view.
asp?rID=42852. doi: 10.3928/01477447-20090728-31. Review. PubMed PMID: 19751001.

Matava MJ. Patellar Tendon Ruptures. J Am Acad Orthop Surg. 1996 Nov;4(6):287-296. PubMed PMID:
10797196.

© 2016 American Academy of Orthopaedic Surgeons 2016 Orthopaedic In-Training Examination


SECTION 9: Sports Medicine • 209

Question 62
According to the World Health Organization (WHO), an appropriate surgical “time-out” just prior to an
incision consists of which verifications and components?

1. Patient’s date of birth, planned surgery, team introductions, aspiration risk


2. Patient’s name, surgical laterality, administration of deep venous thrombosis
(DVT) prophylaxis
3. Planned surgery, DVT prophylaxis, postsurgical bracing
4. Team introductions, patient identification, site and procedure, antibiotic prophylaxis
5. Team introductions, patient identification, planned surgery, nurse break schedule

PREFERRED RESPONSE: 4

RECOMMENDED READINGS
Haynes AB, Weiser TG, Berry WR, Lipsitz SR, Breizat AH, Dellinger EP, Herbosa T, Joseph S, Kibatala
PL, Lapitan MC, Merry AF, Moorthy K, Reznick RK, Taylor B, Gawande AA; Safe Surgery Saves Lives
Study Group. A surgical safety checklist to reduce morbidity and mortality in a global population. N Engl
J Med. 2009 Jan 29;360(5):491-9. doi: 10.1056/NEJMsa0810119. Epub 2009 Jan 14. PubMed PMID:
19144931.

Nakano JM: In search of the perfect surgical timeout. AAOS Now February 2014;8.2:34.
http://www.aaos.org/AAOSNow/2014/Feb/managing/managing9/. Accessed 26 May 2016.

WHO Surgical Safety Checklist and Implementation Manual. http://www.who.int/patientsafety/


safesurgery/ss_checklist/en/. Accessed 26 May 2016.

Question 83
A 45-year-old right-handed recreational tennis player has been experiencing pain localized to the medial
side of his elbow for 2 weeks. The pain began after playing in a tournament, but he did not sustain a
specific injury that he can recall. What is the most appropriate treatment?

1. A corticosteroid injection into the flexor origin


2. Physical therapy and evaluation of his tennis swing
3. Arthroscopic debridement of the flexor origin
4. Open debridement of the flexor origin with submuscular ulnar nerve transposition
5. Continued tennis play with use of a hinged elbow brace

PREFERRED RESPONSE: 2

© 2016 American Academy of Orthopaedic Surgeons 2016 Orthopaedic In-Training Examination


210 • American Academy of Orthopaedic Surgeons

RECOMMENDED READINGS
Amin NH, Kumar NS, Schickendantz MS. Medial epicondylitis: evaluation and management. J Am
Acad Orthop Surg. 2015 Jun;23(6):348-55. doi: 10.5435/JAAOS-D-14-00145. Review. PubMed PMID:
26001427.

Cain EL Jr, Dugas JR, Wolf RS, Andrews JR. Elbow injuries in throwing athletes: a current concepts
review. Am J Sports Med. 2003 Jul-Aug;31(4):621-35. Review. PubMed PMID: 12860556.

Question 99
During shoulder arthroscopy performed on a 56-year-old recreational tennis player, the patient’s rotator
cuff appears intact on the articular side, but he has a partial-thickness tear that is approximately 4-mm
thick and 1.5 cm in the anterior-to-posterior direction. What is the appropriate treatment?

1. Conversion to a full-thickness tear and repair with suture anchors


2. Debridement alone
3. Debridement and acromioplasty
4. No further arthroscopic treatment
5. Acromioplasty alone

PREFERRED RESPONSE: 1

RECOMMENDED READINGS
Cordasco FA, Backer M, Craig EV, Klein D, Warren RF. The partial-thickness rotator cuff tear: is
acromioplasty without repair sufficient? Am J Sports Med. 2002 Mar-Apr;30(2):257-60. PubMed PMID:
11912097.

Wolff AB, Sethi P, Sutton KM, Covey AS, Magit DP, Medvecky M. Partial-thickness rotator cuff tears. J
Am Acad Orthop Surg. 2006 Dec;14(13):715-25. PubMed PMID: 17148619.

© 2016 American Academy of Orthopaedic Surgeons 2016 Orthopaedic In-Training Examination


SECTION 9: Sports Medicine • 211

Video 112 (scan to view)

Question 112
An arthroscopic nerve release in which a transverse scapular ligament is transected is seen in Video
112. If the nerve being released is injured during this procedure, which findings can be expected upon
examination?

1. Loss of sensation at the lateral aspect of the upper arm


2. Weakness in abduction with atrophy in the supraspinatus fossa only
3. Weakness in forward flexion with atrophy in the infraspinatus fossa only
4. Weakness in internal rotation with a positive belly-press test result
5. Weakness in external rotation and abduction, with atrophy in the supraspinatus and
infraspinatus fossa

PREFERRED RESPONSE: 5

RECOMMENDED READINGS
Scully WF, Wilson DJ, Parada SA, Arrington ED. Iatrogenic nerve injuries in shoulder surgery. J Am
Acad Orthop Surg. 2013 Dec;21(12):717-26. doi: 10.5435/JAAOS-21-12-717. Review. PubMed PMID:
24292928.

Boykin RE, Friedman DJ, Higgins LD, Warner JJ. Suprascapular neuropathy. J Bone Joint Surg Am. 2010
Oct 6;92(13):2348-64. doi: 10.2106/JBJS.I.01743. Review. PubMed PMID: 20926731.

© 2016 American Academy of Orthopaedic Surgeons 2016 Orthopaedic In-Training Examination


212 • American Academy of Orthopaedic Surgeons

Question 125
A 12-year-old football player was struck on the outside of his left leg, causing a valgus buckling injury to
his knee. He heard a pop and was unable to bear weight following the injury. After performing a thorough
knee examination, what is the best next step?

1. Anteroposterior, lateral, and oblique radiographs of the left knee


2. MR imaging of the left knee
3. Arthrogram of the left knee
4. Ultrasound examination of the left hip
5. Stress radiographs of the left knee

PREFERRED RESPONSE: 1

RECOMMENDED READINGS
Mayer S, Albright JC, Stoneback JW. Pediatric Knee Dislocations and Physeal Fractures About the Knee.
J Am Acad Orthop Surg. 2015 Sep;23(9):571-80. doi: 10.5435/JAAOS-D-14-00242. Epub 2015 Aug 13.
PubMed PMID: 26271758.

Wall EJ, May MM. Growth plate fractures of the distal femur. J Pediatr Orthop. 2012 Jun;32 Suppl 1:S40-
6. doi: 10.1097/BPO.0b013e3182587086. PubMed PMID: 22588102.

© 2016 American Academy of Orthopaedic Surgeons 2016 Orthopaedic In-Training Examination


SECTION 9: Sports Medicine • 213

Video 146 (scan to view)

Question 146
Video 146 depicts a rehabilitation maneuver that is critical in shoulder pathology treatment for a 21-year-
old Minor League baseball pitcher who is treated for shoulder pain with a focused rehabilitation program.
What is the most likely diagnosis?

1. Glenohumeral internal rotation deficit (GIRD)


2. Long head of the biceps tendinosis
3. Subscapularis rupture
4. Superior labral anterior posterior tear
5. Bankart lesion

PREFERRED RESPONSE: 1

RECOMMENDED READINGS
Braun S, Kokmeyer D, Millett PJ. Shoulder injuries in the throwing athlete. J Bone Joint Surg Am. 2009
Apr;91(4):966-78. doi: 10.2106/JBJS.H.01341. Review. PubMed PMID: 19339585.

Crockett HC, Gross LB, Wilk KE, Schwartz ML, Reed J, O'Mara J, Reilly MT, Dugas JR, Meister K,
Lyman S, Andrews JR. Osseous adaptation and range of motion at the glenohumeral joint in professional
baseball pitchers. Am J Sports Med. 2002 Jan-Feb;30(1):20-6. PubMed PMID: 11798991.

© 2016 American Academy of Orthopaedic Surgeons 2016 Orthopaedic In-Training Examination


214 • American Academy of Orthopaedic Surgeons

Question 159
Which risk factor for anterior cruciate ligament (ACL) injuries contributes most to the higher rate of ACL
injuries among female athletes vs male athletes participating in the same sport?

1. Varus alignment
2. Estrogen levels
3. ACL size
4. Intra-articular notch size
5. Neuromuscular coordination and training

PREFERRED RESPONSE: 5

RECOMMENDED READINGS
Griffin LY, Albohm MJ, Arendt EA, Bahr R, Beynnon BD, Demaio M, Dick RW, Engebretsen L, Garrett
WE Jr, Hannafin JA, Hewett TE, Huston LJ, Ireland ML, Johnson RJ, Lephart S, Mandelbaum BR, Mann
BJ, Marks PH, Marshall SW, Myklebust G, Noyes FR, Powers C, Shields C Jr, Shultz SJ, Silvers H,
Slauterbeck J, Taylor DC, Teitz CC, Wojtys EM, Yu B. Understanding and preventing noncontact anterior
cruciate ligament injuries: a review of the Hunt Valley II meeting, January 2005. Am J Sports Med. 2006
Sep;34(9):1512-32. PubMed PMID: 16905673.

Voskanian N. ACL Injury prevention in female athletes: review of the literature and practical
considerations in implementing an ACL prevention program. Curr Rev Musculoskelet Med. 2013
Jun;6(2):158-63. doi: 10.1007/s12178-013-9158-y. PubMed PMID: 23413024.

Zebis MK, Bencke J, Andersen LL, Døssing S, Alkjaer T, Magnusson SP, Kjaer M, Aagaard P. The
effects of neuromuscular training on knee joint motor control during sidecutting in female elite soccer
and handball players. Clin J Sport Med. 2008 Jul;18(4):329-37. doi: 10.1097/JSM.0b013e31817f3e35.
PubMed PMID: 18614884.

Question 171
A 45-year-old laborer has lateral elbow pain with forceful use of the upper extremity. An examination
reveals he has pain with resisted wrist extension while his elbow is fully extended. Which muscle origin is
most likely involved?

1. Distal biceps brachii


2. Extensor carpi radialis brevis (ECRB)
3. Extensor carpi radialis longus (ECRL)
4. Brachioradialis
5. Pronator teres

PREFERRED RESPONSE: 2

© 2016 American Academy of Orthopaedic Surgeons 2016 Orthopaedic In-Training Examination


SECTION 9: Sports Medicine • 215

RECOMMENDED READINGS
Brummel J, Baker CL 3rd, Hopkins R, Baker CL Jr. Epicondylitis: lateral. Sports Med Arthrosc. 2014
Sep;22(3):e1-6. doi: 10.1097/JSA.0000000000000024. Review. PubMed PMID: 25077751.

Calfee RP, Patel A, DaSilva MF, Akelman E. Management of lateral epicondylitis: current concepts. J Am
Acad Orthop Surg. 2008 Jan;16(1):19-29. Review. PubMed PMID: 18180389.

Figure 181

Question 181
Figure 181 is the anteroposterior radiograph of a 13-year-old track athlete who had the immediate onset of
right hip pain after his trailing limb struck a hurdle and he fell down. The muscle originating at the injured
structure is innervated by which nerve?

1. Genitofemoral
2. Pudendal
3. Femoral
4. Obturator
5. Superior gluteal

PREFERRED RESPONSE: 3

© 2016 American Academy of Orthopaedic Surgeons 2016 Orthopaedic In-Training Examination


216 • American Academy of Orthopaedic Surgeons

RECOMMENDED READINGS
Holden CP, Holman J, Herman MJ. Pediatric pelvic fractures. J Am Acad Orthop Surg. 2007
Mar;15(3):172-7. Review. PubMed PMID: 17341674.

Rossi F, Dragoni S. Acute avulsion fractures of the pelvis in adolescent competitive athletes: prevalence,
location and sports distribution of 203 cases collected. Skeletal Radiol. 2001 Mar;30(3):127-31. PubMed
PMID: 11357449.

Question 184
What is the most common reason for hip arthroscopy revision that addresses femoroacetabular
impingement?

1. Untreated chondral defect


2. Iatrogenic chondral injury
3. Adhesive capsulitis
4. Persistent femoroacetabular impingement
5. Hip instability

PREFERRED RESPONSE: 4

RECOMMENDED READINGS
Phillippon MJ, Schenker ML, Briggs KK, Kuppersmith DA, Maxwell RB, Stubbs AJ. Revision hip
arthroscopy. Am J Sports Med. 2007 Nov;35(11):1918-1921. PubMed PMID: 17703000.

Bogunovic L, Gottlieb M, Pashos G, Baca G, Clohisy JC. Why do hip arthroscopy procedures fail? Clin
Orthop Relat Res. 2013 Aug;417(8):2523-2529. PubMed PMID: 23637056.

Question 188
A 17-year-old gymnast sustains an anterior cruciate ligament (ACL) injury during an awkward landing.
After counseling and discussion, it is decided that she will undergo ACL reconstruction using hamstring
autograft. What is the smallest-diameter graft that can minimize risk for rupture?

1. 7 mm
2. 7.5 mm
3. 8.5 mm
4. 9 mm
5. 10 mm

PREFERRED RESPONSE: 3

© 2016 American Academy of Orthopaedic Surgeons 2016 Orthopaedic In-Training Examination


SECTION 9: Sports Medicine • 217

RECOMMENDED READINGS
Magnussen RA, Lawrence JT, West RL, Toth AP, Taylor DC, Garrett WE. Graft size and patient age
are predictors of early revision after anterior cruciate ligament reconstruction with hamstring autograft.
Arthroscopy. 2012 Apr;28(4):526-531. PubMed PMID: 22305299.

Park SY, Oh H, Park S, Lee JH, Lee SH, Yoon KH. Factors predicting hamstring tendon autograft
diameters and resulting failure rates after anterior cruciate ligament reconstruction. Knee Surg Sports
Traumatol Arthrosc. 2013 May;21(5):1111-1118. PubMed PMID: 22688502.

Question 201
A collegiate swimmer develops medial winging of the scapula. Which nerve roots are most likely to be
involved in an electromyography or nerve conduction study with abnormal findings?

1. C7, C8, T1
2. C6, C7, C8
3. C5, C6, C7
4. C4, C5, C6
5. C3, C4, C5

PREFERRED RESPONSE: 3

RECOMMENDED READINGS
Meininger AK, Figuerres BF, Goldberg BA. Scapular winging: an update. J Am Acad Orthop Surg. 2011
Aug;19(8):453-62. Review. PubMed PMID: 21807913.

Gregg JR, Labosky D, Harty M, Lotke P, Ecker M, DiStefano V, Das M. Serratus anterior paralysis in the
young athlete. J Bone Joint Surg Am. 1979 Sep;61(6A):825-32. PubMed PMID: 479228.

Question 209
Which exercise should typically be avoided during initial therapy following anterior cruciate ligament
(ACL) reconstruction?

1. Vertical squat
2. Stationary bike riding
3. Light leg press
4. Seated leg extensions
5. Use of a stair-climbing machine

PREFERRED RESPONSE: 4

© 2016 American Academy of Orthopaedic Surgeons 2016 Orthopaedic In-Training Examination


218 • American Academy of Orthopaedic Surgeons

RECOMMENDED READINGS
McMullen J. Rehabilitation principles: kinetic chain therapeutic exercise application and progression. In:
Garrick JG, ed. Orthopaedic Knowledge Update: Sports Medicine 3. Rosemont, IL: American Academy of
Orthopaedic Surgeons; 2004:129-136.

Kruse LM, Gray B, Wright RW. Rehabilitation after anterior cruciate ligament reconstruction: a systematic
review. J Bone Joint Surg Am. 2012 Oct 3;94(19):1737-48. doi: 10.2106/JBJS.K.01246. Review. PubMed
PMID: 23032584.

Andersson D, Samuelsson K, Karlsson J. Treatment of anterior cruciate ligament injuries with special
reference to surgical technique and rehabilitation: an assessment of randomized controlled trials.
Arthroscopy. 2009 Jun;25(6):653-85. doi: 10.1016/j.arthro.2009.04.066. Review. PubMed PMID:
19501297.

Question 231
What is an advantage of the lateral decubitus position for arthroscopy vs the beach-chair position?

1. Lower rate of deep venous thrombosis (DVT)


2. Lower rate of nerve injury with creation of an anteroinferior portal
3. Easier conversion to an open procedure
4. Ease of airway management
5. Decreased concern about cerebral hypoperfusion with hypotensive analgesia

PREFERRED RESPONSE: 5

RECOMMENDED READINGS
Gelber PE, Reina F, Caceres E, Monllau JC. A comparison of risk between the lateral decubitus and the
beach-chair position when establishing an anteroinferior shoulder portal: a cadaveric study. Arthroscopy.
2007 May;23(5):522-8. PubMed PMID: 17478284.

Li X, Eichinger JK, Hartshorn T, Zhou H, Matzkin EG, Warner JP. A comparison of the lateral decubitus
and beach-chair positions for shoulder surgery: advantages and complications. J Am Acad Orthop Surg.
2015 Jan;23(1):18-28. doi: 10.5435/JAAOS-23-01-18. Review. PubMed PMID: 25538127.

© 2016 American Academy of Orthopaedic Surgeons 2016 Orthopaedic In-Training Examination


SECTION 9: Sports Medicine • 219

Figure 236

Question 236
Figure 236 is the sagittal MR image of a 19-year-old football player. What is the strongest predisposing
factor to this condition?

1. Playing on soft surfaces


2. Hamstring and quadriceps inflexibility
3. Lower weight
4. Increased quadriceps strength
5. Decreased training frequency

PREFERRED RESPONSE: 2

RECOMMENDED READINGS
Beam JW, Lozman PR, Uribe JW. Surgical treatment of chronic patellar tendinitis in a collegiate football
player. J Athl Train. 1995 Oct;30(4):342-4. PubMed PMID: 16558360.

Witvrouw E, Bellemans J, Lysens R, Danneels L, Cambier D. Intrinsic risk factors for the development of
patellar tendinitis in an athletic population. A two-year prospective study. Am J Sports Med. 2001 Mar-
Apr;29(2):190-5. PubMed PMID: 11292044.

Zwerver J, Bredeweg SW, van den Akker-Scheek I. Prevalence of Jumper's knee among nonelite
athletes from different sports: a cross-sectional survey. Am J Sports Med. 2011 Sep;39(9):1984-8. doi:
10.1177/0363546511413370. Epub 2011 Jul 7. PubMed PMID: 21737835.

© 2016 American Academy of Orthopaedic Surgeons 2016 Orthopaedic In-Training Examination


220 • American Academy of Orthopaedic Surgeons

Figure 244

Question 244
Figure 244 is the radiograph of an 11-year-old boy who has had foot pain for 4 weeks while playing
basketball for his school team. What is the best next step?

1. Coalition excision with fat interposition


2. Coalition excision with extensor digitorum brevis interposition
3. Subtalar arthrodesis
4. 4 weeks of immobilization in a short-leg cast
5. Excision of accessory navicular

PREFERRED RESPONSE: 4

RECOMMENDED READINGS
Vincent KA. Tarsal coalition and painful flatfoot. J Am Acad Orthop Surg. 1998 Sep-Oct;6(5):274-81.
Review. PubMed PMID: 9753754.

Mosca VS. Subtalar coalition in pediatrics. Foot Ankle Clin. 2015 Jun;20(2):265-81. doi: 10.1016/j.
fcl.2015.02.005. Epub 2015 Apr 18. Review. PubMed PMID: 26043243.

© 2016 American Academy of Orthopaedic Surgeons 2016 Orthopaedic In-Training Examination


SECTION 9: Sports Medicine • 221

Question 252
A 38-year-old recreational runner experiences occasional mild hip pain after long, aggressive runs. An
examination reveals full range of motion with nonspecific tenderness in the groin area and no pain with
hip flexion and internal rotation. Plain radiographs are unremarkable. MR imaging obtained by his primary
care provider shows a labral tear. Initial treatment should include

1. a 3-month trial of nonsteroidal anti-inflammatory drugs.


2. arthroscopy for labrum excision vs repair.
3. an intra-articular steroid injection.
4. an MRI arthrogram to confirm the labrum tear.
5. reassurance and treatment of symptoms.

PREFERRED RESPONSE: 5

RECOMMENDED READINGS
Schmitz MR, Campbell SE, Fajardo RS, Kadrmas WR. Identification of acetabular labral pathological
changes in asymptomatic volunteers using optimized, noncontrast 1.5-T magnetic resonance imaging. Am
J Sports Med. 2012 Jun;40(6):1337-1341. PubMed PMID: 22422932.

Register B, Pennock AT, Ho CP, Strickland CD, Lawand A, Philippon MJ. Prevalence of abnormal
hip findings in asymptomatic participants: a prospective, blinded study. Am J Sports Med. 2012
Dec;40(12):2720-2724. PubMed PMID: 23104610.

© 2016 American Academy of Orthopaedic Surgeons 2016 Orthopaedic In-Training Examination


222 • American Academy of Orthopaedic Surgeons

Figure 264

Question 264
What is the proper location of the femoral insertion of the medial patellofemoral ligament (MPFL) in
Figure 264, and what position should the knee be in when setting tension?

1. Position A and fixed with the knee in full extension


2. Position A and fixed with the knee in 70 degrees of flexion
3. Position A and fixed with the knee at 30 degrees of flexion
4. Position B and fixed with the knee at 30 degrees of flexion
5. Position B and fixed with the knee in full extension

PREFERRED RESPONSE: 4

RECOMMENDED READINGS
Schöttle PB, Schmeling A, Rosenstiel N, Weiler A. Radiographic landmarks for femoral tunnel placement
in medial patellofemoral ligament reconstruction. Am J Sports Med. 2007 May;35(5):801-804. PubMed
PMID: 17267773.

Wijdicks CA, Griffith CJ, LaPrade RF, et al. Radiographic identification of the primary medial knee
structures. J Bone Joint Surg Am. 2009 Mar 1;91(3):521-529. PubMed PMID: 19255211.

© 2016 American Academy of Orthopaedic Surgeons 2016 Orthopaedic In-Training Examination


SECTION 9: Sports Medicine • 223

Figure 275a Figure 275b

Figure 275c Figure 275d

Question 275
Figures 275a and 275b are MR images of a 27-year-old recreational athlete who has no history of trauma
but is having pain and mechanical symptoms when performing overhead activities and lifting. He has not
experienced pain relief after participating in 2 months of physical therapy, but did experience 6 weeks
of pain relief after receiving an acromioclavicular joint injection. An examination reveals full range of
motion, good rotator cuff strength, mild pain with O’Brien testing, positive cross-arm pain, and tenderness
over the acromioclavicular joint. He has no tenderness in the bicipital groove. Arthroscopic images are
shown in Figures 275c and 275d. In addition to a distal clavicle resection, how would you treat the lesion
shown in the arthroscopic images?

1. Anterior capsulorrhaphy
2. No additional treatment
3. Rotator cuff debridement
4. Biceps tenodesis
5. Arthroscopic superior labrum anterior and posterior (SLAP) repair

PREFERRED RESPONSE: 2

© 2016 American Academy of Orthopaedic Surgeons 2016 Orthopaedic In-Training Examination


224 • American Academy of Orthopaedic Surgeons

RECOMMENDED READINGS
Weber SC, Martin DF, Seiler JG 3rd, Harrast JJ. Superior labrum anterior and posterior lesions of the
shoulder: incidence rates, complications, and outcomes as reported by American Board of Orthopaedic
Surgery Part II candidates. Am J Sports Med. 2012 July;40(7):1538-1543. PubMed PMID: 22628153.

Gobezie R, Zurakowski D, Lavery K, Millett PJ, Cole BJ, Warner JJ. Analysis of interobserver and
intraobserver variability in the diagnosis and treatment of SLAP tears using the Snyder classification. Am
J Sports Med. 2008 Jul;36(7):1373-1379. PubMed PMID: 18354142.

Waterman BR, Cameron KL, Hsiao M, Langston JR, Clark NJ, Owens BD. Trends in the diagnosis of
SLAP lesions in the US military. Knee Surg Sports Traumatol Arthrosc. 2015 May;23(5):1453-9. doi:
10.1007/s00167-013-2798-z. PubMed PMID: 24318507.

Tuite MJ, Currie JW, Orwin JF, Baer GS, del Rio AM. Sublabral clefts and recesses in the anterior,
inferior, and posterior glenoid labrum at MR arthrography. Skeletal Radiol. 2013 Mar;42(3):353-62. doi:
10.1007/s00256-012-1496-0. PubMed PMID: 22893302.

© 2016 American Academy of Orthopaedic Surgeons 2016 Orthopaedic In-Training Examination


SECTION 10: Trauma • 225

SECTION 10: Trauma

Figure 2a Figure 2b

Question 2
What is the most biomechanically optimal fixation method to address the fracture shown in Figures 2a
and 2b?

1. Bicortical lag screws


2. Partially threaded cancellous screws
3. Tension band construct with smooth wires
4. Medial antiglide plate
5. Precontoured lateral locking plate

PREFERRED RESPONSE: 4

RECOMMENDED READINGS
Hak DJ, Egol KA, Gardner MJ, Haskell A. The "not so simple" ankle fracture: avoiding problems and
pitfalls to improve patient outcomes. Instr Course Lect. 2011;60:73-88. PubMed PMID: 21553763.

Ricci WM, Tornetta P, Borrelli J Jr. Lag screw fixation of medial malleolar fractures: a biomechanical,
radiographic, and clinical comparison of unicortical partially threaded lag screws and bicortical fully
threaded lag screws. J Orthop Trauma. 2012 Oct;26(10):602-6. PubMed PMID: 22437423. 

© 2016 American Academy of Orthopaedic Surgeons 2016 Orthopaedic In-Training Examination


226 • American Academy of Orthopaedic Surgeons

Question 6
A 25-year-old man had a severe pronation external rotation ankle injury with an oblique fibular fracture
above the joint line and a 10% posterior malleolar fracture. After restoring fibular length and rotation after
performing plate fixation, the approach to the posterior malleolar fracture and the optimal syndesmosis
closure can be achieved by applying the clamp at which position?

1. Proximal to the syndesmosis level with midmedial tibial and fibular placement with reduction
and fixation of the posterior malleolus
2. A point distal to the syndesmosis level with the medial clamp at the mid anteroposterior (AP)
tibia and fibular diameter, with no reduction and fixation of the posterior malleolus
3. At the syndesmosis level with the clamp applied to the anterior medial tibia and lateral
malleolar ridge, with reduction and fixation of the posterior malleolus
4. At the syndesmosis level with midmedial tibial and fibular placement with no reduction and
fixation of the posterior malleolus
5. At the syndesmosis level with the midmedial tibia and fibular ridge, with reduction and
fixation of the posterior malleolus

PREFERRED RESPONSE: 5

RECOMMENDED READINGS
Gardner MJ, Graves ML, Higgins TF, Nork SE. Technical Considerations in the Treatment of Syndesmotic
Injuries Associated With Ankle Fractures. J Am Acad Orthop Surg. 2015 Aug;23(8):510-8. doi: 10.5435/
JAAOS-D-14-00233. PMID: 26209146.

Phisitkul P, Ebinger T, Goetz J, Vaseenon T, Marsh JL. Forceps reduction of the syndesmosis in rotational
ankle fractures: a cadaveric study J Bone Joint Surg Am. 2012 Dec 19;94(24):2256-61. doi: 10.2106/
JBJS.K.01726. PMID: 23318616.

Miller AN, Carroll EA, Parker RJ, Boraiah S, Helfet DL, Lorich DG. Direct visualization for syndesmotic
stabilization of ankle fractures. Foot Ankle Int. 2009 May;30(5):419-26. doi: 10.3113/FAI.2009.0419.
PMID: 19439142.

© 2016 American Academy of Orthopaedic Surgeons 2016 Orthopaedic In-Training Examination


SECTION 10: Trauma • 227

Figure 10a Figure 10b

Question 10
Figures 10a and 10b are the emergency department radiographs of a 32-year-old healthy man who is
involved in a motor vehicle collision. He has an isolated injury. What is the best next step?

1. Anterior total hip arthroplasty (THA)


2. Open reduction and internal fixation (ORIF)
3. Closed reduction percutaneous pinning
4. Posterior THA
5. Hemiarthroplasty

PREFERRED RESPONSE: 2

RECOMMENDED READINGS
Callaghan JJ, Liu SS, Haidukewych GJ. Subcapital fractures: a changing paradigm. J Bone Joint Surg
Br. 2012 Nov;94(11 Suppl A):19-21. doi: 10.1302/0301-620X.94B11.30617. Review. PubMed PMID:
23118374.

Bhandari M, Devereaux PJ, Swiontkowski MF, Tornetta P 3rd, Obremskey W, Koval KJ, Nork S, Sprague
S, Schemitsch EH, Guyatt GH. Internal fixation compared with arthroplasty for displaced fractures of
the femoral neck. A meta-analysis. J Bone Joint Surg Am. 2003 Sep;85-A(9):1673-81. PubMed PMID:
12954824.

© 2016 American Academy of Orthopaedic Surgeons 2016 Orthopaedic In-Training Examination


228 • American Academy of Orthopaedic Surgeons

Question 14
Loss of active knee extension after use of an anterior subcutaneous internal fixator in the surgical
treatment of unstable pelvic fractures is attributable to

1. lateral femoral cutaneous nerve injury.


2. obturator nerve injury.
3. saphenous nerve injury.
4. femoral nerve injury.
5. thrombosis of the femoral artery.

PREFERRED RESPONSE: 4

RECOMMENDED READINGS
Hesse D, Kandmir U, Solberg B, Stroh A, Osgood G, Sems SA, Collinge CA. Femoral nerve palsy after
pelvic fracture treated with INFIX: a case series. J Orthop Trauma. 2015 Mar;29(3):138-43. doi: 10.1097/
BOT.0000000000000193. PubMed PMID: 24983430.

Lewallen DG. Neurovascular injury associated with hip arthroplasty. Instr Course Lect. 1998;47:275-83.
Review. PubMed PMID: 9571429.

© 2016 American Academy of Orthopaedic Surgeons 2016 Orthopaedic In-Training Examination


SECTION 10: Trauma • 229

Figure 18a Figure 18b

Question 18
Figures 18a and 18b are the clinical photographs of a 30-year-old man seen 1 year after undergoing locked
antegrade intramedullary nailing for a comminuted right femoral fracture. He has pain over his right hip.
What is the most likely diagnosis?

1. A decrease in femoral anteversion on the right during surgery


2. A malreduction externally rotating the proximal right femur during nailing
3. A malreduction internally rotating the distal right femur during nailing
4. An increase in femoral anteversion on the right during surgery
5. An acceptable outcome

PREFERRED RESPONSE: 1

RECOMMENDED READINGS
Dimitriou D, Tsai TY, Yue B, Rubash HE, Kwon YM, Li G. Side-to-side variation in normal femoral
morphology: 3D CT analysis of 122 femurs. Orthop Traumatol Surg Res. 2016 Feb;102(1):91-7. doi:
10.1016/j.otsr.2015.11.004. Epub 2016 Jan 19. PubMed PMID: 26867707.

Karaman O, Ayhan E, Kesmezacar H, Seker A, Unlu MC, Aydingoz O. Rotational malalignment after
closed intramedullary nailing of femoral shaft fractures and its influence on daily life. Eur J Orthop Surg
Traumatol. 2014 Oct;24(7):1243-7. doi: 10.1007/s00590-013-1289-8. Epub 2013 Aug 11. PubMed PMID:
23934503.

Espinoza C, Sathy AK, Moore DS, Starr AJ, Reinert CM. Use of inherent anteversion of an intramedullary
nail to avoid malrotation in femur fractures. J Orthop Trauma. 2014 Feb;28(2):e34-8. doi: 10.1097/
BOT.0b013e318298e48c. PubMed PMID: 23689227.

© 2016 American Academy of Orthopaedic Surgeons 2016 Orthopaedic In-Training Examination


230 • American Academy of Orthopaedic Surgeons

Question 32
A 72-year-old woman sustained a displaced femoral neck fracture after a fall. She is a community
ambulator and plays tennis and golf weekly. What is the best functional treatment option for her hip?

1. Total hip arthroplasty (THA)


2. Hemiarthroplasty
3. Hip resurfacing
4. Internal fixation with sliding hip and antirotation screws
5. Internal fixation with cannulated screws

PREFERRED RESPONSE: 1

RECOMMENDED READINGS
Florschutz AV, Langford JR, Haidukewych GJ, Koval KJ. Femoral neck fractures: current management. J
Orthop Trauma. 2015 Mar;29(3):121-9. doi: 10.1097/BOT.0000000000000291. Review. PubMed PMID:
25635363.

Avery PP, Baker RP, Walton MJ, Rooker JC, Squires B, Gargan MF, Bannister GC. Total hip replacement
and hemiarthroplasty in mobile, independent patients with a displaced intracapsular fracture of the femoral
neck: a seven- to ten-year follow-up report of a prospective randomised controlled trial. J Bone Joint Surg
Br. 2011 Aug;93(8):1045-8. PubMed PMID: 21768626.

Hedbeck CJ, Enocson A, Lapidus G, Blomfeldt R, Törnkvist H, Ponzer S, Tidermark J. Comparison of


bipolar hemiarthroplasty with total hip arthroplasty for displaced femoral neck fractures: a concise four
year follow-up of a randomized trial. J Bone Joint Surg Am. 2011 Mar 2;93(5):445-50. PubMed PMID:
21368076.

Question 40
When comparing sliding hip screw fixation to intramedullary nailing (IMN) in the management of
transverse/reverse oblique trochanteric and subtrochanteric fractures, sliding hip screw fixation is
associated with

1. higher revision surgery rates.


2. lower pain.
3. better satisfaction.
4. better quality of life.
5. better mobility.

PREFERRED RESPONSE: 1

© 2016 American Academy of Orthopaedic Surgeons 2016 Orthopaedic In-Training Examination


SECTION 10: Trauma • 231

RECOMMENDED READINGS
Matre K, Havelin LI, Gjertsen JE, Vinje T, Espehaug B, Fevang JM. Sliding hip screw versus IM nail in
reverse oblique trochanteric and subtrochanteric fractures. A study of 2716 patients in the Norwegian Hip
Fracture Register. Injury. 2013 Jun;44(6):735-42. doi: 10.1016/j.injury.2012.12.010. Epub 2013 Jan 8.
PubMed PMID: 23305689.

Miedel R, Ponzer S, Törnkvist H, Söderqvist A, Tidermark J. The standard Gamma nail or the Medoff
sliding plate for unstable trochanteric and subtrochanteric fractures. A randomised, controlled trial. J Bone
Joint Surg Br. 2005 Jan;87(1):68-75. PubMed PMID: 15686240.

Question 57
Which variable is associated with poor outcomes and early need for arthroplasty in the setting of
acetabulum fracture?

1. Anatomical fracture reduction


2. Posterior hip dislocation
3. Femoral head cartilage lesion
4. Initial displacement of the articular surface of less than 20 mm
5. Postsurgical congruence of the acetabular roof

PREFERRED RESPONSE: 3

RECOMMENDED READINGS
Tannast M, Najibi S, Matta JM. Two to twenty-year survivorship of the hip in 810 patients with
operatively treated acetabular fractures. J Bone Joint Surg Am. 2012 Sep 5;94(17):1559-67. PubMed
PMID: 22992846.

Liebergall M, Mosheiff R, Low J, Goldvirt M, Matan Y, Segal D. Acetabular fractures. Clinical outcome
of surgical treatment. Clin Orthop Relat Res. 1999 Sep;(366):205-16. PubMed PMID: 10627737.

© 2016 American Academy of Orthopaedic Surgeons 2016 Orthopaedic In-Training Examination


232 • American Academy of Orthopaedic Surgeons

Question 63
A healthy 24-year-old woman who is identified as American Society of Anesthesiologists (ASA) class
I has a Gustilo-Anderson grade I open bimalleolar ankle fracture. Antibiotics, irrigation with low-flow
cystoscopy tubing, irrigation and debridement, open reduction and internal fixation (ORIF), and primary
wound closure are associated with

1. a high probability of wound healing.


2. a high probability of wound necrosis.
3. a higher probability of wound necrosis than if pulsatile lavage were used.
4. no difference in wound necrosis for healthy patients and those with medical comorbidities.
5. none of these; wounds should never be primarily closed in open fractures.

PREFERRED RESPONSE: 1

RECOMMENDED READINGS
Ovaska MT, Madanat R, Mäkinen TJ. Predictors of Postoperative Wound Necrosis Following
Primary Wound Closure of Open Ankle Fractures. Foot Ankle Int. 2016 Apr;37(4):401-6. doi:
10.1177/1071100715609182. PubMed PMID: 26830839.

Jenkinson RJ, Kiss A, Johnson S, Stephen DJ, Kreder HJ. Delayed wound closure increases deep-infection
rate associated with lower-grade open fractures: a propensity-matched cohort study. J Bone Joint Surg Am.
2014 Mar 5;96(5):380-6. doi: 10.2106/JBJS.L.00545. PubMed PMID: 24599199.

Question 73
Impingement and penetration of the anterior cortex of the distal femur during intramedullary nailing
(IMN) in the setting of proximal femur fractures are seen in patients with

1. a posterior starting point.


2. a small radius of curvature of the nail.
3. tall stature.
4. decreased femoral bow.
5. an anterior starting point.

PREFERRED RESPONSE: 1

RECOMMENDED READINGS
Roberts JW, Libet LA, Wolinsky PR. Who is in danger? Impingement and penetration of the anterior
cortex of the distal femur during intramedullary nailing of proximal femur fractures: preoperatively
measurable risk factors. J Trauma Acute Care Surg. 2012 Jul;73(1):249-54. doi: 10.1097/
TA.0b013e318256a0b6. PubMed PMID: 22743391.

Ostrum RF, Levy MS. Penetration of the distal femoral anterior cortex during intramedullary nailing
for subtrochanteric fractures: a report of three cases. J Orthop Trauma. 2005 Oct;19(9):656-60. PubMed
PMID: 16247312.

© 2016 American Academy of Orthopaedic Surgeons 2016 Orthopaedic In-Training Examination


SECTION 10: Trauma • 233

Figure 78

Question 78
During the classic Henry approach to the right forearm (Figure 78), where is the radial artery located?

1. Above retractor A
2. Between retractors A and C
3. Below retractors B and C
4. Below the pronator terres
5. The artery is not seen in this approach

PREFERRED RESPONSE: 3

RECOMMENDED READINGS
Campbell WS, Canale ST, Beaty JS, eds. Campbell’s Operative Orthopaedics. Philadelphia, PA: Elsevier/
Mosby; 2013:120.

Catalano LW 3rd, Zlotolow DA, Hitchcock PB, Shah SN, Barron OA. Surgical exposures of the radius
and ulna. J Am Acad Orthop Surg. 2011 Jul;19(7):430-8. Review. PubMed PMID: 21724922.

© 2016 American Academy of Orthopaedic Surgeons 2016 Orthopaedic In-Training Examination


234 • American Academy of Orthopaedic Surgeons

Figure 82a Figure 82b Figure 82c

Question 82
Which soft-tissue structure associated with the fracture seen in Figures 82a through 82c is most
commonly injured?

1. Medial collateral ligament


2. Medial meniscus
3. Popliteus tendon
4. Lateral meniscus
5. Lateral collateral ligament

PREFERRED RESPONSE: 4

RECOMMENDED READINGS
Gardner MJ, Yacoubian S, Geller D, Pode M, Mintz D, Helfet DL, Lorich DG. Prediction of soft-tissue
injuries in Schatzker II tibial plateau fractures based on measurements of plain radiographs. J Trauma.
2006 Feb;60(2):319-23; discussion 324. PubMed PMID: 16508489.

Gardner MJ, Yacoubian S, Geller D, Suk M, Mintz D, Potter H, Helfet DL, Lorich DG. The incidence
of soft tissue injury in operative tibial plateau fractures: a magnetic resonance imaging analysis of 103
patients. J Orthop Trauma. 2005 Feb;19(2):79-84. PubMed PMID: 15677922.

© 2016 American Academy of Orthopaedic Surgeons 2016 Orthopaedic In-Training Examination


SECTION 10: Trauma • 235

Figure 91

Question 91
Figure 91 depicts the external rotation stress test performed after open reduction and internal fixation on
the lateral malleolus of a bimalleolar-equivalent ankle fracture. What is the best next step?

1. Close the wounds and place a below-knee plaster splint


2. Reduce the syndesmosis with a king tong clamp and fix it with 2- x 3.5-mm screws with 4
cortexes each
3. Reduce the syndesmosis with a king tong clamp and fix it with a 3.5-mm screw
with 4 cortexes
4. Reduce the syndesmosis with a king tong clamp and fix it with a 4.5-mm screw with
4 cortexes
5. Repair the deltoid ligament

PREFERRED RESPONSE: 1

RECOMMENDED READINGS
Lower extremity fractures. In: Canale ST, Beaty JH, eds. Campbell’s Operative Orthopaedics. Vol 2. 12th
ed. Philadelphia, PA: Elsevier-Mosby; 2013:3044.

van den Bekerom MP. Diagnosing syndesmotic instability in ankle fractures. World J Orthop. 2011 Jul
18;2(7):51-6. doi: 10.5312/wjo.v2.i7.51. PubMed PMID: 22474636.

© 2016 American Academy of Orthopaedic Surgeons 2016 Orthopaedic In-Training Examination


236 • American Academy of Orthopaedic Surgeons

Question 97
When comparing intramedullary nailing (IMN) to percutaneous plating for the treatment of distal tibial
metaphyseal fractures, IMN is associated with

1. longer surgical time.


2. more radiation.
3. difficult implant removal.
4. shorter surgical time.
5. lower pain scores.

PREFERRED RESPONSE: 4

RECOMMENDED READINGS
Guo JJ, Tang N, Yang HL, Tang TS. A prospective, randomised trial comparing closed intramedullary
nailing with percutaneous plating in the treatment of distal metaphyseal fractures of the tibia. J Bone Joint
Surg Br. 2010 Jul;92(7):984-8. doi: 10.1302/0301-620X.92B7.22959. PubMed PMID: 20595119.

Im GI, Tae SK. Distal metaphyseal fractures of tibia: a prospective randomized trial of closed
reduction and intramedullary nail versus open reduction and plate and screws fixation. J Trauma. 2005
Nov;59(5):1219-23; discussion 1223. PubMed PMID: 16385303.

© 2016 American Academy of Orthopaedic Surgeons 2016 Orthopaedic In-Training Examination


SECTION 10: Trauma • 237

Figure 102a Figure 102b

Figure 102c Figure 102d

Question 102
Figures 102a through 102d are the anteroposterior and lateral radiographs and axial and sagittal CT scans
of a 40-year-old woman who was mowing her lawn when she slipped on wet grass and sustained an ankle
injury. When performing open reduction and internal fixation, which interval is best used to expose and
reduce the structure marked by the arrow?

1. Anterior tibial tendon and extensor hallucis longus


2. Peroneus brevis and peroneus tertius
3. Peroneus brevis and fibula
4. Flexor hallucis longus and peroneus longus
5. Flexor hallucis longus and flexor digitorum longus

PREFERRED RESPONSE: 4

© 2016 American Academy of Orthopaedic Surgeons 2016 Orthopaedic In-Training Examination


238 • American Academy of Orthopaedic Surgeons

RECOMMENDED READINGS
Irwin TA, Lien J, Kadakia AR. Posterior malleolus fracture. J Am Acad Orthop Surg. 2013 Jan;21(1):32-
40. doi: 10.5435/JAAOS-21-01-32. Review. PubMed PMID: 23281469.

Tornetta P 3rd, Ostrum RF, Trafton PG. Trimalleolar ankle fracture. J Orthop Trauma. 2001
Nov;15(8):588-90. PubMed PMID: 11733680.

Figure 107a Figure 107b

Question 107
Figures 107a and 107b are the radiographs of a 45-year-old patient. Which fixation method can most
effectively prevent malalignment?

1. Intramedullary nailing (IMN)


2. IMN with fibular fixation
3. IMN with polar screws
4. Plating of the distal tibia
5. External fixation

PREFERRED RESPONSE: 4

© 2016 American Academy of Orthopaedic Surgeons 2016 Orthopaedic In-Training Examination


SECTION 10: Trauma • 239

RECOMMENDED READINGS
Kwok CS, Crossman PT, Loizou CL. Plate versus nail for distal tibial fractures: a systematic review and
meta-analysis. J Orthop Trauma. 2014 Sep;28(9):542-8. doi: 10.1097/BOT.0000000000000068. Review.
PubMed PMID: 24464094.

Vallier HA, Cureton BA, Patterson BM. Randomized, prospective comparison of plate versus
intramedullary nail fixation for distal tibia shaft fractures. J Orthop Trauma. 2011 Dec;25(12):736-41. doi:
10.1097/BOT.0b013e318213f709. PubMed PMID: 21904230.

Figure 111

Question 111
The appropriate entry for an intramedullary tibia nail being used for fixation of a central third diaphyseal
tibial fracture is ideally positioned at which point in Figure 111?

1. A
2. B
3. C
4. D
5. E

PREFERRED RESPONSE: 2

RECOMMENDED READINGS
McConnell T, Tornetta P III, Tilzey J, Casey D. Tibial portal placement: the radiographic correlate of the
anatomic safe zone. J Orthop Trauma. 2001Mar-Apr;15(3):207-9. PubMed PMID: 11265012.

Song SJ, Jeong BO. Three-dimensional analysis of the intramedullary canal axis of tibia: clinical
relevance to tibia intramedullary nailing. Arch Orthop Trauma Surg. 2010 Jul;130(7):903-7. Epub 2009
Nov 3. PubMed PMID: 19885665.

© 2016 American Academy of Orthopaedic Surgeons 2016 Orthopaedic In-Training Examination


240 • American Academy of Orthopaedic Surgeons

Figure 114

Question 114
Figure 114 is the radiograph of a 23-year-old man who is seen in the emergency department after a motor
vehicle collision. He is hemodynamically stable, alert, and oriented, but he has pain in his left leg and hip.
An examination should reveal that the limb is

1. short and internally rotated.


2. short and in neutral alignment.
3. short and externally rotated.
4. long and externally rotated.
5. normal length and internally rotated.

PREFERRED RESPONSE: 3

RECOMMENDED READINGS
Canale ST, Beaty JH, Campbell WC. Fractures and dislocations. In: Campbell WS, Canale ST, Beaty JS,
eds. Campbell’s Operative Orthopaedics. Philadelphia, PA: Mosby/Elsevier; 2013:3249.

Dislocations of the Hip. In: Rockwood CA, Green DP, Heckman JD, Bucholz RW, eds. Rockwood and
Green’s Fractures in Adults. Philadelphia, PA: Lippincott Williams & Wilkins; 2001:1789.

© 2016 American Academy of Orthopaedic Surgeons 2016 Orthopaedic In-Training Examination


SECTION 10: Trauma • 241

Question 120
A 33-year-old man injures his right wrist and left leg in a motorcycle collision. He has a closed left
midshaft tibia fracture and a closed comminuted right intra-articular distal radius fracture. Prior to
treatment, he reports mild paresthesias in his right hand but can identify light touch on all digits. The
surgeon performs closed reduction and splinting of both injuries, and the patient reports resolution of
the median nerve paresthesias. Eight hours later, the surgeon is called to evaluate the patient because of
increasing pain in his right arm. He appears agitated and uncomfortable, with a heart rate of 120 and blood
pressure of 135/90 mm Hg. The surgeon opens his splint, and his right forearm compartments are soft. His
fingers are pink and well perfused with brisk capillary refill. He cannot identify light touch to his thumb,
index, or long finger, but can identify light touch to his small finger. He can actively flex and extend the
digits through a small arc of motion with pain and has discomfort with passive stretch. What is the best
next step?

1. Measure forearm compartment pressures


2. Obtain a stat complete blood count (CBC) and electrocardiogram (EKG)
3. Obtain electromyography (EMG) study
4. Emergent forearm fasciotomies
5. Emergent carpal tunnel release

PREFERRED RESPONSE: 5

RECOMMENDED READINGS
Niver GE, Ilyas AM. Carpal tunnel syndrome after distal radius fracture. Orthop Clin North Am. 2012
Oct;43(4):521-7. doi: 10.1016/j.ocl.2012.07.021. Epub 2012 Sep 4. Review. PubMed PMID: 23026468.

Davis DI, Baratz M. Soft tissue complications of distal radius fractures. Hand Clin. 2010 May;26(2):229-
35. doi: 10.1016/j.hcl.2009.11.002. Review. PubMed PMID: 20494749.

© 2016 American Academy of Orthopaedic Surgeons 2016 Orthopaedic In-Training Examination


242 • American Academy of Orthopaedic Surgeons

Figure 123a Figure 123b Figure 123c*


*Used with permission from Boyce RH, Singh K, Obremskey WT. Acute Management of Traumatic Knee Dislocations for the Generalist. J
Am Acad Orthop Surg. 2015 Dec;23(12):761-768. doi: 10.5435/JAAOS-D-14-00349. Epub 2015 Oct 22. Review. PubMed PMID: 26493970.

Question 123
Figures 123a and 123b are the radiographs of a 37-year-old man who was the front-seat passenger in a
motor vehicle collision. He is unable to move his knee and describes a shifting sensation. An examination
reveals limited range of motion and the appearance as shown in the clinical photograph in Figure 123c.
Foot pulses are palpable with an ankle-brachial index of 0.95. Several unsuccessful attempts at closed
reduction are made. What is the best next step?

1. Skeletal traction using a proximal tibial pin in the emergency room


2. Skeletal traction using a calcaneal pin in the emergency room
3. Open reduction through a posterior approach
4. Open reduction through an anteromedial approach
5. Closed reduction in the operating room using femoral distraction.

PREFERRED RESPONSE: 4

RECOMMENDED READINGS
Rihn JA, Groff YJ, Harner CD, Cha PS. The acutely dislocated knee: evaluation and management. J Am
Acad Orthop Surg. 2004 Sep-Oct;12(5):334-46. Review. PubMed PMID: 15469228.

Wand JS. A physical sign denoting irreducibility of a dislocated knee. J Bone Joint Surg Br. 1989
Nov;71(5):862. PubMed PMID: 2584265.

© 2016 American Academy of Orthopaedic Surgeons 2016 Orthopaedic In-Training Examination


SECTION 10: Trauma • 243

Question 128
An increased incidence of lateral meniscal tears is noted in lateral tibial plateau fractures associated with

1. more than 10 mm of joint depression.


2. less than 10 mm of joint depression.
3. patients older than 48 years of age.
4. a low-energy mechanism of injury.
5. young female patients.

PREFERRED RESPONSE: 1

RECOMMENDED READINGS
Ringus VM, Lemley FR, Hubbard DF, Wearden S, Jones DL. Lateral tibial plateau fracture depression
as a predictor of lateral meniscus pathology. Orthopedics. 2010 Feb;33(2):80-4. doi: 10.3928/01477447-
20100104-05. PubMed PMID: 20192139.

Stahl D, Serrano-Riera R, Collin K, Griffing R, Defenbaugh B, Sagi HC. Operatively Treated Meniscal
Tears Associated With Tibial Plateau Fractures: A Report on 661 Patients. J Orthop Trauma. 2015
Jul;29(7):322-4. doi: 10.1097/BOT.0000000000000290. PubMed PMID: 25635356.

© 2016 American Academy of Orthopaedic Surgeons 2016 Orthopaedic In-Training Examination


244 • American Academy of Orthopaedic Surgeons

Figure 132a Figure 132b Figure 132c

Question 132
Figures 132a and 132b are the radiographs of a 12-year-old boy who injured his right leg while
playing football. He was urgently treated with a closed reduction and percutaneous pinning. A follow-
up scanogram radiograph obtained 1 year later is shown in Figure 132c. His examination at this time
demonstrates a nonantalgic smooth gait with full motion of the right knee and full symmetric strength
in both lower extremities. A bone age radiograph is performed at this time, demonstrating a bone age of
approximately 12 years. What is the best next step?

1. Discharge from care


2. Continued observation every 6 to 9 months
3. Lengthening of the right femur using an external fixator
4. Lengthening of the right femur using an intramedullary device
5. Left distal femoral epiphysiodesis

PREFERRED RESPONSE: 5

RECOMMENDED READINGS
Basener CJ, Mehlman CT, DiPasquale TG. Growth disturbance after distal femoral growth plate
fractures in children: a meta-analysis. J Orthop Trauma. 2009 Oct;23(9):663-7. doi: 10.1097/
BOT.0b013e3181a4f25b. PubMed PMID: 19897989.

Arkader A, Warner WC Jr, Horn BD, Shaw RN, Wells L. Predicting the outcome of physeal fractures of
the distal femur. J Pediatr Orthop. 2007 Sep;27(6):703-8. PubMed PMID: 17717475.

© 2016 American Academy of Orthopaedic Surgeons 2016 Orthopaedic In-Training Examination


SECTION 10: Trauma • 245

Figure 135

Question 135
Figure 135 is a fluoroscopic image seen during antegrade intramedullary nailing of a femur with a
piriformis entry nail. What is the best next step?

1. Move the starting awl laterally


2. Move the starting awl posteriorly
3. Move the starting awl medially
4. Move the starting awl anteriorly
5. Further tap the awl before using a starting reamer

PREFERRED RESPONSE: 2

RECOMMENDED READINGS
Rudloff MI. Fractures of the lower extremity. In: Canale ST, Beaty JH, eds. Campbell’s Operative
Orthopaedics. Vol 2. 12th ed. Philadelphia, PA: Elsevier-Mosby; 2013:2617-2724.

Diaphyseal femoral fractures. In: Rockwood CA, Green DP, Heckman JD, Bucholz RW, eds. Rockwood
and Green’s Fractures in Adults. Philadelphia, PA: Lippincott Williams & Wilkins; 2001:1909.

Wheeless Textbook of Orthopaedic Surgery, http://www.wheelessonline.com/ortho/femoral_im_nail_


entry_point. (Accessed June 13, 2016)

© 2016 American Academy of Orthopaedic Surgeons 2016 Orthopaedic In-Training Examination


246 • American Academy of Orthopaedic Surgeons

Question 137
When compared against unicortical screw fixation for medial malleolar fractures, bicortical screw fixation
is associated with

1. superior biomechanical strength.


2. higher loosening rates.
3. higher nonunion rates.
4. higher screw removal rates.
5. worse clinical outcomes.

PREFERRED RESPONSE: 1

RECOMMENDED READINGS
Ricci WM, Tornetta P, Borrelli J Jr. Lag screw fixation of medial malleolar fractures: a biomechanical,
radiographic, and clinical comparison of unicortical partially threaded lag screws and bicortical fully
threaded lag screws. J Orthop Trauma. 2012 Oct;26(10):602-6. PubMed PMID: 22437423.

Perren SM, Frigg R, Hehli M, et al. Lag screw. In: Colton CL, Fernandez Dell’Oca A, Holz U, et al, eds.
AO Principles of Fracture Management. Stuttgart, Germany: Thieme; 2000:157-167.

© 2016 American Academy of Orthopaedic Surgeons 2016 Orthopaedic In-Training Examination


SECTION 10: Trauma • 247

Figure 145

Question 145
Figure 145 is the external rotation stress test performed after open reduction and internal fixation on the
lateral malleolus of a bimalleolar-equivalent ankle fracture. What is the best next step?

1. Close the wounds and place a below-knee plaster splint


2. Close the wounds and place an above-knee plaster splint
3. Reduce and stabilize the syndesmosis
4. Repair the deltoid ligament and retest the ankle
5. Perform a Cotton test

PREFERRED RESPONSE: 3

RECOMMENDED READINGS
van den Bekerom MP. Diagnosing syndesmotic instability in ankle fractures. World J Orthop. 2011 Jul
18;2(7):51-6. doi: 10.5312/wjo.v2.i7.51. PubMed PMID: 22474636.

Sman AD, Hiller CE, Refshauge KM. Diagnostic accuracy of clinical tests for diagnosis of ankle
syndesmosis injury: a systematic review. Br J Sports Med. 2013 Jul;47(10):620-8. doi: 10.1136/
bjsports-2012-091702. Epub 2012 Dec 6. Review. PubMed PMID: 23222193.

© 2016 American Academy of Orthopaedic Surgeons 2016 Orthopaedic In-Training Examination


248 • American Academy of Orthopaedic Surgeons

Figure 150a Figure 150b

Question 150
Which nerve is most commonly injured when a patient sustains the fracture seen in Figures 150a
and 150b?

1. Lateral antebrachial cutaneous


2. Radial
3. Median
4. Ulnar
5. Anterior interosseous

PREFERRED RESPONSE: 5

RECOMMENDED READINGS
Abzug JM, Herman MJ. Management of supracondylar humerus fractures in children: current concepts. J
Am Acad Orthop Surg. 2012 Feb;20(2):69-77. Review. PubMed PMID: 22302444.

Babal JC, Mehlman CT, Klein G. Nerve injuries associated with pediatric supracondylar humeral
fractures: a meta-analysis. J Pediatr Orthop. 2010 Apr-May;30(3):253-63. PubMed PMID: 20357592.

© 2016 American Academy of Orthopaedic Surgeons 2016 Orthopaedic In-Training Examination


SECTION 10: Trauma • 249

Figure 157

Question 157
Figure 157 is the radiograph of a 24-year-old man who sustained a low-velocity gunshot wound to his left
humerus. He is neurovascularly intact. What is the best next step?

1. Intramedullary (IM) nailing with exploration of his radial nerve


2. Open reduction and internal fixation (ORIF) with a bridge plate
3. ORIF with lag screws and a neutralization plate
4. ORIF with compression plating
5. Indirect reduction and a coaptation splint

PREFERRED RESPONSE: 5

RECOMMENDED READINGS
Vaidya R, Sethi A, Oliphant BW, Gibson V, Sethi S, Meehan R. Civilian gunshot injuries of the humerus.
Orthopedics. 2014 Mar;37(3):e307-12. doi: 10.3928/01477447-20140225-66. PubMed PMID: 24762161.

Dougherty PJ, Najibi S. Gunshot and wartime injury. In: Bucholz RW, Court-Brown CM, Heckman JD,
Tornetta P III, eds. Rockwood and Green's fractures in adults. Vol 1. 7th ed. Philadelphia, PA: Lippincott
Williams & Wilkins; 2011:303-330.

© 2016 American Academy of Orthopaedic Surgeons 2016 Orthopaedic In-Training Examination


250 • American Academy of Orthopaedic Surgeons

Figure 160a Figure 160b

Question 160
Figures 160a and 160b are the CT scans of a hemodynamically unstable 45-year-old polytrauma patient
who arrived at the emergency department with a cervical spine collar. He is brought in on a spinal board
and in a pelvic binder. The patient’s pelvic ring

1. has no injury; remove the binder.


2. has an injury; keep the binder.
3. has an anterior injury on CT scan; keep the binder.
4. has a posterior injury on CT scan; keep the binder.
5. may have an injury not seen on these CT images; keep the binder.

PREFERRED RESPONSE: 5

RECOMMENDED READINGS
Swartz J, Vaidya R, Hudson I, Oliphant B, Tonnos F. Effect of Pelvic Binder Placement on OTA
Classification of Pelvic Ring Injuries Using Computed Tomography. Does It Mask the Injury? J Orthop
Trauma. 2016 Jun;30(6):325-30. doi: 10.1097/BOT.0000000000000515. PubMed PMID: 26709813.

Clements J, Jeavons R, White C, McMurtry I. The Concealment of Significant Pelvic Injuries on


Computed Tomography Evaluation by Pelvic Compression Devices. J Emerg Med. 2015 Nov;49(5):675-8.
doi: 10.1016/j.jemermed.2015.03.014. PubMed PMID: 26054310.

© 2016 American Academy of Orthopaedic Surgeons 2016 Orthopaedic In-Training Examination


SECTION 10: Trauma • 251

Question 161
A 42-year-old man has a severe impact injury to his knee. An examination reveals a split depressed lateral
plateau fracture. MR imaging reveals a possibly torn lateral meniscus. What is the best surgical approach?

1. Arthroscopic evaluation, joystick elevation of the depressed fracture, and fixation


2. Arthroscopic detachment of the anterior tibio-meniscal ligament, fracture reduction, and
fixation followed by meniscal repair
3. Anterolateral approach with partial meniscal excision if there is a radial tear, meniscal repair if
there is a longitudinal injury, and fracture reduction and fixation
4. Anterolateral approach, detachment of the anterior meniscal ligament, repair of radial and
longitudinal tears, and fracture reduction with fixation
5. Anterolateral approach with repair only of the meniscal rim and fracture reduction and fixation

PREFERRED RESPONSE: 3

RECOMMENDED READINGS
Stahl D, Serrano-Riera R, Collin K, Griffing R, Defenbaugh B, Sagi HC. Operatively Treated Meniscal
Tears Associated With Tibial Plateau Fractures: A Report on 661 Patients. J Orthop Trauma. 2015
Jul;29(7):322-4. doi: 10.1097/BOT.0000000000000290. PubMed PMID: 25635356.

Ruiz-Ibán MÁ, Diaz-Heredia J, Elías-Martín E, Moros-Marco S, Cebreiro Martinez Del Val I. Repair
of meniscal tears associated with tibial plateau fractures: a review of 15 cases. Am J Sports Med. 2012
Oct;40(10):2289-95. PubMed PMID: 22962298.

Question 164
What is the most common complication after use of an anterior subcutaneous internal fixator in the
surgical treatment of an unstable pelvic fracture?

1. Heterotopic ossification
2. Infection
3. Prominent hardware
4. Revision surgery
5. Quadriceps weakness

PREFERRED RESPONSE: 1

© 2016 American Academy of Orthopaedic Surgeons 2016 Orthopaedic In-Training Examination


252 • American Academy of Orthopaedic Surgeons

RECOMMENDED READINGS
Vaidya R, Kubiak EN, Bergin PF, Dombroski DG, Critchlow RJ, Sethi A, Starr AJ. Complications of
anterior subcutaneous internal fixation for unstable pelvis fractures: a multicenter study. Clin Orthop Relat
Res. 2012 Aug;470(8):2124-31. doi: 10.1007/s11999-011-2233-z. PubMed PMID: 22219004.

Vaidya R, Colen R, Vigdorchik J, Tonnos F, Sethi A. Treatment of unstable pelvic ring injuries with an
internal anterior fixator and posterior fixation: initial clinical series. J Orthop Trauma. 2012 Jan;26(1):1-8.
doi: 10.1097/BOT.0b013e318233b8a7. PubMed PMID: 22048183.

Hesse D, Kandmir U, Solberg B, Stroh A, Osgood G, Sems SA, Collinge CA. Femoral nerve palsy after
pelvic fracture treated with INFIX: a case series. J Orthop Trauma. 2015 Mar;29(3):138-43. doi: 10.1097/
BOT.0000000000000193. PubMed PMID: 24983430.

Figure 168

Question 168
While preforming a piriformis entry femoral nail procedure for a midshaft femur fracture, the surgeon
begins locking proximally through the guide handle; however, trouble is encountered while drilling the
proximal part of the nail and a fluoroscopic shot is administered (Figure 168). What is the best next step?

1. Tighten the guide to the nail


2. Exchange the locking arm for a second-generation locking arm
3. Use a smaller-diameter drill
4. Remove the nail to assess if there is a fault with the guide device
5. Remove the guide wire

PREFERRED RESPONSE: 5

© 2016 American Academy of Orthopaedic Surgeons 2016 Orthopaedic In-Training Examination


SECTION 10: Trauma • 253

RECOMMENDED READINGS
Lower extremity fractures. In: Canale ST, Beaty JH, eds. Campbell’s Operative Orthopaedics. Vol 2. 12th
ed. Philadelphia, PA: Elsevier-Mosby; 2013:3200.

Diaphyseal femoral fractures. In: Rockwood CA, Green DP, Heckman JD, Bucholz RW, eds. Rockwood
and Green’s Fractures in Adults. Philadelphia, PA: Lippincott Williams & Wilkins; 2001:1909.

Question 173
When compared to autograft, use of calcium phosphate cement to augment subarticular defects in unstable
tibial plateau fractures demonstrates

1. less time to union.


2. less subsidence.
3. better union rates.
4. higher subsidence.
5. early arthritis.

PREFERRED RESPONSE: 2

RECOMMENDED READINGS
Russell TA, Leighton RK; Alpha-BSM Tibial Plateau Fracture Study Group. Comparison of autogenous
bone graft and endothermic calcium phosphate cement for defect augmentation in tibial plateau fractures.
A multicenter, prospective, randomized study. J Bone Joint Surg Am. 2008 Oct;90(10):2057-61. doi:
10.2106/JBJS.G.01191. PubMed PMID: 18829901.

Welch RD, Zhang H, Bronson DG. Experimental tibial plateau fractures augmented with calcium
phosphate cement or autologous bone graft. J Bone Joint Surg Am. 2003 Feb;85-A(2):222-31. PubMed
PMID: 12571298.

© 2016 American Academy of Orthopaedic Surgeons 2016 Orthopaedic In-Training Examination


254 • American Academy of Orthopaedic Surgeons

Figure 182a Figure 182b Figure 182c

Question 182
Figures 182a and 182b are the clinical photograph and postsurgical radiograph of a 45-year-old man who
sustained an open tibia fracture with loss of pulses in his leg. He goes into immediate surgery and his leg
is reduced, which results in improved foot perfusion. An intraoperative angiogram is performed (Figure
182c). This is followed by debridement and irrigation and external fixation. This patient has a Gustilo and
Anderson grade

1. I injury.
2. II injury.
3. IIIA injury.
4. IIIB injury.
5. IIIC injury.

PREFERRED RESPONSE: 4

RECOMMENDED READINGS
Canale ST, Beaty JH, Campbell WC. Fractures and dislocations. In: Campbell WS, Canale ST, Beaty JS,
eds. Campbell’s Operative Orthopaedics. Philadelphia, PA: Elsevier/Mosby; 2013:3030.

Gustilo RB, Mendoza RM, Williams DN. Problems in the management of type III (severe) open fractures:
a new classification of type III open fractures. J Trauma. 1984 Aug;24(8):742-6. PubMed PMID: 6471139.

© 2016 American Academy of Orthopaedic Surgeons 2016 Orthopaedic In-Training Examination


SECTION 10: Trauma • 255

Question 190
Outcomes following implant removal after ankle syndesmotic screw fixation have demonstrated

1. intact syndesmotic screws were associated with better outcomes.


2. loose screws were associated with the worst outcomes.
3. fractured screws were associated with the worst outcomes.
4. patients who underwent screw removal experienced the worst outcomes.
5. patients who underwent removal of loose or broken screws had similar outcomes.

PREFERRED RESPONSE: 5

RECOMMENDED READINGS
Manjoo A, Sanders DW, Tieszer C, MacLeod MD. Functional and radiographic results of patients with
syndesmotic screw fixation: implications for screw removal. J Orthop Trauma. 2010 Jan;24(1):2-6. doi:
10.1097/BOT.0b013e3181a9f7a5. PubMed PMID: 20035170.

Lash N, Horne G, Fielden J, Devane P. Ankle fractures: functional and lifestyle outcomes at 2 years. ANZ
J Surg. 2002 Oct;72(10):724-30. PubMed PMID: 12534384.

© 2016 American Academy of Orthopaedic Surgeons 2016 Orthopaedic In-Training Examination


256 • American Academy of Orthopaedic Surgeons

Figure 196a Figure 196b Figure 196c

Question 196
When surgically treating the acute injury shown in Figures 196a through 196c, what is the most important
step in restoring the relationship between the tibia and the fibula?

1. Repairing the deltoid


2. Anatomically reducing the fibula
3. Debriding the medial gutter
4. Obtaining contralateral ankle figures for comparison
5. Using a clamp for reduction of the syndesmosis

PREFERRED RESPONSE: 2

RECOMMENDED READINGS
Gardner MJ, Graves ML, Higgins TF, Nork SE. Technical Considerations in the treatment of Syndesmotic
Injuries Associated With Ankle Fractures. J Am Acad Orthop Surg. 2015 Aug;23(8):510-8. doi: 10.5435/
JAAOS-D-14-00233. Review. PubMed PMID: 26209146.

Gardner MJ, Demetrakopoulos D, Briggs SM, Helfet DL, Lorich DG. Malreduction of the tibiofibular
syndesmosis in ankle fractures. Foot Ankle Int. 2006 Oct;27(10):788-92. PubMed PMID: 17054878.

Zalavras C, Thordarson D. Ankle syndesmotic injury. J Am Acad Orthop Surg. 2007 Jun;15(6):330-9.
Review. PubMed PMID: 17548882.

Chen CY, Lin KC. Iatrogenic syndesmosis malreduction via clamp and screw placement. J Orthop
Trauma. 2013 Oct;27(10):e248-9. doi: 10.1097/BOT.0b013e3182a70221. PubMed PMID: 24060696. 

© 2016 American Academy of Orthopaedic Surgeons 2016 Orthopaedic In-Training Examination


SECTION 10: Trauma • 257

Question 204
How does timing of open reduction and internal fixation (ORIF) for tibial plateau fractures after
fasciotomy influence infection risk?

1. Increases risk for infection before fasciotomy closure


2. Increases risk for infection when performed during fasciotomy closure
3. Increases risk for infection after fasciotomy closure
4. Decreases risk for infection before fasciotomy closure
5. Timing does not influence infection risk

PREFERRED RESPONSE: 5

RECOMMENDED READINGS
Zura RD, Adams SB Jr, Jeray KJ, Obremskey WT, Stinnett SS, Olson SA; Southeastern Fracture
Consortium Foundation. Timing of definitive fixation of severe tibial plateau fractures with compartment
syndrome does not have an effect on the rate of infection. J Trauma. 2010 Dec;69(6):1523-6. doi: 10.1097/
TA.0b013e3181d40403. PubMed PMID: 20495494.

Shah SN, Karunakar MA. Early wound complications after operative treatment of high energy tibial
plateau fractures through two incisions. Bull NYU Hosp Jt Dis. 2007;65(2):115-9. PubMed PMID:
17581103.

Question 208
In the management of displaced periprosthetic distal femoral fractures, when comparing locked plating,
retrograde intramedullary nail (RIMN), nonlocking plating techniques, and nonsurgical treatment, locked
plating has been shown to

1. offer no advantage over nonsurgical treatment.


2. offer no advantage over conventional (nonlocked) plating.
3. demonstrate an increased trend toward nonunion when compared to RIMN.
4. demonstrate an increased trend toward malunion when compared to RIMN.
5. demonstrate a statistically significant increase in the need for secondary surgical procedures.

PREFERRED RESPONSE: 3

RECOMMENDED READINGS
Ristevski B, Nauth A, Williams DS, Hall JA, Whelan DB, Bhandari M, Schemitsch EH. Systematic
review of the treatment of periprosthetic distal femur fractures. J Orthop Trauma. 2014 May;28(5):307-12.
doi: 10.1097/BOT.0000000000000002. Review. PubMed PMID: 24149447.

Kolb W, Guhlmann H, Windisch C, Marx F, Koller H, Kolb K. Fixation of periprosthetic femur fractures
above total knee arthroplasty with the less invasive stabilization system: a midterm follow-up study. J
Trauma. 2010 Sep;69(3):670-6. doi: 10.1097/TA.0b013e3181c9ba3b. PubMed PMID: 20838138.

© 2016 American Academy of Orthopaedic Surgeons 2016 Orthopaedic In-Training Examination


258 • American Academy of Orthopaedic Surgeons

Figure 212a Figure 212b

Question 212
A 30-year-old woman undergoes a CT scanogram to assess for rotation after locked intramedullary (IM)
nailing of her right femur. Figures 212a and 212b, the resulting CT images, reveal

1. femoral anteversion of 11 degrees.


2. femoral anteversion of 19 degrees.
3. femoral retroversion of 11 degrees.
4. femoral retroversion of 19 degrees.
5. neutral version.

PREFERRED RESPONSE: 3

RECOMMENDED READINGS
Koerner JD, Patel NM, Yoon RS, Sirkin MS, Reilly MC, Liporace FA. Femoral version of the general
population: does "normal" vary by gender or ethnicity? J Orthop Trauma. 2013 Jun;27(6):308-11. doi:
10.1097/BOT.0b013e3182693fdd. PubMed PMID: 23032191.

Gardner MJ, Citak M, Kendoff D, Krettek C, Hüfner T. Femoral fracture malrotation caused by freehand
versus navigated distal interlocking. Injury. 2008 Feb;39(2):176-80. PubMed PMID: 17888433.

© 2016 American Academy of Orthopaedic Surgeons 2016 Orthopaedic In-Training Examination


SECTION 10: Trauma • 259

Question 218
A 24-year-old man has an open forearm fracture with comminution and no significant contamination. The
radius fracture is comminuted in the middle third of the radius, and there is a transverse fracture at the
middle of the ulna. While discussing the case details with the operating room charge nurse, which plate
should you use to treat the fracture?

1. One-third tubular plate


2. 3.5-mm reconstruction plate
3. 2.7-mm limited-contact dynamic compression plate (LC-DCP)
4. 4.5-mm LC-DCP
5. 3.5-mm LC-DCP

PREFERRED RESPONSE: 5

RECOMMENDED READINGS
Anderson LD, Sisk D, Tooms RE, Park WI 3rd. Compression-plate fixation in acute diaphyseal fractures
of the radius and ulna. J Bone Joint Surg Am. 1975 Apr;57(3):287-97. PubMed PMID: 1091653.

Moed BR, Kellam JF, Foster RJ, Tile M, Hansen ST Jr. Immediate internal fixation of open fractures of the
diaphysis of the forearm. J Bone Joint Surg Am. 1986 Sep;68(7):1008-17. PubMed PMID: 3745238.

Jones JA. Immediate internal fixation of high-energy open forearm fractures. J Orthop Trauma.
1991;5(3):272-9. PubMed PMID: 1941308.

Question 219
What is a statistically significant predictor of postsurgical infection for patients with pelvic and acetabular
fractures?

1. No presurgical angiography embolization


2. No blood transfusion
3. Body mass index (BMI) lower than 30
4. Obesity with leukocytosis
5. Interfacility transfer

PREFERRED RESPONSE: 4

RECOMMENDED READINGS
Sagi HC, Dziadosz D, Mir H, Virani N, Olson C. Obesity, leukocytosis, embolization, and injury severity
increase the risk for deep postoperative wound infection after pelvic and acetabular surgery. J Orthop
Trauma. 2013 Jan;27(1):6-10. doi: 10.1097/BOT.0b013e31825cf382. PubMed PMID: 23263468.

Manson TT, Perdue PW, Pollak AN, OʼToole RV. Embolization of pelvic arterial injury is a risk factor
for deep infection after acetabular fracture surgery. J Orthop Trauma. 2013 Jan;27(1):11-5. doi: 10.1097/
BOT.0b013e31824d96f6. PubMed PMID: 22495529.
© 2016 American Academy of Orthopaedic Surgeons 2016 Orthopaedic In-Training Examination
260 • American Academy of Orthopaedic Surgeons

Question 224
A 32-year-old man fell and sustained an elbow hyperextension injury with a posterior dislocation that was
reduced in the emergency department. Radiographs revealed a displaced 30% radial head fragment and a
10% coronoid fracture. An examination under anesthesia reveals both varus and valgus instability. What
should be repaired?

1. Lateral collateral ligament (LCL) and radial head fixation


2. Medial collateral ligament (MCL) and LCL
3. MCL and LCL with fixation of the radial head and coronoid fractures
4. MCL and LCL and radial head fracture fixation
5. MCL and radial head fracture

PREFERRED RESPONSE: 4

RECOMMENDED READINGS
Papatheodorou LK, Rubright JH, Heim KA, Weiser RW, Sotereanos DG. Terrible triad injuries of the
elbow: does the coronoid always need to be fixed? Clin Orthop Relat Res. 2014 Jul;472(7):2084-91. doi:
10.1007/s11999-014-3471-7. PubMed PMID: 24474322.

Mathew PK, Athwal GS, King GJ. Terrible triad injury of the elbow: current concepts. J Am Acad Orthop
Surg. 2009 Mar;17(3):137-51. Review. PubMed PMID: 19264707.

Question 226
An 82-year-old woman who underwent left total knee arthroplasty 5 years ago now has left knee pain and
an inability to bear weight after falling onto her flexed left knee. Plain radiographs show a comminuted
supracondylar femur fracture 6 cm proximal to the femoral component with no sign of component
loosening. The fracture is displaced, shortened 2 cm, and aligned in 15 degrees of varus and 25 degrees of
extension. Closed reduction is performed, but the fracture remains displaced. She had no antecedent knee
pain. What is the best treatment at this time?

1. Surgical fixation with a locking plate or intramedullary nail (IMN)


2. Revision surgery with a stemmed revision femoral component combined with a distal
femoral allograft
3. Revision surgery with a long-stem femoral component
4. Revision surgery with a distal femur-replacing implant
5. Placement of a long-leg cast in 30 degrees of knee flexion

PREFERRED RESPONSE: 1

© 2016 American Academy of Orthopaedic Surgeons 2016 Orthopaedic In-Training Examination


SECTION 10: Trauma • 261

RECOMMENDED READINGS
Nauth A, Ristevski B, Bégué T, Schemitsch EH. Periprosthetic distal femur fractures: current concepts.
J Orthop Trauma. 2011 Jun;25 Suppl 2:S82-5. doi: 10.1097/BOT.0b013e31821b8a09. Review. PubMed
PMID: 21566481.

Gliatis J, Megas P, Panagiotopoulos E, Lambiris E. Midterm results of treatment with a retrograde nail for
supracondylar periprosthetic fractures of the femur following total knee arthroplasty. J Orthop Trauma.
2005 Mar;19(3):164-70. PubMed PMID: 15758669.

Kolb W, Guhlmann H, Windisch C, Marx F, Koller H, Kolb K. Fixation of periprosthetic femur fractures
above total knee arthroplasty with the less invasive stabilization system: a midterm follow-up study. J
Trauma. 2010 Sep;69(3):670-6. doi: 10.1097/TA.0b013e3181c9ba3b. PubMed PMID: 20838138.

Question 230
Functional outcomes after patella fracture fixation demonstrate that patients

1. have a high hardware removal rate.


2. have no limitations in maximum strength.
3. have no difficulty in achieving range of motion.
4. have outcome scores equal to scores for healthy knees.
5. with retained hardware have minimal pain.

PREFERRED RESPONSE: 1

RECOMMENDED READINGS
LeBrun CT, Langford JR, Sagi HC. Functional outcomes after operatively treated patella fractures. J
Orthop Trauma. 2012 Jul;26(7):422-6. doi: 10.1097/BOT.0b013e318228c1a1. PubMed PMID: 22183197.

Bayar A, Sener E, Keser S, Meray J, Simşek A, Senköylü A. What leads to unfavourable Cybex test
results for quadriceps power after modified tension band osteosynthesis of patellar fractures? Injury. 2006
Jun;37(6):520-4. PubMed PMID: 16574121.

© 2016 American Academy of Orthopaedic Surgeons 2016 Orthopaedic In-Training Examination


262 • American Academy of Orthopaedic Surgeons

Figure 235

Question 235
Figure 235 is the clinical photograph after performing a locked right femoral retrograde nail procedure for
a midshaft femoral fracture. These findings are consistent with a/an

1. decrease in femoral anteversion on the right during surgery.


2. decrease in femoral anteversion on the left during surgery.
3. malreduction externally rotating the distal right femur during nailing.
4. malreduction internally rotating the distal right femur during nailing.
5. acceptable outcome.

PREFERRED RESPONSE: 4

RECOMMENDED READINGS
Dimitriou D, Tsai TY, Yue B, Rubash HE, Kwon YM, Li G. Side-to-side variation in normal femoral
morphology: 3D CT analysis of 122 femurs. Orthop Traumatol Surg Res. 2016 Feb;102(1):91-7. doi:
10.1016/j.otsr.2015.11.004. PubMed PMID: 26867707.

Karaman O, Ayhan E, Kesmezacar H, Seker A, Unlu MC, Aydingoz O. Rotational malalignment after
closed intramedullary nailing of femoral shaft fractures and its influence on daily life. Eur J Orthop Surg
Traumatol. 2014 Oct;24(7):1243-7. doi: 10.1007/s00590-013-1289-8. PubMed PMID: 23934503.

Espinoza C, Sathy AK, Moore DS, Starr AJ, Reinert CM. Use of inherent anteversion of an intramedullary
nail to avoid malrotation in femur fractures. J Orthop Trauma. 2014 Feb;28(2):e34-8. doi: 10.1097/
BOT.0b013e318298e48c. PubMed PMID: 23689227.

© 2016 American Academy of Orthopaedic Surgeons 2016 Orthopaedic In-Training Examination


SECTION 10: Trauma • 263

Question 240
When comparing outcomes among elderly patients who undergo internal screw fixation vs
hemiarthroplasty for treatment of displaced femoral neck fractures, internal screw fixation is associated
with

1. higher revision surgery rates.


2. less pain at 12 months.
3. better satisfaction at 12 months.
4. better quality of life at 12 months.
5. lower revision surgery rates.

PREFERRED RESPONSE: 1

RECOMMENDED READINGS
Gjertsen JE, Vinje T, Engesaeter LB, Lie SA, Havelin LI, Furnes O, Fevang JM. Internal screw fixation
compared with bipolar hemiarthroplasty for treatment of displaced femoral neck fractures in elderly
patients. J Bone Joint Surg Am. 2010 Mar;92(3):619-28. doi: 10.2106/JBJS.H.01750. PubMed PMID:
20194320.

Gjertsen JE, Vinje T, Lie SA, Engesaeter LB, Havelin LI, Furnes O, Fevang JM. Patient satisfaction, pain,
and quality of life 4 months after displaced femoral neck fractures: a comparison of 663 fractures treated
with internal fixation and 906 with bipolar hemiarthroplasty reported to the Norwegian Hip Fracture
Register. Acta Orthop. 2008 Oct;79(5):594-601. doi: 10.1080/17453670810016597. PubMed PMID:
18839364.

© 2016 American Academy of Orthopaedic Surgeons 2016 Orthopaedic In-Training Examination


264 • American Academy of Orthopaedic Surgeons

Figure 250a Figure 250b

Question 250
Figures 250a and 250b are the radiographs of a 45-year-old man who sustained a comminuted left distal
femoral extra-articular fracture. Which method was used to reduce and fix the fracture?

1. An absolutely stable construct demonstrating primary bone healing


2. An indirect reduction and healing by secondary intention
3. A relatively stable construct demonstrating primary bone healing
4. A relatively stable construct and secondary bone healing
5. Direct reduction and secondary bone healing

PREFERRED RESPONSE: 4

RECOMMENDED READINGS
Sfeir C, Ho L, Doll BA, Azari K, Hollinger JO. Fracture repair. In: Lieberman JR, Friedlaender GE, eds.
Bone Regeneration and Repair: Biology and Clinical Applications. Totowa, New Jersey: Humana Press;
2005:21-43.

Perren SM. Fracture healing: fracture healing understood as the result of a fascinating cascade of physical
and biological interactions. Part I. An Attempt to Integrate Observations from 30 Years AO Research. Acta
Chir Orthop Traumatol Cech. 2014;81(6):355-64. Review. PubMed PMID: 25651289.

© 2016 American Academy of Orthopaedic Surgeons 2016 Orthopaedic In-Training Examination


SECTION 10: Trauma • 265

Question 259
A 64-year-old man is seen in the emergency department after tripping over a rug. An examination reveals
a midshaft femoral fracture through a lytic lesion. Laboratory studies including a complete blood count,
metabolic panel to assess renal and hepatic function, urinalysis, prostate specific antigen, and protein
electrophoresis are conducted; all values are within defined limits. What is the best next step?

1. MRI of the lesion; CT scans of the chest, abdomen, and pelvis; and an open biopsy for
later planning
2. MRI of the lesion; CT scans of the chest, abdomen, and pelvis; and biopsy of the lesion at the
time of fixation
3. MRI of the lesion; CT scan and positron emission tomography (PET)-CT scans of the chest,
abdomen, and pelvis; biopsy of the lesion at the time of surgery
4. CT scans of the lesion, chest, abdomen, and pelvis and a closed biopsy with an oncology
review for method of fixation
5. CT scan of the lesion; CT scan and PET-CT scans of the chest, abdomen, and pelvis; biopsy of
the lesion at the time of surgery

PREFERRED RESPONSE: 5

RECOMMENDED READINGS
Scolaro JA, Lackman RD. Surgical management of metastatic long bone fractures: principles and
techniques. J Am Acad Orthop Surg. 2014 Feb;22(2):90-100. doi: 10.5435/JAAOS-22-02-90. PMID:
24486755.

Hahn S, Heusner T, Kümmel S, Köninger A, Nagarajah J, Müller S, Boy C, Forsting M, Bockisch A,


Antoch G, Stahl A. Comparison of FDG-PET/CT and bone scintigraphy for detection of bone metastases
in breast cancer. Acta Radiol. 2011 Nov 1;52(9):1009-14. doi: 10.1258/ar.2011.100507. PubMed PMID:
21969709.

Question 261
What is a risk factor for distal femoral fracture locked plate fixation failure?

1. Closed fracture
2. Body mass index (BMI) lower than 25
3. Diabetes
4. Longer plate length
5. Osteopenia

PREFERRED RESPONSE: 3

© 2016 American Academy of Orthopaedic Surgeons 2016 Orthopaedic In-Training Examination


266 • American Academy of Orthopaedic Surgeons

RECOMMENDED READINGS
Ricci WM, Streubel PN, Morshed S, Collinge CA, Nork SE, Gardner MJ. Risk factors for failure
of locked plate fixation of distal femur fractures: an analysis of 335 cases. J Orthop Trauma. 2014
Feb;28(2):83-9. doi: 10.1097/BOT.0b013e31829e6dd0. PubMed PMID: 23760176.

Kregor PJ, Stannard JA, Zlowodzki M, Cole PA. Treatment of distal femur fractures using the less
invasive stabilization system: surgical experience and early clinical results in 103 fractures. J Orthop
Trauma. 2004 Sep;18(8):509-20. PubMed PMID: 15475846.

Figure 266a Figure 266b Figure 266c

Question 266
Figures 266a through 266c are the radiographs and clinical photograph of a healthy 30-year-old man who
arrived at the hospital with an unstable ankle fracture. The surgeon attempted to reduce and splint the
injury; this failed, and a second attempt resulted in the resultant radiographs. What is the best next step?

1. Eventual surgery when the swelling improves


2. Closed reduction and percutaneous fixation
3. Indirect reduction and external fixation
4. Direct reduction and external fixation
5. Direct reduction and absolute stable fixation

PREFERRED RESPONSE: 3

© 2016 American Academy of Orthopaedic Surgeons 2016 Orthopaedic In-Training Examination


SECTION 10: Trauma • 267

RECOMMENDED READINGS
Carroll EA, Koman LA. External fixation and temporary stabilization of femoral and tibial trauma. J Surg
Orthop Adv. 2011 Spring;20(1):74-81. PubMed PMID: 21477538.

Strauss EJ, Petrucelli G, Bong M, Koval KJ, Egol KA. Blisters associated with lower-extremity fracture:
results of a prospective treatment protocol. J Orthop Trauma. 2006 Oct;20(9):618-22. PubMed PMID:
17088664.

Question 269
Following intramedullary nail fixation for a diaphyseal femur fracture, what is the maximum acceptable
difference in rotational malreduction between the surgical and contralateral legs for femoral version?

1. 5 degrees
2. 10 degrees
3. 15 degrees
4. 20 degrees
5. 25 degrees

PREFERRED RESPONSE: 3

RECOMMENDED READINGS
Ayalon OB, Patel NM, Yoon RS, Donegan DJ, Koerner JD, Liporace FA. Comparing femoral version
after intramedullary nailing performed by trauma-trained and non-trauma trained surgeons: is there a
difference? Injury. 2014 Jul;45(7):1091-4. doi: 10.1016/j.injury.2014.01.024. PubMed PMID: 24630333.

Omar M, Suero EM, Hawi N, Decker S, Krettek C, Citak M. Preoperative virtual reduction reduces
femoral malrotation in the treatment of bilateral femoral shaft fractures. Arch Orthop Trauma Surg. 2015
Oct;135(10):1385-9. doi: 10.1007/s00402-015-2285-8. PubMed PMID: 26185053.

© 2016 American Academy of Orthopaedic Surgeons 2016 Orthopaedic In-Training Examination


2016 Orthopaedic In-Training Examination—Question Numbers by Content Domain

(*Excluded from Scoring: 9, 69, 164, 253, 259, 268)

Basic Science (32)


*9, 22, 24, 36, 52, 61, 67, 72, 84, 87, 100, 106, 115, 118, 126, 134, 142, 148, 153, 155, 166, 170,
176, 186, 192, 198, 203, 207, 227, 237, 255, 274

Foot and Ankle (22)


1, 16, 31, 46, 54, *69, 80, 93, 105, 117, 130, 147, 158, 179, 193, 200, 211, 216, 222, 251, 260,
*268

Hand (22)
8, 15, 28, 39, 48, 58, 65, 75, 88, 96, 103, 133, 140, 175, 228, 232, 245, 246, 256, 263, 267, 271

Hip & Knee Reconstruction (28)


5, 20, 30, 44, 50, 59, 76, 98, 108, 121, 127, 136, 139, 144, 154, 165, 177, 191, 197, 206, 214,
215, 225, 238, 242, 243, 248, 258

Oncology (22)
4, 17, 25, 37, 47, 60, 71, 79, 90, 109, 119, 151, 156, 172, 187, 217, 241, 254, 257, 262, 270, 273

Pediatrics (35)
3, 13, 21, 29, 33, 38, 43, 49, 56, 64, 70, 74, 81, 86, 92, 95, 104, 113, 122, 129, 138, 143, 152,
162, 169, 174, 183, 189, 195, 210, 223, 229, 239, 247, 265

Shoulder and Elbow (21)


12, 26, 34, 41, 53, 66, 85, 89, 101, 116, 131, 149, 167, 178, 185, 202, 213, 221, 234, *253, 272

Spine (20)
7, 19, 27, 35, 45, 55, 68, 77, 94, 110, 124, 141, 163, 180, 194, 199, 205, 220, 233, 249

Sports Medicine (23)


11, 23, 42, 51, 62, 83, 99, 112, 125, 146, 159, 171, 181, 184, 188, 201, 209, 231, 236, 244, 252,
264, 275

Trauma (50)
2, 6, 10, 14, 18, 32, 40, 57, 63, 73, 78, 82, 91, 97, 102, 107, 111, 114, 120, 123, 128, 132, 135,
137, 145, 150, 157, 160, 161, *164, 168, 173, 182, 190, 196, 204, 208, 212, 218, 219, 224, 226,
230, 235, 240, 250, *259, 261, 266, 269

You might also like